Google Groupes n'accepte plus les nouveaux posts ni abonnements Usenet. Les contenus de l'historique resteront visibles.

Photons et théorie

16 vues
Accéder directement au premier message non lu

Live it up

non lue,
3 oct. 2002, 18:18:1503/10/2002
à
Bonjour,

Ne connaissant la physique qu'au travers de quelques ouvrages de
vulgarisation et n'ayant pas le temps de me mettre à l'étudier, j'ai
quelques questions (qui sont certainement truffées d'erreurs dû à la
vulgarisation massive des sciences-physiques) à vous soumettre :

1) Puisqu'il va à la vitesse de la lumière, le temps du photon ne s'écoule
pas. Comment peut'il perdre ou gagner de l'énergie si son temps ne passe
plus ? Pour passer d'un état à un autre il faut bien que du temps (en
l'occurence celui du photon) se soit écoulé entre ces 2 états.

2) J'ai lu que pour atteindre la vitesse de la lumière il faudrait une
energie infini (donc une masse infini). Pourrait'on concevoir que le photon
ait une masse infini mais, son temps ne passant plus, il ne pourrait pas
emettre des ondes gravitationnelles ? Ou en d'autre termes que le photon a
une masse indetectable par un appareil de mesure...

3) Une dernière... Il me semble qu'aucune théorie physique n'a comme
postulat que "L'espace et le temps sont continus". Est-ce que je me trompe ?
Si non : Cela est-il tellement evident qu'il n'est pas besoin de le postuler
ou bien cette affirmation est un théorème résultant des théories (là, y'a
peut de chances quand même) ? A t'on une preuve de la continuité de l'espace
temps ? N'existe t'il pas des modèles/théorie où l'espace et/ou le temps
sont quantifiés ?

J'attends avec impatience vos réponses et vos explications quant-à mes
questions et erreurs de compréhension.

Cordialement,
Bertrand.

srp

non lue,
3 oct. 2002, 23:30:4103/10/2002
à
Live it up a écrit :

>
> Bonjour,
>
> Ne connaissant la physique qu'au travers de quelques ouvrages de
> vulgarisation et n'ayant pas le temps de me mettre à l'étudier, j'ai
> quelques questions (qui sont certainement truffées d'erreurs dû à la
> vulgarisation massive des sciences-physiques) à vous soumettre :
>
> 1) Puisqu'il va à la vitesse de la lumière, le temps du photon ne
> s'écoule pas. Comment peut'il perdre ou gagner de l'énergie si son
> temps ne passe plus ? Pour passer d'un état à un autre il faut bien
> que du temps (en l'occurence celui du photon) se soit écoulé entre
> ces 2 états.

L'idée que le temps s'arrête lorsque la vitesse de la lumière
est atteinte est une conclusion de la relativité générale. Si
la relativité générale s'avérait fausse en fin de compte, alors
cette conclusion, qui semble passablement illogique, j'en conviens
avec vous, tomberait en même temps.



> 2) J'ai lu que pour atteindre la vitesse de la lumière il faudrait
> une energie infini (donc une masse infini). Pourrait'on concevoir
> que le photon ait une masse infini mais, son temps ne passant plus,
> il ne pourrait pas emettre des ondes gravitationnelles ? Ou en
> d'autre termes que le photon a une masse indetectable par un
> appareil de mesure...

Il est connu et vérifié que le photon n'a pas de masse, encore moins
une masse infinie.

D'autre part, il est aussi connu et vérifié qu'il est impossible
dans un champ magnétique d'accélérer un électron (par exemple),
pour lui faire atteindre la vitesse de la lumière. On peut le
faire s'en approcher de très près, mais pas l'atteindre, peu
importe l'énergie qu'on y met.

Il semble bien que le phénomène soit réel, du moins pour les
cas d'accélération dans un champ magnétique.

> 3) Une dernière... Il me semble qu'aucune théorie physique n'a
> comme postulat que "L'espace et le temps sont continus". Est-ce
> que je me trompe ?

L'espace-temps de la RG est continu par définition.

> Si non : Cela est-il tellement evident qu'il n'est pas besoin de
> le postuler ou bien cette affirmation est un théorème résultant
> des théories (là, y'a peut de chances quand même) ? A t'on une
> preuve de la continuité de l'espace temps ?

L' "espace-temps" tel qu'il a été défini par Minkowski, suite
aux explorations de Poincaré, est un modèle géométrique. Il n'a
pas besoin de preuve puisqu'il est posé en postulat.

> N'existe t'il pas des modèles/théorie où l'espace et/ou le temps
> sont quantifiés ?

Aucun qui ait "pris", ma connaissance. Le seul modèle accepté
actuellement est celui de la RG.



> J'attends avec impatience vos réponses et vos explications quant-à
> mes questions et erreurs de compréhension.

Des questions impliquent une recherche de compréhension. Les erreurs
de parcours en cours de cheminement n'ont donc aucune importance.

--
André Michaud
Service de Recherche Pédagogique http://pages.globetrotter.net/srp/

Gérard Samouillan

non lue,
4 oct. 2002, 03:39:5504/10/2002
à

> 1) Puisqu'il va à la vitesse de la lumière, le temps du photon ne s'écoule
> pas. Comment peut'il perdre ou gagner de l'énergie si son temps ne passe
> plus ? Pour passer d'un état à un autre il faut bien que du temps (en
> l'occurence celui du photon) se soit écoulé entre ces 2 états.

Pluôt que de dire que son temps ne passe plus, il est plus rigoureux
de dire que sa durée de vie propre est nulle...
Ou tout simplement, qu'il n'existe pas .
La physique nous donne de nombreux exemples d'observables qui ne
sont crés que par l'observateur...un arc en ciel, par exemple.
L'idée que le photon serait un corpuscule physique qui aurait une
existence propre est à l'origine d toutes les contradictions que l'on peut
formuler à son sujet.
Bien sur, on peut créer des théories qui manient le photon en tant
qu'entité,
comme on pourrait créer des théories qui manient l'arc en ciel, à l'aide de
postulats add hoc.
Par exemple: un arc en ciel se déplace avec l'observateur et se maintient
toujours à la même distance de celui ci....
Il en est de même du photon, quand on dit que sa vitesse est la même
pour tout observateur...cela montre tout simplement que ce n'est pas un
objet réel.
Bien sûr, on pourrait discutter de ce qui est réel...mais c'est une autre
histoire...

> 2) J'ai lu que pour atteindre la vitesse de la lumière il faudrait une
> energie infini (donc une masse infini). Pourrait'on concevoir que le
photon
> ait une masse infini mais, son temps ne passant plus, il ne pourrait pas
> emettre des ondes gravitationnelles ? Ou en d'autre termes que le photon a
> une masse indetectable par un appareil de mesure...

la réponse est dans la réponse précédente....

GS


>
.
>
>
>


Didier Lauwaert

non lue,
4 oct. 2002, 05:16:0104/10/2002
à
"Live it up" <live...@wanadoo.fr> wrote in message news:<anifn1$jkd$1...@news-reader11.wanadoo.fr>...
> Bonjour,
>

snip, le (1) et le (2), je ne sais trop comment y répondre :o)
Les questions sont en fait très mal posées et il faudrait
pratiquement expliquer les principes relativistes, quantiques, etc...
Cela prendrait des Km d'explications.

D'autres y arriveront peut-être mieux que moi.
Je m'attaque donc seulement au (3).

>
> 3) Une dernière... Il me semble qu'aucune théorie physique n'a comme
> postulat que "L'espace et le temps sont continus". Est-ce que je me trompe ?

Oui. Toutes les théories orthodoxes ont ce postulat :o)

> Si non : Cela est-il tellement evident qu'il n'est pas besoin de le postuler

disons qu'on ne le précise que rarement vu que toutes les théories classiques
le considère comme allant de soit.

> ou bien cette affirmation est un théorème résultant des théories (là, y'a

non.

> peut de chances quand même) ? A t'on une preuve de la continuité de l'espace

non.

> temps ? N'existe t'il pas des modèles/théorie où l'espace et/ou le temps
> sont quantifiés ?

Très bonne question cette fois.

Lorsqu'on essaye de combiner la RG (relativité générale) et la
physique quantique, on tombe sur des problèmes épouvantables
(en particulier l'apparition de résultats infinis rédhibitoires).

Une analyse fine des problèmes (j'ai lu quelques bons articles,
assez techniques, sur ce point) montre que le problème
ne peut, pratiquement, être résolu que de deux manières.
- utiliser des "objets étendus" (au lieu d'objets ponctuels
comme le considère la physique quantique "traditionnelle").
Par exemple les "cordes quantiques" (théorie des cordes).
- utiliser une structure différente de l'espace-temps
par exemple, un espace-temps fractal ou une géométrie
"non commutative".

Aucune de ces approches n'a malheureusement encore abouti
(complexité des développements mathématiques dont les
outils restent parfois à inventer, absence de résultats
expérimentaux permettant de départager les modèles,...).

J'aime assez l'approche par les géométries non commutatives.
Malheureusement, il n'y a pas approche plus difficile à
vulgariser que celle là !!!!
Comme la géométrie envisagée est non "classique", on
ne peut en trouver une correspondance "dessinée".
La seule manière de la décrire est l'approche
algébrique. Cela en fait une théorie extraimement
abstraite.

srp

non lue,
4 oct. 2002, 12:52:3904/10/2002
à
newsgroups a écrit :
>
> Bonjour,
> une petite question de rien du tout:

>
> "> D'autre part, il est aussi connu et vérifié qu'il est impossible
> > dans un champ magnétique d'accélérer un électron (par exemple),
> > pour lui faire atteindre la vitesse de la lumière. On peut le
> > faire s'en approcher de très près, mais pas l'atteindre, peu
> > importe l'énergie qu'on y met.
> >
> > Il semble bien que le phénomène soit réel, du moins pour les
> > cas d'accélération dans un champ magnétique.
>
> tu pourrais m'en dire plus sur l'accélération d'une charge par
> un champ _magnétique_ :-) ?

Il faut d'abord comprendre que lorsqu'on associe les conclusions de
de Broglie au sujet de la structure dynamique interne du photon à
la découverte de Abraham et kaufmann au sujet de la distinction à
faire entre l'inertie longitudinale et l'inertie transversale,
s'impose la conclusion que le photon serait en équilibre à la
vitesse de la lumière dans le vide simplement parce qu'une partie
de son énergie, la moitié à vrai dire, par symétrie, sert simplement
à "pousser" l'autre moitié, celle qui est quantifiée orthogonalement
sous forme électromagnétique.

Lorsqu'on parle d'un électron, on parle au départ d'une quantité
d'énergie qui est totalement quantifiée, contrairement au photon.
Son niveau d'énergie le plus bas, c'est à dire son niveau de
moindre action, est de .511 MeV, et à ce niveau, il est immobile
dans l'espace, vitesse zero.

Nous savons que le champ magnétique d'un électro-aimant est constitué
de la même substance que l'électron. Il est constitué de la somme
des aspects magnétiques des électrons qui sont forcés de circuler
dans le bobinage en alignant leur spin en parallèle.

Lorsqu'on place un électron dans un tel champ magnétique, le
niveau d'énergie local ambiant combiné à celui de l'électron
dépasse .511 MeV. Tout se passe alors comme si l'énergie en
excès qui interagit avec celle de l'électron cherchait un
équilibre similaire à celui du photon dans le vide.

L'énergie locale qui interagit avec l'électron semble donc tendre
à se diviser en deux, une partie qui s'associe à la quantité
quantifiée de l'électron (augmentant sa masse mesurable) et
l'autre moitié qui s'exprime sous forme de vitesse de l'électron
dans l'espace (quantité de mouvement non quantifiée).

Autrement dit, l'énergie en excès se répartirait à parts égales
entre l'espace normal et la partie électromagnétique quantifiée
orthogonalement, exactement comme dans un photon normal, avec
cette différence que du côté quantifié, s'ajoute le boulet inerte
de l'électron que le "pseudo-photon" doit maintenant trainer.

Plus le champ magnétique sera puissant, plus la vitesse de l'électron
deviendra grande, mais il y aura toujours cet excès d'énergie
du côté quantifié (les 8.186 x 10^-14 joule inertes de l'électron).

Un photon de .6 MeV, par exemple file à la vitesse de la lumière
.3 MeV qui pousee .3 MeV. Équilibre.

Un électron dont l'énergie totale est .6 MeV va à la vitesse
que permet .045 MeV qui pousse .556 MeV. Déséquilibre.

À tout jamais, un électron est condamné à ne jamais atteindre
la vitesse de la lumière dans un champ magnétique à cause du
déséquilibre permanent de .511 MeV du côté quantifié de
l'événement.

newsgroups

non lue,
4 oct. 2002, 13:34:5904/10/2002
à
Une remarque:
Tu dis

"> À tout jamais, un électron est condamné à ne jamais atteindre
> la vitesse de la lumière dans un champ magnétique à cause du
> déséquilibre permanent de .511 MeV

ou tout simplement parce qu'un champ magnétique ne peut pas
fournir du travail à une charge, non ?
Je laisse le reste de ce que tu dis aux amateurs de crypto-physique.


François.

srp

non lue,
4 oct. 2002, 16:16:0804/10/2002
à
newsgroups a écrit :

>
> Une remarque:
> Tu dis
>
> "> À tout jamais, un électron est condamné à ne jamais atteindre
> > la vitesse de la lumière dans un champ magnétique à cause du
> > déséquilibre permanent de .511 MeV
>
> ou tout simplement parce qu'un champ magnétique ne peut pas
> fournir du travail à une charge, non ?

Vous oubliez que l'électron est de manière vérifiée une particule
"électromagnétique", pas une simple "charge", comme vous dites,
qui est, soit dit en passant, une caractéristique de l'électron
que personne n'a d'ailleurs encore réussi à comprendre, sauf les
pontificateurs, bien sûr.

> Je laisse le reste de ce que tu dis aux amateurs de crypto-physique.

Je dirais, aux amateurs de mécanique.

Perso, je laisse les présomptions au sujet du comportement de
la "charge" aux pontificateurs en attendant qu'elle soit
comprise, et les maths aux amateurs de mathématique.

François Guillet

non lue,
5 oct. 2002, 05:59:0405/10/2002
à

"Gérard Samouillan" <gerard.s...@free.fr> a écrit dans le message de
news: 3d9d4410$0$6106$626a...@news.free.fr...

> Pluôt que de dire que son temps ne passe plus, il est plus rigoureux
> de dire que sa durée de vie propre est nulle...
> Ou tout simplement, qu'il n'existe pas .
> La physique nous donne de nombreux exemples d'observables qui ne
> sont crés que par l'observateur...un arc en ciel, par exemple.
> L'idée que le photon serait un corpuscule physique qui aurait une
> existence propre est à l'origine d toutes les contradictions que l'on
peut
> formuler à son sujet.
> Bien sur, on peut créer des théories qui manient le photon en tant
> qu'entité,
> comme on pourrait créer des théories qui manient l'arc en ciel, à l'aide
de
> postulats add hoc.
> Par exemple: un arc en ciel se déplace avec l'observateur et se maintient
> toujours à la même distance de celui ci....
> Il en est de même du photon, quand on dit que sa vitesse est la même
> pour tout observateur...cela montre tout simplement que ce n'est pas un
> objet réel.
> Bien sûr, on pourrait discutter de ce qui est réel...mais c'est une autre
> histoire...

Dire que ce n'est "pas un objet réel", qu'il "n'existe pas"... ne répond pas
à la question.
Le "photon" est, contrairement à "arc en ciel", une terminologie issue non
pas de l'expérience ou du sens commun, mais de la physique. Si elle ne le
renie pas, alors il faudrait qu'elle le définisse autrement que par la
négative.

FG

Pierre

non lue,
5 oct. 2002, 07:39:1805/10/2002
à
>L'idée que le temps s'arrête lorsque la vitesse de la lumière
>est atteinte est une conclusion de la relativité générale.

Non

C' est une conclusion déja présente dans la relativité restreinte ou le 2eme
postulat de la RR conduit à considérer que des horloges fixes dans deux
référentiels inertiels R et R' ayant une vitesse relative entre eux , ne
marquent pas le meme temps.

Le rapport du temps propre (celui du réferentiel R ou l'objet est au repos) au
temps impropre (celui d'un autre référentiel inertiel R' ou le meme objet est
en mouvement ) est donné par la RR

Temps impropre= Temps propre /sqrt (1-v²/c²)
ou v est la vitesse relative (d'éloignement ou de rapprochement ) des deux
référentiels. D'aprés cette formule, on a toujours

Temps propre < Temps impropre

Dans le cas ou l'objet a une vitesse proche de la lumière dans R' on voit que
l'on a

Temps propre <<< Temps impropre

C'est à dire qu à un intervalle de 1 s entre deux évènements relatifs à l'objet
( temps impropre) dans le référentiel R' ou l'objet est en mouvement
correspond peut etreun intervalle de 0.001 s (temps propre) entre les deux
memes évènements dans R ou l'objet est au repos.

Si v=c (cas du photon) le temps propre est zero quelque soit le temps impropre.


>L' "espace-temps" tel qu'il a été défini par Minkowski, suite
>aux explorations de Poincaré, est un modèle géométrique. Il n'a
>pas besoin de preuve puisqu'il est posé en postulat.

NON. L'espace temps n'est qu' une reformulation géométrique de la partie
cinématique de la RR. Ce n'est pas un postulat.


srp

non lue,
5 oct. 2002, 09:30:2605/10/2002
à
Pierre a écrit :

>
> >L'idée que le temps s'arrête lorsque la vitesse de la lumière
> >est atteinte est une conclusion de la relativité générale.
>
> Non
>
> C' est une conclusion déja présente dans la relativité restreinte
> ou le 2eme postulat de la RR conduit à considérer que des horloges
> fixes dans deux référentiels inertiels R et R' ayant une vitesse
> relative entre eux , ne marquent pas le meme temps.
>
> Le rapport du temps propre (celui du réferentiel R ou l'objet est
> au repos) au temps impropre (celui d'un autre référentiel inertiel
> R' ou le meme objet est en mouvement ) est donné par la RR
>
> Temps impropre= Temps propre /sqrt (1-v²/c²)
> ou v est la vitesse relative (d'éloignement ou de rapprochement )
> des deux référentiels. D'aprés cette formule, on a toujours
>
> Temps propre < Temps impropre
>
> Dans le cas ou l'objet a une vitesse proche de la lumière dans R'
> on voit que l'on a
>
> Temps propre <<< Temps impropre
>
> C'est à dire qu à un intervalle de 1 s entre deux évènements relatifs
> à l'objet ( temps impropre) dans le référentiel R' ou l'objet est en
> mouvement correspond peut etreun intervalle de 0.001 s (temps propre)
> entre les deux memes évènements dans R ou l'objet est au repos.
>
> Si v=c (cas du photon) le temps propre est zero quelque soit le temps
> impropre.

Vous avez raison.

C'est une conclusion de la RR. Je confondais. Je dois dire que je
ne m'y réfère jamais personnellement, tout comme à la RG, car
plusieurs constats m'ont démontré que certains aspects des deux
théories sont faux.

Lorsqu'ils s'agit de conclusions qui impliquent les deux, je ne
me donne pas toujours la peine de creuser ma mémoire pour remonter
à la source spécifique.

Dans le cas du ralentissement du temps, puisque la supposé preuve
que le temps ralenti avec avec l'augmentation de la "courbure"
de l'espace-temps (RG), que l'augmentation de fréquence des
photons émis par le césium dans des horloges atomiques amenées
en altitudes, et qui était supposée prouver la RG de manière
définitive, peut s'expliquer autrement (ce qui falsifie en même
temps l'idée que le temps peut varier avec la vitesse), c'est la
RG qui m'est venue à l'esprit en premier.

Quant à l'idée que le temps s'arrête à la vitesse de la lumière,
il s'agit d'un non-sens à mes yeux, extrapolation irrationnelle
fondée sur la contraction de Lorentz.

Tout ne peut arriver qu'au moment présent, inexorablement, quoi
qu'on en dise.

> >L' "espace-temps" tel qu'il a été défini par Minkowski, suite
> >aux explorations de Poincaré, est un modèle géométrique. Il n'a
> >pas besoin de preuve puisqu'il est posé en postulat.
>
> NON. L'espace temps n'est qu' une reformulation géométrique de la
> partie cinématique de la RR. Ce n'est pas un postulat.

C'est un postulat en ce qui concerne la réalité physique, car
toutes les supposées preuves de cette courbures sont explicables
autrement, sans compter qu'elle est directement falsifiée par
l'incapacité de la RG à rendre compte des trajectoires hyperboliques
des sondes Pioneer 10 et 11, et de l'anomalie inverse de la
trajectoire de la sonde Mariner 10 lorsqu'elle frôla Mercure
en 1975.

ricky

non lue,
5 oct. 2002, 18:34:2905/10/2002
à
bonjour

> en altitudes, et qui était supposée prouver la RG de manière
> définitive, peut s'expliquer autrement (ce qui falsifie en même
> temps l'idée que le temps peut varier avec la vitesse),

s 'explqiue comment ?
et comment relier cette explication aux autrs preuves fournies par les
accelerateurs de particules ?

> Quant à l'idée que le temps s'arrête à la vitesse de la lumière,
> il s'agit d'un non-sens à mes yeux, extrapolation irrationnelle
> fondée sur la contraction de Lorentz.
> Tout ne peut arriver qu'au moment présent, inexorablement, quoi
> qu'on en dise.

le temps ne s'arrete pas à la vitesse de la lumiere !

simplement, il faut ne pas oublier la notion de referentiel et d'observateur

le temps s'ecoule toujours vu d'un observateur car le photon evolue... par
rapport a un referentiel lie au photon lui meme, tant a la vitesse c et
n'evoluant pas, la duree de vie est nulle...
tout comme vu d'un observateur sur le quai, un train va a une vitesse V par
rapport a l'observateur et vu d'un onservateur dans le train, le train va a
la vitesse 0 par rapport a l'observateur

de plus, quoi qu'on en dise, on est toujours au moment present qu'on soit
avec duree de vie nulle ou non, donc cela ne va pas a l'encontre d'une regle
quelconque .. il s'agit simplement de voir les choses d'un autre point de
vue, de decrire la meme "realite" par deux observations differentes

> C'est un postulat en ce qui concerne la réalité physique, car
> toutes les supposées preuves de cette courbures sont explicables
> autrement,

comment

>sans compter qu'elle est directement falsifiée par
> l'incapacité de la RG à rendre compte des trajectoires hyperboliques
> des sondes Pioneer 10 et 11,

la seule incapacite a ete de prevoir la trajectoire lointaine de ces sondes,
pour la simple raison que pour les calculs, il faut deja savoir ce qui se
passe la bas ! Comme aucune observation correcte n'existe, aucune theorie
n'est capable de rendre compte de quoi que ce soit .. de fait ce n'est pas
du tout une faille de la rg

prenons par exemple pionneer 10 : pas de probleme ...
je cite "Des chercheurs britanniques du Queen Mary and Westfield College
viennent de repérer un gros astéroïde en analysant de vieilles données de
vol d'une sonde américaine. En novembre et décembre 1992, la sonde Pionneer
10 a été déviée de sa trajectoire pendant 25 jours par l'attraction
gravitationnelle d'un corps céleste inconnu. L'engin se trouvait alors
au-delà de l'orbite de Pluton, à 8,4 milliards de kilomètres de la Terre"

de fait, la RG et la soit disante "faille" dans la trajectoire ont permis,
pour la seconde fois apres la decouverte de neptune, de decouvrir un astre
par son simple effet gravitationnel..
c'est maintenant considere comme un des gros succes de la RG...

etc etc

bref la RG n'a pas encore ete invalidee de maniere aussi sure que certains
ne le croient ...

@+
ricky


srp

non lue,
5 oct. 2002, 18:59:3305/10/2002
à
Pour les jeunes qui lisent ces lignes pour la première fois, ricky
est un triste individu qui m'a dénigré sur fsp pendant des mois,
ainsi que tout le travail de vulgarisation fait par le Service de
Recherche Pédagogique pour l'éducation, parce que je refuse
systématiquement de discuter avec lui de physique, car je suis un
causaliste, comme de Broglie, Planck, Schrödinger, Abraham et bien
d'autres, et qu'il appartient, comme la majorité des physiciens
contemporains, à l'école de pensée de Copenhague.

Il tente obsessivement par tous les moyens de discuter avec moi
malgré mon refus systématique, dans le but de me lasser pour
éventuellement me faire taire sur ce forum, en insinuant que
des résistances passées à d'autres présomptueux de son genre
étaient une attitude systématique envers tout le monde.

Comme vous pourrez le constater si vous relisez les enfilades dans
lesquelles ces supposés "physiciens" sont impliqués, ils attaquent
systématiquement, très agressivement et souvent avec rage et mépris,
(particulièrement dans le cas de ricky) tous ceux qui tentent de
donner aux jeunes les explications connues des phénomènes physiques
qui sont plus claires et simples que ce que la mécanique quantique
peut offrir.

La raison de leur acharnement particulier à mon endroit est que je
les dénoncent publiquement chaque fois qu'ils s'en prennent à moi,
en référant les lecteurs à l'analyse que Franco Selleri a fait du
problème dans "Le grand débat de la théorie quantique", Flammarion,
que je conseille à tous les jeunes de lire, et parce que j'ai remis
en circulation en Amérique du Nord certaines découvertes du passé
qu'ils ne mentionnent jamais dans les ouvrages de référence
qu'ils mettent à la disposition des jeunes dans les universités,
parce qu'elles contredisent leur dogmes fondamentaux.

Michel Talon, qui ne se gêne pas pour traiter Louis de Broglie et
ses collègues de zozos sur ce forum, nous a raconté ici même
comment ses collègues ont réussi à faire cesser tout enseignement
causaliste à la suite de la retraite de de Broglie, et comment au
CERN, on avait fait fermer le dernier projet de recherche qui ne
cadrait pas avec la philosophie de Copenhague.

Il y a cependant moyen d'apprendre la véritable physique
fondamentale, mais plus dans les facultés des universités, où
les programmes sont entièrement assujettis à la philosophie de
Copenhague depuis les années 60. Le seul moyen d'y parvenir
actuellement est de le faire hors du cadre académique.

À force de "discuter" avec ricky, j'ai fini par résumer, suite
à ses affirmations et confirmations, les principaux dogmes de
cette philosophie dans deux pseudo-définitions en apparence
caricaturales, mais absolument pas exagérées, métaphoriquement
destinées à paraître dans un dictionnaire du futur.

--------------------------------------------------------
"École de pensée de Copenhague" :

Philosophie irrationnelle de recherche instituée en physique
fondamentale par Heisenberg et Bohr dont le fondement était
la négation de l'existence de la réalité physique sous-jacente
aux théories éléborées pour la décrire, ce qui empêcha toute
recherche pendant plus de 70 ans, qui aurait eu pour but
d'augmenter nos connaissances au sujet de cette réalité physique
objective, dont la compréhension était, comme nous le savons
aujourd'hui, nécessaire à la survie de notre espèce.
(voir aussi "Minus copenhagensis").

La somme de leur contribution collective à la physique fondamentale
se résume donc par la force des choses à un espace blanc entre deux
paragraphes dans le grand livre de l'histoire.

À de rares exceptions près, tous les physiciens de la seconde
moitié du 20e siècle ont adhéré sans le moindre questionnement
à cette philosophie considérée ridicule par tous leurs contemporains
rationnels.

Extrait de la Préface écrite par Karl Popper, de "Le grand débat
de la Théorie quantique" de Franco Selleri publié vers la fin
du 20e siècle:

"Le livre du Pr. Franco Selleri, que j'ai lu dès sa parution, dans
sa première édition allemande, a non seulement une grande importance
historique, mais également un grand attrait. Il a été écrit par
quelqu'un qui a connu le lavage de cerveau de l'enseignement
traditionnel de la mécanique quantique et qui, en essayant de
trouver des remèdes de façon critique, tente d'en éliminer les
traces.

Le point le plus important, dans ce lavage de cerveau, c'est le
problème de la compréhension.... Bohr, Heisenberg, Pauli et
compagnie disaient tous : " Ne cherchez pas à comprendre la
mécanique quantique, elle est presque totalement incompréhensible."
-------------------------------------------------------------

--------------------------------------------------------------
"Minus copenhagensis" :

Membre d'une confrérie de pseudo-physiciens qui s'accapara jadis
(de 1927 jusqu'au début du 3e millénaire) tout l'espace disponible
en recherche fondamentale et empêchèrent pendant plus de 70 ans
toute recherche destinée à augmenter le bagage de connaissances
de l'humanité au sujet de la réalité physique sous-jacente à toutes
nos théories.

Les dogmes fondamentaux de la confrérie, qui était connue sous
le nom de "École de Copenhague" (voir définition), étaient :

a) qu'il n'existe pas de réalité physique objective sous-jacente
à la mécanique quantique,

b) que si cette réalité physique existait tout de même, elle ne
pouvait être comprise que par Dieu (d'où le sobriquet "minus
copenhagensis" qui leur fut accolé par la génération d'étudiants
qui les forca à la retraite, étant donné qu'ils se considéraient
eux-mêmes comme une sous-espèce d'homo sapiens sapiens qui était
incapable, contrairement à l'espèce mère, de raisonner logiquement,
dû à une régression auto-infligée du néo-cortex, régression qu'ils
considéraient comme l'état final de l'espèce humaine.

c) et que selon eux, ce n'était pas le rôle des physiciens d'explorer
quoi que ce soit qui sous-tendrait la mécanique quantique.

Leurs principales références philosophiques étaient

1) l'affirmation non fondée de Heisenberg qu'il était impossible
que les particules suivent leur trajectoire de moindre action
entre leur point de départ et leur point de détection, et que,
contrairement aux preuves expérimentales déjà connues et au
simple sens commun, toutes les particules étaient physiquement
des paquets d'ondes;

2) la recommandation de Feynman à tous ses confrères, dans
son célèbre article de 1949 ("Space-Time Approach to Quantum
Electrodynamics", Richard Feynman, Phys. Rev. 76, 769 (1949),
page 771.), de ne pas chercher à comprendre la séquence
temporelle précise d'événements qui se produit lors de collisions
entre les particules, car ce qui pourrait être découvert pourrait
entrer en contradiction avec les dogmes établis de la confrérie;

3) la lubie irrationnelle de John Archibald Wheeler que l'univers
disparaît et reparaît chaque fois qu'il cligne des yeux, et qu'il
est possible de changer le passé simplement en regardant fixement
les étoiles lointaines, théorie qu'il formalisa, dans un article
du magazine DISCOVER de juin 2002.

Le déclin de la confrérie fut fulgurant après que de nombreux
étudiants eurent pris connnaissance, dans des circonstances
demeurées nébuleuses, d'importantes découvertes concernant
la réalité physique, qui étaient tombées dans l'oublie à force
de ne plus être référencées dans les ouvrages de référence
utilisés dans le milieu académique pour la formation des
générations successives de physiciens, découvertes qui
contredisaient radicalement les dogmes des tenants de la
philosophie de Copenhague.

Les étudiants les ont immédiatement soupçonné d'avoir volontairement
caché ces découvertes aux générations successives pendant des
décennies pour empêcher le plus longtemps possible la révélation de
leur fraude intellectuelle, mais ils se trompaient; il ne s'agissait
que d'un simple élagage non malicieux tout à fait conforme à la
bêtise qui est indissociable de la philosophie de la confrérie.
--------------------------------------------------------------

ricky

non lue,
5 oct. 2002, 19:45:4605/10/2002
à
hello

> Il tente obsessivement par tous les moyens de discuter avec moi
> malgré mon refus systématique,

ben sur un fourm ;-)

>dans le but de me lasser pour
> éventuellement me faire taire sur ce forum,

marrant que tu puisses parler a ma place !
comment connais tu mon but ?
mon but est juste soit de prouver tes erreurs, ce que j'arrive a faire
semble t il puisque tu devies, soit de comprendre une theorie...

> des résistances passées à d'autres présomptueux de son genre
> étaient une attitude systématique envers tout le monde.

ben quand 90% de ceux qui t'on contredit sur tes bases theoriques ont eu le
meme traitement, tu traduit cela comment ?
de plus ta phrase contient un jugement personnel !

> systématiquement, très agressivement et souvent avec rage et mépris,
> (particulièrement dans le cas de ricky)

oh rage et mepris .. ou ?

>tous ceux

TOUS ceux ?
euh puis je avoir juste un autre pseudo que j'aurais attaque sur ce sujet
avec "rage " ? oui ? ah pas de reponse ?

> La raison de leur acharnement particulier à mon endroit

bref vous connaissez les raisons de tout le monde quoi :-)
pprix nobel de psycho sans doute ?

> les dénoncent publiquement

oh dans quel message ? a part des inepties et des insultes, pas vu de
"denociation" ...

>chaque fois qu'ils s'en prennent à moi,
> en référant les lecteurs à l'analyse que Franco Selleri a fait du
> problème dans "Le grand débat de la théorie quantique", Flammarion,
> que je conseille à tous les jeunes de lire,

ah oui
vous n'en conseillez qu'un alors !

>et parce que j'ai remis
> en circulation

ah oui
en fait remis en vente plutot :-) d ou l explication


> en Amérique du Nord certaines découvertes du passé
> qu'ils ne mentionnent jamais dans les ouvrages de référence

ce qui est faux comme chaque etudiant peut le voir !

> Michel Talon, qui ne se gêne pas pour traiter Louis de Broglie et
> ses collègues de zozos sur ce forum, nous a raconté ici même
> comment ses collègues ont réussi à faire cesser tout enseignement
> causaliste à la suite de la retraite de de Broglie, et comment au
> CERN, on avait fait fermer le dernier projet de recherche qui ne
> cadrait pas avec la philosophie de Copenhague.

ah.. sans doute un exemple non traduit de sa prose ???

@-


seb

non lue,
5 oct. 2002, 20:38:0305/10/2002
à
Bonjour.

srp : "parce que je refuse


systématiquement de discuter avec lui de physique"

je me suis déjà exprimé sur le "cas srp" donc inutile de me répéter.
et de tte façon cette petite quote parle d'elle-même.
si vous voulez parler de physique ok. sinon ne parlez pas.
vous n'avez AUCUN argument valable, ou alors toujours les mêmes copié/collé
(d'où le flood)
En effet #psycho serait un channel plus approprié pour vous.

/me a l'impression que le "débat" tourne en rond depuis un sacré moment.

/seb


srp

non lue,
5 oct. 2002, 22:51:0205/10/2002
à
seb a écrit :

>
> Bonjour.
>
> srp : "parce que je refuse
> systématiquement de discuter avec lui de physique"
>
> je me suis déjà exprimé sur le "cas srp" donc inutile de me
> répéter.

Vous le faites pourtant ici. Ça commence drôlement à ressembler
à un copier collé, bref, un spam.

> et de tte façon cette petite quote parle d'elle-même.

Absolument. Je discute physique avec qui me plait, c'est à dire
pas avec ricky ni aucun autre borné inconditionnel de l'école de
Copenhague.

> si vous voulez parler de physique ok.

J'en parle avec qui je veux. Pas besoin de la permission d'un autre
petit m...... anonyme qui s'imagine faire la loi sur ce fomum et qui
de plus, n'a jamais eu les c...... pour venir parler ici à visage
découvert.

> sinon ne parlez pas.

Vous allez vous y prendre comment pour me faire taire ?

> vous n'avez AUCUN argument valable, ou alors toujours les mêmes
> copié/collé (d'où le flood)
> En effet #psycho serait un channel plus approprié pour vous.

Connerie irrationnelle.



> /me a l'impression que le "débat" tourne en rond depuis un sacré
> moment.

Avec les inconditionnels de l'école de Copenhague, ça fait 70 ans
que le débat tourne en rond. D'où le changement de stratégie :

Retrait du droit de regard à tous les bornés inconditionnels
de l'école de Copenhague.

newsgroups

non lue,
6 oct. 2002, 03:13:4606/10/2002
à
Bonjour,

> Vous oubliez que l'électron est de manière vérifiée une particule
> "électromagnétique", pas une simple "charge", comme vous dites,
> qui est, soit dit en passant, une caractéristique de l'électron
> que personne n'a d'ailleurs encore réussi à comprendre, sauf les
> pontificateurs, bien sûr.

les propriétés intrinsèques d'un électron sont sa masse, sa charge et son
spin. C'est ca que tu appelles "éléctromagnétique" ?
Si la réponse est oui, tu emploies des termes trop vagues pour
être pertinent, si c'est non, alors ce que tu dis est simplement
faux.
Ton charabia est incompréhensible, répétitif est irritant à la longue.

> > Je laisse le reste de ce que tu dis aux amateurs de crypto-physique.
>
> Je dirais, aux amateurs de mécanique.
>
> Perso, je laisse les présomptions au sujet du comportement de
> la "charge" aux pontificateurs en attendant qu'elle soit
> comprise, et les maths aux amateurs de mathématique.

Tu devrais laisser à d'autres le soin de s'occuper de ces sujets.

François.

Norbert

non lue,
6 oct. 2002, 05:20:4606/10/2002
à
"srp" <s...@globetrotter.net> a écrit dans le message news:
3D9F9BC4...@globetrotter.net

> seb a écrit :
> >
> > Bonjour.
> >
> > srp : "parce que je refuse
> > systématiquement de discuter avec lui de physique"
> >
> > je me suis déjà exprimé sur le "cas srp" donc inutile de me
> > répéter.
>
> Vous le faites pourtant ici. Ça commence drôlement à ressembler
> à un copier collé, bref, un spam.

Et c'est *vous* qui faites cette remarque ?
Alors que vous polluez le forum avec votre diatribe sans cesse répétée
depuis des mois?


--
à bientot
==================================
les secrets de l'univers http://nrumiano.free.fr
un atlas de l'univers http://atunivers.free.fr
images du ciel http://images.ciel.free.fr
==================================


srp

non lue,
6 oct. 2002, 07:30:0106/10/2002
à
newsgroups a écrit :

>
> Bonjour,
>
> > Vous oubliez que l'électron est de manière vérifiée une particule
> > "électromagnétique", pas une simple "charge", comme vous dites,
> > qui est, soit dit en passant, une caractéristique de l'électron
> > que personne n'a d'ailleurs encore réussi à comprendre, sauf les
> > pontificateurs, bien sûr.
>
> les propriétés intrinsèques d'un électron sont sa masse, sa charge
> et son spin. C'est ca que tu appelles "éléctromagnétique" ?
> Si la réponse est oui,

La réponse est non. D'ailleurs, sa masse et sa charge sont des
propriétés relatives. Son spin est une propriété de sa structure
dynamique.

En passant, informez-nous donc de la profondeur des connaissances
de l'orthodoxie :

Qu'est-ce que la masse?

Qu'est-ce que la charge?

Qu'est-ce que le spin?

> tu emploies des termes trop vagues pour être pertinent,

Absolument pas. J'emploi les termes les plus précis qui soient
pour parler des particules électromagnétiques.

Ignorez vous donc que le photon est une particule électromagnétique,
et que l'électron peut être produit par découplage en paire
électron/position de tout photon d'énergie 1.022 MeV ou plus?

Contestez-vous que le photon est une particule électromagnétique?

Contestez-vous aussi l'électron ne peut être lui-même que de
nature électromagnétique puisqu'il est constitué de la même
substance que le photon?

> si c'est non, alors ce que tu dis est simplement faux.

Si vous dites que c'est faux, vous ne connaissez tout simplement
rien à la physique des particules.

> Ton charabia est incompréhensible, répétitif est irritant à la
> longue.

Si vous ne comprenez pas que l'électron est une particule
électromagnétique, il est clair que vous ne comprendrez rien
à ce que j'en dis.

> > > Je laisse le reste de ce que tu dis aux amateurs de
> > > crypto-physique.
> >
> > Je dirais, aux amateurs de mécanique.
> >
> > Perso, je laisse les présomptions au sujet du comportement de
> > la "charge" aux pontificateurs en attendant qu'elle soit
> > comprise, et les maths aux amateurs de mathématique.
>
> Tu devrais laisser à d'autres le soin de s'occuper de ces sujets.

Si je trouve vos réponses satisfaisantes à mes petites questions
posées plus haut, je me tais.

Sinon, vu que personne d'autre n'ose parler de véritable physique
causaliste sur ce forum, je continuerai.

ricky

non lue,
6 oct. 2002, 07:32:1506/10/2002
à
hello

> Absolument. Je discute physique avec qui me plait, c'est à dire
> pas avec ricky ni aucun autre borné inconditionnel de l'école de
> Copenhague.
>

mais je ne souhaite pas discuter physique avec vous ! je n'ai que faire de
vos idees sectaires et de votre volonte de vendre vos produits ... vous vous
prenez pour qui ?

de plus, pour parler physique avec vous, encore faudrait il que vous parliez
vous meme de physique et non de delires ...

vous etes sur un forum, pas sur votre site ou vous endoctrinez les gens ...

j'ai le droit sur un forum de repondre a une personne (y compris a vos
messages) pour souligner des aneries ou des choses justes, pour le reste du
forum qui existe, ne vous ne deplaise... ma reponse ne vous etait pas
destinee ...
elle montrait simplement des erreurs de votre part et les questions qui y
etaient attenantes, pour qu'une autre personne puisse reagir avec un autre
point de vue que le votre ...

vous n'etes pas seul sur ce forum et vous essayez a tout prix de me faire
taire (ainsi que tous ceux qui ont d'autres idees que les votres) pour pas
que les jeunes puisse comprendre votre imposture selon vos dires ...

ensuite, tout le monde peut voir que c'est vous qui voulez imposez vos lois
sur un forum au depend des regles generales du bon fonctionnement du net...

en esperant que vous ne me repondrez pas (j'aime bien reutiliser vos pseudo
argumentaires)

@-


srp

non lue,
6 oct. 2002, 07:35:3106/10/2002
à
Norbert a écrit :

>
> "srp" <s...@globetrotter.net> a écrit dans le message news:
> 3D9F9BC4...@globetrotter.net
> > seb a écrit :
> > >
> > > Bonjour.
> > >
> > > srp : "parce que je refuse
> > > systématiquement de discuter avec lui de physique"
> > >
> > > je me suis déjà exprimé sur le "cas srp" donc inutile de me
> > > répéter.
> >
> > Vous le faites pourtant ici. Ça commence drôlement à ressembler
> > à un copier collé, bref, un spam.
>
> Et c'est *vous* qui faites cette remarque ?

Bien sûr, c'est la deuxième fois que cet anonyme vient vomir
la même chose sur un de mes messages, c'est donc un spam, non?

> Alors que vous polluez le forum avec votre diatribe sans cesse
> répétée depuis des mois?

Je ne le répète qu'à ceux qui tentent de me faire taire, ce qui
inclus par définition tous les tenants de l'école de Copengague,
car les référence que je donne concernent des recherches qui
contredisent leur philosophie irrationnelle.

Je converse normalement avec tous les autres.

srp

non lue,
6 oct. 2002, 07:40:3806/10/2002
à

--------------------------------------------------------------
"Minus copenhagensis" :

--

srp

non lue,
6 oct. 2002, 07:41:1706/10/2002
à
Pour les jeunes qui lisent ces lignes pour la première fois, ricky
est un triste individu qui m'a dénigré sur fsp pendant des mois,
ainsi que tout le travail de vulgarisation fait par le Service de
Recherche Pédagogique pour l'éducation, parce que je refuse
systématiquement de discuter avec lui de physique, car je suis un
causaliste, comme de Broglie, Planck, Schrödinger, Abraham et bien
d'autres, et qu'il appartient, comme la majorité des physiciens
contemporains, à l'école de pensée de Copenhague.

Il tente obsessivement par tous les moyens de discuter avec moi
malgré mon refus systématique, dans le but de me lasser pour
éventuellement me faire taire sur ce forum, en insinuant que

des résistances passées à d'autres présomptueux de son genre
étaient une attitude systématique envers tout le monde.

Comme vous pourrez le constater si vous relisez les enfilades dans

lesquelles ces supposés "physiciens" sont impliqués, ils attaquent

systématiquement, très agressivement et souvent avec rage et mépris,

(particulièrement dans le cas de ricky) tous ceux qui tentent de
donner aux jeunes les explications connues des phénomènes physiques
qui sont plus claires et simples que ce que la mécanique quantique
peut offrir.

La raison de leur acharnement particulier à mon endroit est que je

les dénoncent publiquement chaque fois qu'ils s'en prennent à moi,

en référant les lecteurs à l'analyse que Franco Selleri a fait du
problème dans "Le grand débat de la théorie quantique", Flammarion,

que je conseille à tous les jeunes de lire, et parce que j'ai remis
en circulation en Amérique du Nord certaines découvertes du passé

qu'ils ne mentionnent jamais dans les ouvrages de référence

qu'ils mettent à la disposition des jeunes dans les universités,
parce qu'elles contredisent leur dogmes fondamentaux.

Michel Talon, qui ne se gêne pas pour traiter Louis de Broglie et


ses collègues de zozos sur ce forum, nous a raconté ici même
comment ses collègues ont réussi à faire cesser tout enseignement
causaliste à la suite de la retraite de de Broglie, et comment au
CERN, on avait fait fermer le dernier projet de recherche qui ne
cadrait pas avec la philosophie de Copenhague.

Il y a cependant moyen d'apprendre la véritable physique

newsgroups

non lue,
6 oct. 2002, 10:42:1906/10/2002
à

Rarement vu un tel tissu d'inepties.

>
> > si c'est non, alors ce que tu dis est simplement faux.
>
> Si vous dites que c'est faux, vous ne connaissez tout simplement
> rien à la physique des particules.

snip

> Si je trouve vos réponses satisfaisantes à mes petites questions
> posées plus haut, je me tais.
>
> Sinon, vu que personne d'autre n'ose parler de véritable physique
> causaliste sur ce forum, je continuerai.


Alors tu continueras à t'égosiller pour rien.

François.

srp

non lue,
6 oct. 2002, 11:46:1906/10/2002
à
newsgroups a écrit :

>
> Rarement vu un tel tissu d'inepties.
>
> >
> > > si c'est non, alors ce que tu dis est simplement faux.
> >
> > Si vous dites que c'est faux, vous ne connaissez tout simplement
> > rien à la physique des particules.
>
> snip

Je vois.

Tel que je m'y attendais, le défi était trop dangeureux.

Peur de faire un faux pas devans les pairs et de se faire rappeler
publiquement à l'ordre. Peur de l'opinion des jeunes. Le piège
insoluble de l'orthodoxie.

On préfère s'esquiver.



> > Si je trouve vos réponses satisfaisantes à mes petites questions
> > posées plus haut, je me tais.
> >
> > Sinon, vu que personne d'autre n'ose parler de véritable physique
> > causaliste sur ce forum, je continuerai.
>
> Alors tu continueras à t'égosiller pour rien.

Erreur. Les jeunes ne sont pas stupides. Ils sont tous capables
de vérifier les références livresques que j'ai donné dans le
passé, et que je vais continuer à donner, et de se faire leur
propre idée sur la question.

Pierre

non lue,
6 oct. 2002, 12:15:2506/10/2002
à
>Comme vous pourrez le constater si vous relisez les enfilades

Non merci!! Pas la peine de relire , tout le mode connait maintenant votre
position.

D'ailleurs, il serait assez courtois de votre part d'arréter de répéter votre
manifeste "Minus copenhagensis" sans en changer une virgule.
(ça devient du bourrage de crâne ).

La probabilité qu'on y apprenne quelque chose de nouveau diminue à chaque
répétition.

Ceci dit, si votre opinion, conviction, intuition (comme vous voulez) est que
l'on doit pouvoir trouver un modèle causaliste au delà de l'interprétation de
Copenhague, indiquez nous quelques pistes ou hypothèses que nous pourrions
discuter sur ce forum.

Par exemple l'effet tunnel : comment pourrait on expliquer cet effet avec un
modèle causaliste , ne faisant pas appel au principe d'incertitude et aux
probabiltés ?(que dit SELLERI à ce sujet ?)

Voila un débat qui pourrait nous changer des précédents threads interminables
et pleins d'invectives (que je n'ai surtout pas l'intention de relire)

srp

non lue,
6 oct. 2002, 15:49:4506/10/2002
à
Pierre a écrit :

>
> >Comme vous pourrez le constater si vous relisez les enfilades
>
> Non merci!! Pas la peine de relire , tout le mode connait maintenant
> votre position.
>
> D'ailleurs, il serait assez courtois de votre part d'arréter de
> répéter votre manifeste "Minus copenhagensis" sans en changer une
> virgule.
> (ça devient du bourrage de crâne ).

À mes yeux, c'est une protestation systématique contre la prétention
de certains, que tout le monde est obligé de discuter avec eux,
simplement parce qu'ils sont physiquement "à portée de voix".

C'est faux dans un parc public, et c'est tout aussi faux sur un
forum public. La liberté de choisir ses interlocuteurs est aussi
fondamentale ici que dans un parc. Personne n'est obligé de
supporter les intrus désagréables ou malveillants, quelles que
soient les circonstances.

> La probabilité qu'on y apprenne quelque chose de nouveau diminue
> à chaque répétition.

Mauvaise perception. Il n'est pas destiné à informer qui que ce
soit dans l'orthodoxie. Je sais ce qu'elle pense l'orthodoxie.
Les destinataires sont clairement identifés dès la première ligne
dudit message.



> Ceci dit, si votre opinion, conviction, intuition (comme vous
> voulez) est que l'on doit pouvoir trouver un modèle causaliste
> au delà de l'interprétation de Copenhague, indiquez nous quelques
> pistes ou hypothèses que nous pourrions discuter sur ce forum.

Je ne crois à aucune discussion constructive à laquelle
participeraient des inconditionnels de l'école de Copenhague.

Personne ne pourra être plus explicite, et en vain, avec eux que
l'ont été Einstein, de Broglie, Lochak, Selleri.

Personne ne peut changer une conviction irrationelle profonde.
Sentez-vous libre de discuter de toutes les théories que vous
voulez avec eux, mais sans moi.

De toute manière, il ne s'agit pas de trouver un "modèle causaliste",
il s'agit de finir de comprendre la mécanique électromagnétique des
particules, recherche qui a été totalement abandonnée avec
l'avénement et l'hégémonie subséquente de la philosophie défaitiste
de Heisenberg, et dont la reprise dépend directement de la reprise
de conscience par la génération montante de l'existence réelle de
la réalité physique sous-jacente à nos théories.

Il s'agit de quelque chose de totalement incompréhensible pour
les inconditionnels de l'école de Copenhague. Ce débat et cette
reprise, encore à venir, ne pourront se faire que sans eux,
puisqu'il est fondamental à leur philosophie que cette réalité
physique sous-jacente n'existe pas.



> Par exemple l'effet tunnel : comment pourrait on expliquer
> cet effet avec un modèle causaliste , ne faisant pas appel
> au principe d'incertitude et aux probabiltés ?

Je crois me souvenir que vous m'avez dit un jour que vous aviez
adhéré à l'école de Copenhague parce qu'il ne semblait pas y
avoir d'autre choix, autrement dit, par défaut.

En considération de ce fait, et le fait que vous vous êtes
donné la peine de mettre en perspective correcte que l'idée du
ralentissement du temps était une conclusion de la RR et non
de la RG, comme je l'avançais superficiellement, je vais tenter
de vous l'expliquer.

Les effets de Goos Hänchen et Himbert qui permettent l'effet tunnel
associés au phénomène de réflexion totale dans les prismes (et
autres phénomènes similaires associés aux électrodes), pourraient
bien n'être en fin de compte qu'un effet de réfraction limite dû
au fait que les particules concernées, photons, électrons, etc.,
sortent du matériau à un angle tellement proche de la surface
opposée, que les interactions coulombiennes combinées de l'ensemble
des particules du corps, avec ces particules en mouvement, réussit
à les forcer à repénétrer le corps aussitôt après le plus bref des
séjours à l'extérieur des atomes de surface (d'où le phénomène
de réflexion totale à certains angles); un peu comme des comètes
sur trajectoires paraboliques entament la deuxième partie de leurs
trajectoires après avoir frôlé le côté éloigné du Soleil.

L'effet tunnel lui-même devient alors très simple à expliquer par le
fait que d'approcher une lentille à suffisamment grande proximité du
prisme à un des points de la surface réfléchissant les photons (ou
une électrode, si des électrons sont impliqués) permet à un certain
nombre de ces photons (où électrons) de continuer leur route dans le
matériau de la lentille (ou de l'électrode), en raison de
l'intensification, au point de grande proximité, des interactions
coulombiennes entre ces photons (ou électrons) et les particules
constituant la lentille (ou l'électrode) qui vont compenser de façon
égalisatrice celles dues aux particules du prisme (ou du matériel
ionisé négativement).

> (que dit SELLERI à ce sujet ?)

Je ne m'en souviens pas, et je n'ai pas envie de me retaper son
ouvrage pour une telle raison.

> Voila un débat

Ne comptez pas sur moi pour débatre d'un tel sujet ici, considérant
le niveau d'irrationalité entretenu par les tenants de l'école de
Copenhague. Mais sentez vous bien libre de le faire.

> qui pourrait nous changer des précédents threads interminables et
> pleins d'invectives (que je n'ai surtout pas l'intention de relire)

Lorsque ricky et autres prétencieux irrationnels à la sauce
Copenhague ne viennent pas essuyer leurs gros sabots sur mes
messages, mes interventions sont des plus calmes.

ricky

non lue,
6 oct. 2002, 16:50:0006/10/2002
à
bonjour

> À mes yeux,

A VOS YEUX...
mais on est sur un forum et vos yeux, on s'en fiche.. lisez la netiquette et
les FAQ des forums ... ce sont eux qui comptent

>c'est une protestation systématique contre la prétention
> de certains, que tout le monde est obligé de discuter avec eux,
> simplement parce qu'ils sont physiquement "à portée de voix".

non simplement parceque vous utilisez un espace qui ne vous appartient pas
et qui a des regles du jeu que vous refusez
c'est comme si vous vous invitiez a un match de foot et decidiez de jouer
avec les mains ... il y a des regles

> C'est faux dans un parc public, et c'est tout aussi faux sur un
> forum public. L

cette comparaison prouve votre stupidite et vos qualites de pedagogue...
le parc public a ses regles et les forums ont les leur... regles que vous ne
vous donnez meme pas la peine de lire d'ailleurs alrors qu'elles sont
postees regulierement

>a liberté de choisir ses interlocuteurs est aussi
> fondamentale ici que dans un parc.

sauf que la comparaison est erronee
vous cherchez a jouer avec les mains dans un jeu de foot ball.. cette
comparaison est plus exacte car vous detournez les regles a votre guise

> Personne n'est obligé de
> supporter les intrus désagréables ou malveillants, quelles que
> soient les circonstances.

non mais d'accepter qu'il y ai des reponses a vos messages pour que les
autres voient les failles oui .. ce sont les regles du jeu ici
ne repondez pas a mes questions , ca oui , car vous ne etes incapable.. cela
prouve que vous n'etes pas venu ici pour parler physique mais seulement pour
vendre vos bouquins, ce qui n'est pas l'enjeu de ce forum... mais empecher
quelqu un dz vous repondre n'est pas dans les regles du forum.. surtout en
prenant tout le forum en otage, ce qui prouve encore plus vos soit disantes
methodes pedagogiques qui sont visiblement sectaires ...

> Mauvaise perception. Il n'est pas destiné à informer qui que ce
> soit dans l'orthodoxie. Je sais ce qu'elle pense l'orthodoxie.
> Les destinataires sont clairement identifés dès la première ligne
> dudit message.

vous prouvez donc que vous etes un ane et que vous profitez du forum.. quand
on veut cibler un destinataire, la regle c'est le mail ... sur un forum,
vous ciblez tout le monde par definition
apprenez a utiliser le net avant de faire croire que vous connaissez
quelquechose a la pedagogie..
et arretez de vous cacher devant la remise en circulation de quoique ce
soit..

> Je ne crois à aucune discussion constructive à laquelle
> participeraient des inconditionnels de l'école de Copenhague.

c'est votre probleme, pas celui d'un forum

> Personne ne pourra être plus explicite, et en vain, avec eux que
> l'ont été Einstein, de Broglie, Lochak, Selleri.

c'est votre croyance jamais demontre par vous
et meme pire, on a demontre, et on est beaucoup a le faire, que vous
deformez leur propos

> Personne ne peut changer une conviction irrationelle profonde.
> Sentez-vous libre de discuter de toutes les théories que vous
> voulez avec eux, mais sans moi.

sans vous, il n'y a pas de probleme
suelement vous etes sur un forum avec ses regles que vous ignorez
volontairement

> De toute manière, il ne s'agit pas de trouver un "modèle causaliste",
> il s'agit de finir de comprendre la mécanique électromagnétique des
> particules, recherche qui a été totalement abandonnée avec
> l'avénement et l'hégémonie subséquente de la philosophie défaitiste
> de Heisenberg, et dont la reprise dépend directement de la reprise
> de conscience par la génération montante de l'existence réelle de
> la réalité physique sous-jacente à nos théories.

ce qui est une anerie

je ne m'adresse pas a vous, puisque votre abscence de dialogue montre votre
cote secatire et la tentative d'embrigadement que vous menez depuis
plusieurs annees deja... donc inutile de faire croire que je vous repond a
vous ... je me fiche pas mal de ce que les sectes pensent...

il est facile de voir dans les divers articles, de nature a pour la science
en passant par les programmes de fac et autre, que cette recherche a
toujours continuee.... simplement cette recherche est devenue plus
rationnelle, et a elimine tous les pseudo prophetes qui pensaient avoir
acces directement a une "realite objective" sans jamais passer par le filtre
de l'observation.. bref la science a compris que pour evoluer, ce qu'elle
fait, elle devait voir les choses en face et tenter de coller a la realite
et a sa complexite, plutot que de se crpoire dieu et de pretendre connaitre
quelcuqechoe qui ne lui est pas DIRECTEMENT accessible

> Il s'agit de quelque chose de totalement incompréhensible pour
> les inconditionnels de l'école de Copenhague.

ce qui n'a pas de rapport direct avec copenhague, ecole de pensee qui ne se
resume aps a ce simple precepte qui est utilise depuis bien plus longtemps
que cela..
on remlarquera en passant que ni de broglie, ni maxwell n'ont jamais utilise
la notion de "realite objective" et que l'on retrouve (meme chez flammarion)
dans les divers livres les notion de THEORIE, OBSERVATIONS, concordance a la
realite, accord entre l'observation du reel et la theorie mais jamais une
phrase du genre "c'est la realite objective"

c'est ce qui gene beaucoup les faux prophetes et qui les obliges a tenter
d'imposer leur idees autrement, car ils sont dans l'incapacite de trouver
une phrase de ce genre nulle part ..sauf peut etre dans un ou deux livres
d'alchimistes dont les travaux n'ont jamais rien donnes

on constate d'ailleurs que comme tout prophete, ces dernieres s'adressent
touojurs aux jeunes et au futur, car il s ont toujours echoues au present !

> puisqu'il est fondamental à leur philosophie que cette réalité
> physique sous-jacente n'existe pas.

la realite existe mais la theorie ne s'appuie que sur l'observable sous
jacent ..
mais c'est trop complexe a comprendre pour les mages !

> L'effet tunnel lui-même devient alors très simple à expliquer par le
> fait que d'approcher une lentille à suffisamment grande proximité du
> prisme à un des points de la surface réfléchissant les photons (ou
> une électrode, si des électrons sont impliqués) permet à un certain
> nombre de ces photons (où électrons) de continuer leur route dans le
> matériau de la lentille (ou de l'électrode), en raison de
> l'intensification, au point de grande proximité, des interactions
> coulombiennes entre ces photons (ou électrons) et les particules
> constituant la lentille (ou l'électrode) qui vont compenser de façon
> égalisatrice celles dues aux particules du prisme (ou du matériel
> ionisé négativement).

sauf qu'on arrive a faire ce genre de prouesse avec des barrieress
energetiques telles qu'il ne s'agirait plus d'une compensation mais
carrement d'une amplification enorme de l'energie par la magie du saint
esprit...
les interactions dont srp parlent foncitonnent dans quelques cas de figure
mais en aucun cas dans les dernier microscopes a effet de champs ...

> Je ne m'en souviens pas, et je n'ai pas envie de me retaper son
> ouvrage pour une telle raison.

marrant ;-)
on constate l'eviction a chaque probleme

> Lorsque ricky et autres prétencieux irrationnels à la sauce
> Copenhague ne viennent pas essuyer leurs gros sabots sur mes
> messages, mes interventions sont des plus calmes.

vos messages sont sur un forum et ne vous apprtiennent plus
ensuite, un sipmle google prouve que vous appelez "irrationnels" toute
personne qui arrive a vous mettre en defaut ... donc vous admettez
simplement que vos interventions seront plus calmes quand plus personne ne
vous contredira ..
le probleme est qu'il s'agit de forum et pas de votre jouet personnel

@-
PS : je vous signal que ce message ne vous est pas specifiquement destine
... de fait, je sens que vous allez demontrer une fois de plus que vous
tentez d'empecher "les jeunes" d'acceder a d'autres informations que celles
que vous daignez leur donner

srp

non lue,
6 oct. 2002, 17:03:2406/10/2002
à
Tel que les jeunes peuvent le constater en lisant le dernier
message de ricky, sa mauvaise foi serait difficile à démontrer
de manière plus convaincante. Fermeture d'esprit totale qui s'est
faite jour dès son premier message à mon endroit il y a plusieurs
mois.

Ce triste individu m'a dénigré sur fsp pendant des mois, ainsi que

tout le travail de vulgarisation fait par le Service de Recherche
Pédagogique pour l'éducation, parce que je refuse systématiquement
de discuter avec lui de physique, car je suis un causaliste, comme
de Broglie, Planck, Schrödinger, Abraham et bien d'autres, et qu'il
appartient, comme la majorité des physiciens contemporains, à l'école
de pensée de Copenhague.

Il tente obsessivement par tous les moyens de discuter avec moi
malgré mon refus systématique, dans le but de me lasser pour
éventuellement me faire taire sur ce forum, en insinuant que
des résistances passées à d'autres présomptueux de son genre
étaient une attitude systématique envers tout le monde.

Comme vous pourrez le constater si vous relisez les enfilades dans
lesquelles ces supposés "physiciens" sont impliqués, ils attaquent
systématiquement, très agressivement et souvent avec rage et mépris,
(particulièrement dans le cas de ricky) tous ceux qui tentent de
donner aux jeunes les explications connues des phénomènes physiques
qui sont plus claires et simples que ce que la mécanique quantique
peut offrir.

La raison de leur acharnement particulier à mon endroit est que je

les dénonce publiquement chaque fois qu'ils s'en prennent à moi,

en référant les lecteurs à l'analyse que Franco Selleri a fait du
problème dans "Le grand débat de la théorie quantique", Flammarion,
que je conseille à tous les jeunes de lire, et parce que j'ai remis
en circulation en Amérique du Nord certaines découvertes du passé
qu'ils ne mentionnent jamais dans les ouvrages de référence
qu'ils mettent à la disposition des jeunes dans les universités,
parce qu'elles contredisent leur dogmes fondamentaux.

Michel Talon, qui ne se gêne pas pour traiter Louis de Broglie et
ses collègues de zozos sur ce forum, nous a raconté ici même
comment ses collègues ont réussi à faire cesser tout enseignement
causaliste à la suite de la retraite de de Broglie, et comment au
CERN, on avait fait fermer le dernier projet de recherche qui ne
cadrait pas avec la philosophie de Copenhague.

Il y a cependant moyen d'apprendre la véritable physique
fondamentale, mais plus dans les facultés des universités, où
les programmes sont entièrement assujettis à la philosophie de
Copenhague depuis les années 60. Le seul moyen d'y parvenir
actuellement est de le faire hors du cadre académique.

--

Rei Ichido

non lue,
6 oct. 2002, 18:09:4406/10/2002
à
srp wrote:

> À mes yeux, c'est une protestation systématique contre la prétention
> de certains, que tout le monde est obligé de discuter avec eux,
> simplement parce qu'ils sont physiquement "à portée de voix".
>
> C'est faux dans un parc public, et c'est tout aussi faux sur un
> forum public. La liberté de choisir ses interlocuteurs est aussi
> fondamentale ici que dans un parc. Personne n'est obligé de
> supporter les intrus désagréables ou malveillants, quelles que
> soient les circonstances.

En l'occurence, le personnage désagréable, ça commence à être vous. Je
ne vous cache pas que gacher de la bande passante pour télécharger N fois le
même message, c'est vraiment e****dant. Par conséquent, je vous demande de
bien vouloir ne plus répondre à Ricky par le même message constant. De même,
si Ricky voulait bien accepter de vous laisser tranquille dans votre
coin....

Rei.


Sylvain

non lue,
6 oct. 2002, 18:34:0306/10/2002
à
En m'excusant de ne pas avoir suivi le détail de ces houleuses
discussions (et surtout au cas où une question équivalente aurait déjà
été soulevée)
je voudrais apporter mon grain de sel et poser une question à A.M:

Considérons le paradoxe EPR, sous toutes ses coutures comme il est
connu, c'est-à-dire incluant la question des inégalités de Bell.
Comme exemple de phénomène quantique, la question de sa nature et de
son éventuelle interprétation peut s'analyser par la suite de
questions suivantes:

1) Quel est précisément l'énoncé des prédictions expérimentales issues
de la MQ indépendamment de son formalisme ?
Pour fixer les idées, voir la description en de tels termes que
j'avais mis dans mon texte disponible (.ps) sur
http://spoirier.lautre.net/relativite.html
(voir le chapitre sur le paradoxe EPR; pour se concentrer sur cette
question précise vous pouvez sauter les quelques commentaires généraux
sur la MQ qui le précèdent)
Etes-vous d'accord que c'est bien là la description des prédictions
expérimentales de la MQ sur cette question ?

2) Ces prédictions sont-elles vérifiées par l'expérience ?
Il me semble que oui; en tout cas un certain nombre de critères de
conformité des conditions expérimentales avec cette description ont
été remplis, peut-être pas tous en même temps (quelqu'un a des infos
précises ?), et sinon physiquement il n'y aurait pas de "bonne raison"
après le succès de ces approches, pour que les expériences ultimes
échappent subitement à ce phénomène général de conformité aux
prédictions.

3) Si on est finalement d'accord que tels sont les effets observables
de la MQ, et si vous affirmez qu'il est possible d'interpréter la MQ
par une réalité sous-jacente, alors dans quelle direction pensez-vous
que l'interprétation de ce phénomène devrait pouvoir se trouver ?

Je vous remercie de bien vouloir examiner attentivement ce problème
afin d'y répondre sérieusement.

srp

non lue,
6 oct. 2002, 20:01:1506/10/2002
à
Rei Ichido a écrit :

>
> srp wrote:
>
> > À mes yeux, c'est une protestation systématique contre la prétention
> > de certains, que tout le monde est obligé de discuter avec eux,
> > simplement parce qu'ils sont physiquement "à portée de voix".
> >
> > C'est faux dans un parc public, et c'est tout aussi faux sur un
> > forum public. La liberté de choisir ses interlocuteurs est aussi
> > fondamentale ici que dans un parc. Personne n'est obligé de
> > supporter les intrus désagréables ou malveillants, quelles que
> > soient les circonstances.
>

> En l'occurence, le personnage désagréable, ça commence à être vous.

Je n'ai aucune ambition que l'orthodoxie me trouve agréable. En ce
qui me concerne, ce que pensent de moi tous ceux qui pensent par
ailleurs que de Broglie était un zozo, ce que j'ai vérifié
extensivement être le cas de manière généralisée pour tous les
tenants de l'école de Copenhague, malheureusement pour la science,
la majorité de notre supposée élite en physique fondamentale, est
le moindre de mes soucis.

> Je ne vous cache pas que gacher de la bande passante pour
> télécharger N fois le même message, c'est vraiment e****dant.

Je ne poste ce message que sur provocation.

> Par conséquent, je vous demande de bien vouloir ne plus répondre
> à Ricky par le même message constant.

Je vais tenter de réduire, mais après avoir tenté de le raisonner
pendans des mois, je ne trouve plus rien d'autre à lui répondre,
car c'est ce qui résume la situation, particulièrement dans son
cas.

> De même, si Ricky voulait
> bien accepter de vous laisser tranquille dans votre coin...

Je ne crois pas qu'il soit assez intelligent pour comprendre ce
que vous lui demandez.

srp

non lue,
6 oct. 2002, 21:49:5206/10/2002
à
Sylvain a écrit :

>
> En m'excusant de ne pas avoir suivi le détail de ces houleuses
> discussions (et surtout au cas où une question équivalente aurait
> déjà été soulevée)

En réalité, elle n'est pas houleuse, et ce n'est même pas une
discussion. C'est simplement mon refus total exprimé de ne
discuter avec les tenants de l'école de Copenhague pour aucune
considération.

> je voudrais apporter mon grain de sel et poser une question à A.M:
>
> Considérons le paradoxe EPR, sous toutes ses coutures comme il est
> connu, c'est-à-dire incluant la question des inégalités de Bell.
> Comme exemple de phénomène quantique, la question de sa nature et de
> son éventuelle interprétation peut s'analyser par la suite de
> questions suivantes:
>
> 1) Quel est précisément l'énoncé des prédictions expérimentales
> issues de la MQ indépendamment de son formalisme ?
> Pour fixer les idées, voir la description en de tels termes que
> j'avais mis dans mon texte disponible (.ps) sur
> http://spoirier.lautre.net/relativite.html
> (voir le chapitre sur le paradoxe EPR; pour se concentrer sur
> cette question précise vous pouvez sauter les quelques commentaires
> généraux sur la MQ qui le précèdent)

J'ai réussi à télécharger le fichier .ps mais ne n'ai pas réussi
la le faire afficher. Je n'ai malheureusement pas réussi à
charger le fichier pdf.

Petite note, je suis en désaccord déjà avec l'énoncé suivant qui
commence la section "Pour éviter tout contresens"

"L'objet de la Physique est de décrire les lois physiques,
c'est-à-dire les relations mathématiques qui contraignent
le comportement de la matière.

En ce qui me concerne, l'objet de la physique fondamentale est
de décrire et "comprendre" les particules élémentaires et leurs
interactions.

Les mathématiques ont pour objet de décrire, et non de contraindre
le comportement de la matière. Rien ne peut changer la réalité.
Elle existe indépendemment de ce qu'on en pense. Rien de ce qu'on
peut en penser ne peut la contraindre.

> Etes-vous d'accord que c'est bien là la description des prédictions
> expérimentales de la MQ sur cette question ?

je doute que mon explication ait du sens pour vous si vous pensez au
départ que le paradoxe EPR et son analyse est vraiment significative,
mais je vais essayer, malgré l'absence de votre texte.

Au départ, je suis d'accord avec l'énonsé de EPR à l'effet que
"Every element of the physical reality must have a counterpart
in the physical theory".

Et c'est l'objectif à atteindre avec toutes les nouvelles données
maintenant disponibles qui ne sont pas encore intégrées.

Mais je suis en désaccord avec la définition qu'ils font du
"critère minimal de réalité"

"If, without in any way disturbint a system, we can predict with
certainty (i.e., with probability equal to unity) the value of a
physical quantity, then there exists an element of physical reality
corresponding to this physical quantity."

En ce qui me concerne, on peut extrapoler n'importe quoi d'une
expérience intellectuelle. Le seul critère d'existence physique
d'une particule, par exemple, est la preuve démontrable et
reproduisible qu'elle peut entrer en collision avec une autre
particule.

La seule preuve d'existence physique d'une interaction, est
la vérification expérimentale incontestable, démontrable et
reproduisible de son existence réelle, comme on peut le faire
pour l'interaction coulombienne, et l'interaction entre des
aimants.

Selon moi, tout ce qui existe matériellement ne peut être construit
que de ces particules et de ces interactions dont on peut vérifier
directement l'existence.



> 2) Ces prédictions sont-elles vérifiées par l'expérience ?

Je n'ai pas lu votre texte, mais dans ma perspective, la MQ n'est
qu'une méthode descriptive des niveaux d'énergie des orbitales dans
les atomes. Pas plus prédictive que ne l'est le calcul différentiel
et intégral. Une simple méthode statistique et mathématique.

Heisenberg et Bohr l'ont élevé au status de méthode prédictive
seulement parce qu'ils étaient irrationnellement convaincu qu'elle
définissait le réel, et que par conséquent, elle devait pouvait
pouvoir prédire ce qui arrive dans le réel.

Einstein, Podolsky et Rosen, ont tenté, en tenant un langage
qui à leurs yeux serait compris de l'ensemble des physiciens,
de les convaincre que la MQ ne pouvait pas être élevée à ce
statut (qu'elle n'a jamais eu à mes yeux).

Étant donné que je ne ressens aucun besoin de tenter d'en convaincre
l'orthodoxie (peines perdues d'avance), j'ai très peu d'intérêt
pour ce type d'argumentation.

> Il me semble que oui; en tout cas un certain nombre de critères de
> conformité des conditions expérimentales avec cette description ont
> été remplis, peut-être pas tous en même temps (quelqu'un a des infos
> précises ?),

Selleri en fait une analyse exhaustive et très à date (1986) dans
son livre "Le grand débat de la théorie quantique" (Chapitres
4 et 5)

> et sinon physiquement il n'y aurait pas de "bonne raison"
> après le succès de ces approches, pour que les expériences
> ultimes échappent subitement à ce phénomène général de conformité
> aux prédictions.

Mon opinion ici est que MQ ou pas, on aurait vérifié les mêmes
choses expérimentalement.

Autrement dit, ce n'est pas parce que la MQ existait qu'il a été
possible de découvrir les quarks up et down dans les nucléon,
c'est simplement parce que la technologie avait suffisemment
évolué dans les années 60 pour que l'accélérateur linéaire de
Stanford devienne réalité, avec son énergie suffisente pour
permettre de percer le mystère. Théories prédictives ou pas,
on aurait tout de même essayé, et on aurait tout de même trouvé.

> 3) Si on est finalement d'accord que tels sont les effets
> observables de la MQ,

À mes yeux, ce sont des effets observables, point, MQ ou pas.

> et si vous affirmez qu'il est possible d'interpréter la MQ
> par une réalité sous-jacente,

Il n'y a pas à interpréter la MQ. Tout ce qu'elle peut faire,
elle le fait de manière raisonnable, soit rendre compte des
niveaux d'énergie des orbitales.

Ce qui fait que la mq peut mesurer ces niveaux, c'est parce
qu'ils existent réellement. Ils existaient déjà avant, mais
on n'était pas encore capables de les mesurer aussi précisément.

C'est tout ce que j'y vois.

Pour moi, la MQ n'est rien d'autre qu'une méthode mathématique
parmi d'autres.

> alors dans quelle direction pensez-vous que l'interprétation de
> ce phénomène devrait pouvoir se trouver ?

Ben, pour moi, la MQ n'est pas un facteur dans l'équation. Elle
est là à notre disposition lorsqu'on en a besoin. Le vrai problème
est qu'il devient urgent, après 70 ans d'arrêt, de nous remettre
au plus vite, à intégrer dans notre description intégrée de la
réalité, les nouvelles données dont nous avons pu confirmer la
réalité pendant tout ce temps, et certaines déjà comprises avant
mais qui ont été laissées de côté parce qu'elles ne câdraient pas
soit avec la RR, la RG et la MQ.

Le problème de la physique fondamentale depuis 70 ans, est que
la MQ a été mise sur un piedestal par une foule d'adorateurs
inconditionnels qui sont devenus incapables de voir la réalité
physique sous-jacente qui est irrémédiablement masquée pour
leurs yeux éblouis par la fonction d'onde.



> Je vous remercie de bien vouloir examiner attentivement ce problème
> afin d'y répondre sérieusement.

J'ai fait de mon mieux. Mais je n'ai pas d'illusion sur le résultat.

Si vous pouvez me débloquer votre texte (.ps ou .pdf) de manière
à ce que je puisse y accéder, je pourrais tenter de répondre plus
ponctuellement.

ricky

non lue,
7 oct. 2002, 01:29:1307/10/2002
à
salut rei

> bien vouloir ne plus répondre à Ricky par le même message constant. De
même,
> si Ricky voulait bien accepter de vous laisser tranquille dans votre
> coin....

excuse moi mais tu remarqueras que je n'avais pas repondu a ce personnage
depuis un certain temps, simplement il a continue a me prendre a parti ...

desole mais il a deja reussi a empecher pas mal de personnes de s'exprimer
selon son humeur et surtout selon ce qui le gene... je ne trouve pas normal
qu'une personne fasse la loi come cela depuis des annees, seule contre la
majorite et trouve le moyen de continuer a aller de forum en forum avec les
memes idees...

il appele "intelligence" le fait que les autres le laissent imposer ses
idees...
meme sans que personne ne lui reponde a la fin (ce qui explique peut etre
sa venue ici), il a reussi a plomber le forum anglais pendant longtemps ...

je ferais aussi remarquer que le probleme n'a pas surgit sur la meca q mais
quand j'ai juste repondu a un message sur l'existance d'un referentiel
absolu (tout comme il a traite d'idiots bcp d'autres personnes en mettant
copenhague sur des sujets tres varies meme sans rapport avec la physique)

donc pour resumer, je ne repond pas a cet idiot, ce qui ne l'a d'ailleurs
pas empeche de continuer, par contre je considere pouvoir m'exprimer sur ses
idees ce qu'il a tente d'empecher sur beaucoup de personnes et pas seulement
ici...

@+
ricky


newsgroups

non lue,
7 oct. 2002, 03:47:4807/10/2002
à

>
> Tel que je m'y attendais, le défi était trop dangeureux.
>
> Peur de faire un faux pas devans les pairs et de se faire rappeler
> publiquement à l'ordre. Peur de l'opinion des jeunes. Le piège
> insoluble de l'orthodoxie.
>
> On préfère s'esquiver.

Mais pas du tout, j'ai simplement découvert que j'essayais de raisonner
quelque chose d'aussi malin qu'une paramécie.
J'essaie une dernière fois qd même. Ma question était: selon toi un champ
magnétique
constant augmente t'il l'énergie cinétique d'un électron ? Si oui, explique
moi
stp selon quel mode ? Enfin, dans les accélérateurs du style du lep,
sais-tu à quoi servent les champs magnétiques (hint: f*c*l*s*t*o*)?
Il est grand temps que tu lises (et comprennes) quelques ouvrages
pédagogiques standard
à usage des lycéens, cela t'ouvrirait peut être un peu ce qui te tient
d'esprit.


François.

>


newsgroups

non lue,
7 oct. 2002, 03:51:3807/10/2002
à
Salut,

">
> donc pour resumer, je ne repond pas a cet idiot, ce qui ne l'a d'ailleurs
> pas empeche de continuer, par contre je considere pouvoir m'exprimer sur
ses
> idees ce qu'il a tente d'empecher sur beaucoup de personnes et pas
seulement
> ici...

tu as raison: le laisser conclure peut faire croire à certains que les
"idées"
qu'il déverse ad nauseam sont pertinentes, que sa loghorrée a un sens.
Il faut le bouter hors d'ici, enduit de goudron et couvert de plumes, car
ce n'est qu'un escroc stupide et sans scrupules.

François.


> @+
> ricky
>
>
>


ricky

non lue,
7 oct. 2002, 05:57:3507/10/2002
à
bonjour


> Pour fixer les idées, voir la description en de tels termes que
> j'avais mis dans mon texte disponible (.ps) sur
> http://spoirier.lautre.net/relativite.html

je peux me permettre un "jugement" perso sur ton site ?

d'abord, toujours un merci a ceux qui se decarcassent pour creer des sites
et mettre vraiment des idee, defendre leur theses

ensuite, les questions que tu poses sont importantes

par contre, je pense qu'il reste encore pas mal d'echange de point de vue
a effectuer pour finaliser tes reponses

je trouve certains points a mieux expliciter ...
par exemple, ta definition de la physique laisse a penser que ce n'est
qu'un monstre logique , dont la logique "regit" la matiere ... alors que
c'est plutot le contraire ! les objets sont regis par les lois de la
nature, et la physique tente de rendre compte de ces lois au travers de
divrs langages dont les mathematiques

ensuite, tu presente dans ce cadre les mathematiques comme une "science "
a part entiere, alors que je pense que les mathematiques sont
essentiellement un langage et un outil pour les physiciens ....

par contre j'ai bien apprecie ta vision du fait que la realite n'est
accessible qu'au travers de nos observations, ce qui est une evidence que
certains semblent vouloir outrepasser !

il y a encore pas mal de choses a discuter ;-)

@+
ricky : ps petit a petit, essaye aussi d'aerer ton site pour le rendre
plus accessible, si tu as le temps et l'envie bien sur :-)

Sylvain

non lue,
7 oct. 2002, 06:05:5507/10/2002
à
srp <s...@globetrotter.net> wrote in message news:<3DA0DE35...@globetrotter.net>...

> En réalité, elle n'est pas houleuse, et ce n'est même pas une
> discussion. C'est simplement mon refus total exprimé de ne
> discuter avec les tenants de l'école de Copenhague pour aucune
> considération.

Si vous voulez, on va essayer de faire abstraction des avis des autres
et même du mien afin que vous puissiez exprimer votre pensée en
détails.

> J'ai réussi à télécharger le fichier .ps mais ne n'ai pas réussi
> la le faire afficher. Je n'ai malheureusement pas réussi à
> charger le fichier pdf.

Si vous avez gsview et que vous êtes sous windows, essayez la
procédure que j'ai indiquée avec un msdos prompt: se placer dans le
répertoire convenable avec la commande cd (par exemple "cd bureau"),
puis faire "rename physique.ps.gz physique.ps".
Pour moi ça marche comme ça. Je sais c'est très étrange, il s'en passe
des choses bizarres dans les ordinateurs, l'explication qu'on m'a
donné ne me satisfait pas (j'ai plutôt tendance à penser que le
problème est dans l'hébergeur malgré ce qu'on dit), mais en
attendant...
Pour notre discussion cela commence à la page 25.

> Petite note, je suis en désaccord déjà avec l'énoncé suivant qui
> commence la section "Pour éviter tout contresens"
>
> "L'objet de la Physique est de décrire les lois physiques,
> c'est-à-dire les relations mathématiques qui contraignent
> le comportement de la matière.
>
> En ce qui me concerne, l'objet de la physique fondamentale est
> de décrire et "comprendre" les particules élémentaires et leurs
> interactions.

De toute façon à mon avis les descriptions et les compréhensions se
seront jamais autre chose que des théories mathématiques
particulières: je ne vois pas quel genre de description pourrait
échapper à cette règle. Bien sûr parmi les théories mathématiques vous
pouvez décider de distinguer celles que vous appelez des
compréhensions par opposition aux autres. Et cela peut réellement
faire avancer les choses, comme quand on est passé des relations
mathématiques de l'entropie à sa formulation statistique, laquelle est
plus riche en informations mathématiques. Je ne vois donc pas ce
désaccord comme fondamental.

> Les mathématiques ont pour objet de décrire, et non de contraindre
> le comportement de la matière. Rien ne peut changer la réalité.
> Elle existe indépendemment de ce qu'on en pense. Rien de ce qu'on
> peut en penser ne peut la contraindre.

Il n'est pas facile d'exprimer ce qu'on veut dire.
Bien sûr ce n'est pas les mathématiques en tant que fruits de
l'intelligence humaine qui contraignent ainsi la matière. Je voulais
parler des contraintes inhérentes à la matière, contraintes qui
correspondent à une forme mathématique dans le monde platonicien des
mathématiques et qui précède notre effort de compréhension. Notre
effort de construction mathématique est d'exprimer ces relations.
Bon, j'essaierai de réviser mon texte en ce sens. Merci de cette
remarque.

> > Etes-vous d'accord que c'est bien là la description des prédictions
> > expérimentales de la MQ sur cette question ?
>
> je doute que mon explication ait du sens pour vous si vous pensez au
> départ que le paradoxe EPR et son analyse est vraiment significative,

Qu'importe ce qui est significatif ou a du sens pour tel ou tel et les
analyses qu'on peut donner, il s'agit ici d'une expérience de physique
réelle, et c'est là-dessus que je voudrais connaître votre pensée.

> mais je vais essayer, malgré l'absence de votre texte.
>
> Au départ, je suis d'accord avec l'énonsé de EPR à l'effet que
> "Every element of the physical reality must have a counterpart
> in the physical theory".

> Mais je suis en désaccord avec la définition qu'ils font du
> "critère minimal de réalité"

Là on s'éloigne de la pure description des résultats de mesures
physiques pour entrer dans les commentaires philosophiques que les
auteurs de l'article ayant ouvert la question ont écrit dessus. En
fait je ne me suis pas intéressé de près à leur article, c'est
l'expérience elle-même qui m'intéresse.

>
> "If, without in any way disturbint a system, we can predict with
> certainty (i.e., with probability equal to unity) the value of a
> physical quantity, then there exists an element of physical reality
> corresponding to this physical quantity."
>
> En ce qui me concerne, on peut extrapoler n'importe quoi d'une
> expérience intellectuelle. Le seul critère d'existence physique
> d'une particule, par exemple, est la preuve démontrable et
> reproduisible qu'elle peut entrer en collision avec une autre
> particule.

Je ne vois pas quoi répondre, cela me semble des discussions de
principes qui nous éloignent du sujet. Plutôt que de faire tant de
discours philosophiques a priori je proposerais de partir de
l'expérience elle-même, qui se rapporte à la réalité de l'univers dans
lequel nous vivons en l'occurence, puis de discuter de ses éventuelles
interprétations, plutôt que de se perdre dans les énumérations
abstraites de l'ensemble des univers dont le caractère de réalité
serait acceptable en vue d'en conclure dans quel univers nous devrions
vivre.

> Selon moi, tout ce qui existe matériellement ne peut être construit
> que de ces particules et de ces interactions dont on peut vérifier
> directement l'existence.

C'est votre philosophie. Mais moi je veux vous parler d'expérience.

>
> > 2) Ces prédictions sont-elles vérifiées par l'expérience ?
>
> Je n'ai pas lu votre texte, mais dans ma perspective, la MQ n'est
> qu'une méthode descriptive des niveaux d'énergie des orbitales dans
> les atomes. Pas plus prédictive que ne l'est le calcul différentiel
> et intégral. Une simple méthode statistique et mathématique.

Il y a quand même eu une quantité considérable d'applications et de
vérifications, par exemple le calcul du moment magnétique de
l'électron... enfin j'ai honte de donner un exemple tellement ils sont
innombrables.
Qu'importe après tout ce que les physiciens décident de croire, du
moment que cela permet tant d'applications industrielles...

> Heisenberg et Bohr l'ont élevé au status de méthode prédictive
> seulement parce qu'ils étaient irrationnellement convaincu qu'elle
> définissait le réel, et que par conséquent, elle devait pouvait
> pouvoir prédire ce qui arrive dans le réel.

Elle donne des probabilités. On n'a encore jamais observé de façon
reconnue que l'expérience s'éloignait de la loi des grands nombres
résultant de ces probabilités.
Ensuite, si vous voulez trouver un autre fait plus profond et près de
la réalité qui permettrait de prédire plus précisément que ces lois de
probabilité dans certaines circonstances quel sera effectivement le
résultat parmi ces possibilités, je vous conseille d'ouvrir un casino
basé sur un tel dispositif expérimental et faire vos paris contre des
physiciens de Copenhague sur les résultats des expériences.
Finalement, si vous avez raison mais que les physiciens refusent
toujours de l'admettre, ce serait pour vous un avantage: vous pourriez
toujours continuer à faire des paris avec eux et toujours gagner.
Ainsi la non-reconnaissance obstinée de vos travaux par eux serait
pour vous une source de fortune illimitée.

> Mon opinion ici est que MQ ou pas, on aurait vérifié les mêmes
> choses expérimentalement.
>
> Autrement dit, ce n'est pas parce que la MQ existait qu'il a été

> possible de découvrir les quarks up et down dans les nucléons,

Je voulais concentrer la discussion sur l'expérience EPR.
Si on commence à parler de tout en même temps cela n'aidera pas à voir
clair dans quoi que ce soit.

> À mes yeux, ce sont des effets observables, point, MQ ou pas.

J'en suis fort aise. Mais je pense que le fait de prêter attention
précisément à ce que ces effets observables sont en l'occurence,
pourrait faire avancer la discussion. Il est trop facile de dire
"j'accepte les choses" sans faire attention à ce que sont les choses
qu'on accepte.

> > et si vous affirmez qu'il est possible d'interpréter la MQ
> > par une réalité sous-jacente,
>
> Il n'y a pas à interpréter la MQ. Tout ce qu'elle peut faire,
> elle le fait de manière raisonnable, soit rendre compte des
> niveaux d'énergie des orbitales.

et de l'expérience EPR aussi, il ne faut pas l'oublier.

> > alors dans quelle direction pensez-vous que l'interprétation de
> > ce phénomène devrait pouvoir se trouver ?
>
> Ben, pour moi, la MQ n'est pas un facteur dans l'équation. Elle
> est là à notre disposition lorsqu'on en a besoin. Le vrai problème
> est qu'il devient urgent, après 70 ans d'arrêt, de nous remettre
> au plus vite, à intégrer dans notre description intégrée de la
> réalité, les nouvelles données dont nous avons pu confirmer la
> réalité pendant tout ce temps

Lesquelles (désolé je n'ai pas suivi vos propos précédents) ?
Je suppose que vous parlez de données qui échappent à ce dont la MQ
peut rendre compte ?

> Le problème de la physique fondamentale depuis 70 ans, est que
> la MQ a été mise sur un piedestal par une foule d'adorateurs
> inconditionnels qui sont devenus incapables de voir la réalité
> physique sous-jacente qui est irrémédiablement masquée pour
> leurs yeux éblouis par la fonction d'onde.

Je suis curieux de savoir à quelle vision de la réalité sous-jacente
vous faites allusion: une vision qui apparaît dans quelles expériences
?
Salutations
SP

Gérard Samouillan

non lue,
7 oct. 2002, 04:33:0507/10/2002
à
Elle le défini effectivement autrement: c'est une interaction avec échange
d'énergie, et rien d'autre.
Le malheur, c'est que certains ont parlé de particules, ce qui a induit
beaucoup de monde en erreur.
il est aussi illégal de considérer le temps propre du photon que de
chercher à atteindre le pied d'un arc en ciel
GS
.
.


François Guillet <guillet....@wanadoo.fr> a écrit dans le message :
anmd08$fc3$1...@wanadoo.fr...
>
> "> Dire que ce n'est "pas un objet réel", qu'il "n'existe pas"... ne
répond pas
> à la question.
> Le "photon" est, contrairement à "arc en ciel", une terminologie issue non
> pas de l'expérience ou du sens commun, mais de la physique. Si elle ne le
> renie pas, alors il faudrait qu'elle le définisse autrement que par la
> négative.
>
> FG
>
>
>
>
>
>
>


Sylvain

non lue,
7 oct. 2002, 11:49:1007/10/2002
à
En relisant mon texte, je viens d'ajouter une remarque sur la notion
de spin de l'électron.
J'en profite pour mettre le texte sous forme non compressée pour
éviter le blocage:
http://spoirier.lautre.net/physique.ps

srp

non lue,
7 oct. 2002, 12:24:2907/10/2002
à
j'ai lu dans un autre message que vous disiez de moi :

"Il faut le bouter hors d'ici, enduit de goudron et couvert de

plumes, car ce n'est qu'un escroc stupide et sans scrupules."

Que dire de plus!

Fin de la conversation.

srp

non lue,
7 oct. 2002, 12:52:1507/10/2002
à
Sylvain a écrit :

Malheureusement, en tentant de le lire, j'obtient à chaque fois le
message "Can't load ghostscript.dll", peu importe comment je m'y
prend. En tentant de lire d'autres .gs, j'obtiens maintenant le
même message.

Il faudrait que je réinstalle GSview, mais je n'ai pas de temps
à consacrer à cette recherche pour le moment.

François Guillet

non lue,
7 oct. 2002, 17:06:1907/10/2002
à

"Gérard Samouillan" <gerard.s...@free.fr> a écrit dans le message de
news: 3da16fd4$0$21316$626a...@news.free.fr...

> Elle le défini effectivement autrement: c'est une interaction avec échange
> d'énergie, et rien d'autre.

Une interaction nécessite au moins 2 objets, ce ne peut donc être la
définition d'un seul.

En espace libre il interagit avec quoi ?

FG

srp

non lue,
7 oct. 2002, 18:39:4107/10/2002
à
Sylvain a écrit :

>
> srp <s...@globetrotter.net> wrote in message
> news:<3DA0DE35...@globetrotter.net>...
> > En réalité, elle n'est pas houleuse, et ce n'est même pas une
> > discussion. C'est simplement mon refus total exprimé de ne
> > discuter avec les tenants de l'école de Copenhague pour aucune
> > considération.
>
> Si vous voulez, on va essayer de faire abstraction des avis des
> autres et même du mien afin que vous puissiez exprimer votre
> pensée en détails.

J'exprime déjà ma pensée en détail depuis des années, en faisant
toujours abstraction des avis des autres.



> > J'ai réussi à télécharger le fichier .ps mais ne n'ai pas réussi
> > la le faire afficher. Je n'ai malheureusement pas réussi à
> > charger le fichier pdf.
>
> Si vous avez gsview et que vous êtes sous windows, essayez la
> procédure que j'ai indiquée avec un msdos prompt: se placer dans
> le répertoire convenable avec la commande cd (par exemple "cd
> bureau"), puis faire "rename physique.ps.gz physique.ps".
> Pour moi ça marche comme ça. Je sais c'est très étrange, il
> s'en passe des choses bizarres dans les ordinateurs, l'explication
> qu'on m'a donné ne me satisfait pas (j'ai plutôt tendance à penser
> que le problème est dans l'hébergeur malgré ce qu'on dit), mais en
> attendant...

J'ai tout essayé. Ma version de GSview doit être corrompue.

> Pour notre discussion cela commence à la page 25.

Si vous pouvez me fournir un format autre que .gs, je serai heureux
de le lire votre texte, s'il n'est pas trop extensif. Je n'ai pas
de problèmes avec le format .pdf



> > Petite note, je suis en désaccord déjà avec l'énoncé suivant qui
> > commence la section "Pour éviter tout contresens"
> >
> > "L'objet de la Physique est de décrire les lois physiques,
> > c'est-à-dire les relations mathématiques qui contraignent
> > le comportement de la matière.
> >
> > En ce qui me concerne, l'objet de la physique fondamentale est
> > de décrire et "comprendre" les particules élémentaires et leurs
> > interactions.
>
> De toute façon à mon avis les descriptions et les compréhensions
> se seront jamais autre chose que des théories mathématiques
> particulières:

Eh ben, ça commence plutôt mal! Je suis d'avis diamétralement opposé.

> je ne vois pas quel genre de description pourrait échapper à cette
> règle.

Toutes les descriptions imaginables.

Par exemple, la loi de Coulomb

"Deux particules ponctuelles électrisées, se repoussent ou
s'attirent, selon qu'elles sont de même signe ou de signes
contraires. La force mise en jeu, dirigée selon la droite
qui les joints, est proportionnelle à leurs charges et
inversement proportionnelle au carré de la distance qui
les sépare."

Aucune formulation mathématique de cette loi n'évoquera
jamais pour moi tout le mouvement qui est implicite dans
cette formulation verbale.

Le principe d'Archimède:

"Tout corps plongé dans un liquide subit une poussée verticale
ascendente égale au poids du liquide déplacé".

Etc. Etc.

> Bien sûr parmi les théories mathématiques vous
> pouvez décider de distinguer celles que vous appelez des
> compréhensions par opposition aux autres.

Pour moi, les théories mathématiques ne peuvent intervenir qu'à
posteriori, car il est impossible de décrire ce qu'on a pas
encore compris. Pour pouvoir décrire mathématiquement quelque
chose, il faut d'abord l'avoir compris.

Newton ne pouvait pas décrire avec ses mathématiques l'interaction
coulombienne, parce qu'il n'en avait jamais entendu parler. Maxwell
ne pouvait pas décrire avec ses mathématiques la nature quantique
de l'énergie parce qu'il n'en avait jamais entendu parler. Einstein
ne pouvait pas décrire avec ses mathématiques les trajectoires
hyperboliques dites "anormales" des sondes Pioneer 10 et 11, parce
qu'il n'en avait jamais entendu parler. Et aujourd'hui, on ne peut
pas décrire mathématiquement toute la mécanique électromagnétique
des particules, parce que nous ne la comprenons pas encore.

Donc, lorsque je réfléchis aux particules et leurs interactions,
je ne choisis pas entre les théories mathématiques, qui ne peuvent
décrire que ce que leurs auteurs avaient eux-mêmes compris de la
réalité physique. Je réfléchis aux particules elles-mêmes dans
toute la mesure où j'ai réussi à les visualiser.

> Et cela peut réellement faire avancer les choses, comme quand
> on est passé des relations mathématiques de l'entropie à sa
> formulation statistique, laquelle est plus riche en informations
> mathématiques. Je ne vois donc pas ce désaccord comme fondamental.

Bien au contraire. Il est tout à fait fondamental.

Je parle de la réalité physique, alors que vous voulez qu'on discute
des descriptions déjà existantes de cette réalité, qui sont par
définition incomplètes puisque nous n'avons encore compris toute
la mécanique fondamentale.

> > Les mathématiques ont pour objet de décrire, et non de contraindre
> > le comportement de la matière. Rien ne peut changer la réalité.
> > Elle existe indépendemment de ce qu'on en pense. Rien de ce qu'on
> > peut en penser ne peut la contraindre.
>
> Il n'est pas facile d'exprimer ce qu'on veut dire.
> Bien sûr ce n'est pas les mathématiques en tant que fruits de
> l'intelligence humaine qui contraignent ainsi la matière. Je
> voulais parler des contraintes inhérentes à la matière, contraintes
> qui correspondent à une forme mathématique dans le monde platonicien
> des mathématiques et qui précède notre effort de compréhension.
> Notre effort de construction mathématique est d'exprimer ces
> relations.

Je comprend ce que vous voulez dire. Mais je suis incapable
d'imaginer que toutes les contraintes inhérentes à la matière,
comme vous dites, auraient déjà été entièrement comprises. C'est
ce qui me fait perpétuellement garder mes distances avec les
théories mathématiques en général. Leur degré de précision ne
peut en aucun cas dépasser le degré de précision avec lequel
les contraintes physiques sous-jacentes ont été comprises par
leurs auteurs.

> Bon, j'essaierai de réviser mon texte en ce sens. Merci de cette
> remarque.

Heureux si j'ai pu vous rendre service.



> > > Etes-vous d'accord que c'est bien là la description des
> > > prédictions expérimentales de la MQ sur cette question ?
> >
> > je doute que mon explication ait du sens pour vous si vous
> > pensez au départ que le paradoxe EPR et son analyse est
> > vraiment significative,
>
> Qu'importe ce qui est significatif ou a du sens pour tel ou tel
> et les analyses qu'on peut donner, il s'agit ici d'une expérience
> de physique réelle, et c'est là-dessus que je voudrais connaître
> votre pensée.

Je dois vous dire que j'ai lu au fil du temps peut-être une bonne
douzaine de versions du débat au sujet du paradoxe EPR et de
l'inégalité de Bell. Celle de Selleri m'a semblé particulièrement
lucide.

En dernier recours, j'ai même récupéré les papiers originaux de
Einstein, Podolsky et Rosen, ainsi que la réponse de Bohr pour
analyser moi-même les textes.

Résultats, je suis toujours perplexe. Mon avis est, comme déjà
mentionné, que tant que nous ne comprendrons pas à fond la
mécanique électromagnétique des particules fondamentales,
nous (moi, en tout cas) ne pourrons pas trancher le débat.

Même chose pour l'expérience de Young.

Nous n'en savons tout simplement pas assez pour trancher (c'est
mon avis, du moins).



> > mais je vais essayer, malgré l'absence de votre texte.
> >
> > Au départ, je suis d'accord avec l'énonsé de EPR à l'effet que
> > "Every element of the physical reality must have a counterpart
> > in the physical theory".
> > Mais je suis en désaccord avec la définition qu'ils font du
> > "critère minimal de réalité"
>
> Là on s'éloigne de la pure description des résultats de mesures
> physiques pour entrer dans les commentaires philosophiques que les
> auteurs de l'article ayant ouvert la question ont écrit dessus. En
> fait je ne me suis pas intéressé de près à leur article, c'est
> l'expérience elle-même qui m'intéresse.

Mais comment bien analyser les mesures physiques tant qu'on ne
comprend pas totalement la nature fondamentale des particules,
et leurs interactions réelles?

Je pense que toutes les analyses que nous faisons sont à prendre
avec réserve.



> > "If, without in any way disturbint a system, we can predict with
> > certainty (i.e., with probability equal to unity) the value of a
> > physical quantity, then there exists an element of physical reality
> > corresponding to this physical quantity."
> >
> > En ce qui me concerne, on peut extrapoler n'importe quoi d'une
> > expérience intellectuelle. Le seul critère d'existence physique
> > d'une particule, par exemple, est la preuve démontrable et
> > reproduisible qu'elle peut entrer en collision avec une autre
> > particule.
>
> Je ne vois pas quoi répondre, cela me semble des discussions de
> principes qui nous éloignent du sujet. Plutôt que de faire tant de
> discours philosophiques a priori je proposerais de partir de
> l'expérience elle-même, qui se rapporte à la réalité de l'univers
> dans lequel nous vivons en l'occurence, puis de discuter de ses
> éventuelles interprétations, plutôt que de se perdre dans les
> énumérations abstraites de l'ensemble des univers dont le caractère
> de réalité serait acceptable en vue d'en conclure dans quel univers
> nous devrions vivre.

Encore ici, tout ce que je peux en dire est que je ne peux pas
conclure. Je suis convaincu que nous ne comprenons pas encore
à fond les interactions au niveau fondamental. Toute conclusion
à ce moment ci ne peut être qu'hypothétique. En cas de doute
J'ai toujours préféré réserver mon jugement et continuer à
accumuler de l'information.



> > Selon moi, tout ce qui existe matériellement ne peut être construit
> > que de ces particules et de ces interactions dont on peut vérifier
> > directement l'existence.
>
> C'est votre philosophie. Mais moi je veux vous parler d'expérience.

Moi aussi. Je ne parle que des propriétés vérifiées expérimentalement
de manière certaine. Je n'imagine pas fermer une hypothèse de manière
définitive dont tous les aspects ne seraient pas solidement ancrés
dans de telles vérifications.



> > > 2) Ces prédictions sont-elles vérifiées par l'expérience ?
> >
> > Je n'ai pas lu votre texte, mais dans ma perspective, la MQ n'est
> > qu'une méthode descriptive des niveaux d'énergie des orbitales dans
> > les atomes. Pas plus prédictive que ne l'est le calcul différentiel
> > et intégral. Une simple méthode statistique et mathématique.
>
> Il y a quand même eu une quantité considérable d'applications et
> de vérifications, par exemple le calcul du moment magnétique de
> l'électron... enfin j'ai honte de donner un exemple tellement ils
> sont innombrables.
> Qu'importe après tout ce que les physiciens décident de croire, du
> moment que cela permet tant d'applications industrielles...

La recherche opérationnelle ne peut être fondée que sur ce qui
a vraiment été compris de la réalité physique. La base comprise
il y plus de 50 ans est suffisante pour garder la recherche
opérationnelle afférente en vie pendant des siècles.

Il ne faut pas confondre les progrès en recherche opérationnelle
avec les avancées en recherche fondamentale.

> > Heisenberg et Bohr l'ont élevé au status de méthode prédictive
> > seulement parce qu'ils étaient irrationnellement convaincu qu'elle
> > définissait le réel, et que par conséquent, elle devait pouvait
> > pouvoir prédire ce qui arrive dans le réel.
>
> Elle donne des probabilités. On n'a encore jamais observé de façon
> reconnue que l'expérience s'éloignait de la loi des grands nombres
> résultant de ces probabilités.

Elle ne peut prévoir que ce qui dépend des niveaux d'énergie des
orbitales.

Elle était incapable de prévoir la structure interne des nucléons,
même statistiquement.

> Ensuite, si vous voulez trouver un autre fait plus profond et près
> de la réalité qui permettrait de prédire plus précisément que ces
> lois de probabilité dans certaines circonstances quel sera
> effectivement le résultat parmi ces possibilités, je vous conseille
> d'ouvrir un casino basé sur un tel dispositif expérimental et faire
> vos paris contre des physiciens de Copenhague sur les résultats des
> expériences.

Mais pourquoi qui que ce soit devrait-il perdre du temps à s'acharner
à faire mettre debout un cheval qui l'est déjà?

Le problème n'est pas là du tout.

> Finalement, si vous avez raison mais que les physiciens refusent
> toujours de l'admettre,

Je ne vois pas ce que vous voulez dire ici. Si j'avais raison à
propos de quoi?

> ce serait pour vous un avantage: vous pourriez toujours continuer
> à faire des paris avec eux et toujours gagner.
> Ainsi la non-reconnaissance obstinée de vos travaux par eux serait
> pour vous une source de fortune illimitée.

Quel travaux? Je crois que j'ai perdu le fil de votre pensée.

Je suis un simple vulgarisateur. Je ne fais que rediffuser des
connaissance et conclusions qui ne le sont plus dans les ouvrages
de référence standard.

La vulgarisation n'est pas une activité très lucrative, je vous
l'assure.

> > Mon opinion ici est que MQ ou pas, on aurait vérifié les mêmes
> > choses expérimentalement.
> >
> > Autrement dit, ce n'est pas parce que la MQ existait qu'il a été
> > possible de découvrir les quarks up et down dans les nucléons,
>
> Je voulais concentrer la discussion sur l'expérience EPR.
> Si on commence à parler de tout en même temps cela n'aidera
>`pas à voir clair dans quoi que ce soit.

D'accord. Mais je vous ai déjà donné mon avis sur EPR. Je suis
incapable de conclure sans plus d'information confirmée sur la
mécanique électromagnétique fondamentale des particules.

> > À mes yeux, ce sont des effets observables, point, MQ ou pas.
>
> J'en suis fort aise. Mais je pense que le fait de prêter attention
> précisément à ce que ces effets observables sont en l'occurence,
> pourrait faire avancer la discussion. Il est trop facile de dire
> "j'accepte les choses" sans faire attention à ce que sont les
> choses qu'on accepte.

Si vous pouviez être plus spécifique, je vous donnerais mon opinion.



> > > et si vous affirmez qu'il est possible d'interpréter la MQ
> > > par une réalité sous-jacente,
> >
> > Il n'y a pas à interpréter la MQ. Tout ce qu'elle peut faire,
> > elle le fait de manière raisonnable, soit rendre compte des
> > niveaux d'énergie des orbitales.
>
> et de l'expérience EPR aussi, il ne faut pas l'oublier.

Dans une certaine mesure, mais voir mes réserves plus haut.



> > > alors dans quelle direction pensez-vous que l'interprétation de
> > > ce phénomène devrait pouvoir se trouver ?
> >
> > Ben, pour moi, la MQ n'est pas un facteur dans l'équation. Elle
> > est là à notre disposition lorsqu'on en a besoin. Le vrai problème
> > est qu'il devient urgent, après 70 ans d'arrêt, de nous remettre
> > au plus vite, à intégrer dans notre description intégrée de la
> > réalité, les nouvelles données dont nous avons pu confirmer la
> > réalité pendant tout ce temps
>
> Lesquelles (désolé je n'ai pas suivi vos propos précédents) ?

Principalement la découverte de Abraham & Kaufmann au sujet de
la distinction à faire entre l'inertie longitudinale et l'inertie
transversale. Soit le fait que les quantités de mouvement sont
totalement insensibles à toute interaction transversale.

Pour le reste, le défaut d'intégration en une seule théorie
mécanique de l'ensemble des particules physiques stables.

La non intégration de l'anomalie (par rapport à la RG) vérifiée
des trajectoires de sondes spatiales.

> Je suppose que vous parlez de données qui échappent à ce dont
> la MQ peut rendre compte ?

C'est bien ce qu'il me semble.

> > Le problème de la physique fondamentale depuis 70 ans, est que
> > la MQ a été mise sur un piedestal par une foule d'adorateurs
> > inconditionnels qui sont devenus incapables de voir la réalité
> > physique sous-jacente qui est irrémédiablement masquée pour
> > leurs yeux éblouis par la fonction d'onde.
>
> Je suis curieux de savoir à quelle vision de la réalité
> sous-jacente vous faites allusion: une vision qui apparaît
> dans quelles expériences ?

La simple prise de conscience que la réalité physique est sous-
jacente à toutes les théories que nous avons élaborées à son
sujet, et que nous aurions intérêt à finir de l'explorer, de
manière à mettre nos théories au focus de manière définitive.

> Salutations

Idem.

ricky

non lue,
8 oct. 2002, 00:32:5008/10/2002
à
bonjour

je precise que ma repopnse s'adresse au FORUM

> Pour moi, les théories mathématiques ne peuvent intervenir qu'à
> posteriori, car il est impossible de décrire ce qu'on a pas
> encore compris. Pour pouvoir décrire mathématiquement quelque
> chose, il faut d'abord l'avoir compris.

ce n'est pas necessaire heureusement

en effet, les mathematiques sont une forme de description qui permettent des
deductions plus facilement que d'autres formes delangages...
ainsi, en decrivant certains processus connus sous forme mathematique, et en
appliquant des regles de la logique formelle, il est courant de trouver
d'autres descriptions , parfois de choses dont on n'avait pas connaissance

ainsi, certaines choses non comprises ont ete deduites grace au formalisme
mathematique

> ne pouvait pas décrire avec ses mathématiques les trajectoires
> hyperboliques dites "anormales" des sondes Pioneer 10 et 11, parce
> qu'il n'en avait jamais entendu parler.

comme cela a deja ete decrit, les trajectoirs de ces sondes ne sont pas
anormales ...
la sonde pionneer 10 a simplement ete deviee par un asteroide non connu au
moment de la deviation par exemple...

>Et aujourd'hui, on ne peut
> pas décrire mathématiquement toute la mécanique électromagnétique
> des particules, parce que nous ne la comprenons pas encore.

il n'est pas necessaire de comprendre quelque chose pour le deduire...
la mecanique Q par exemple, a sorti beaucoup d'equations dont le sens
profond n'a commence a etre compris que bien apres.. dans un premier temps,
on considere alors que l'equation est juste un "artifice" de calcul..
ensuite il arrive que cet artifice soit mieux compris et donne naissance a
la comprehension d'un mecanisme ...

> Je réfléchis aux particules elles-mêmes dans
> toute la mesure où j'ai réussi à les visualiser.

les mathematiques, comme le reste ne sont qu'une facon de decrire
quelquechose
a partir du moment ou l'on reflechi a quaelquechose, ou l'on visualise, on
utilise deja une forme de description.. apres , que le langage soit
mathematique, francais ou purement conceptuel est un autre probleme AMHA

> théories mathématiques en général. Leur degré de précision ne
> peut en aucun cas dépasser le degré de précision avec lequel
> les contraintes physiques sous-jacentes ont été comprises par
> leurs auteurs.

les mathematiques ne sont qu'un langage comme le francais ou meme comme le
langage utilise par le cerveau pour traduire les informations (a base d
epoids et de comparaisons abstraites)

> La recherche opérationnelle ne peut être fondée que sur ce qui
> a vraiment été compris de la réalité physique.

le probleme est que , jusqu'a present, tout a toujours ete remis en cause
tot ou tard...
en effet, come nos connaissances sont toujours limitees, rien n'a jamais ete
vraiment compris de maniere sur et pour toujours...
amis ocme on dit, il faut bien partir de quelque part pour avancer !

> Elle était incapable de prévoir la structure interne des nucléons,
> même statistiquement.

comme toute theorie, la notion de structure est elle meme une description
mathematique ... cette description existe et est decrite sous la forme de
decompositio d'equations...
apres cela, on peut soi donner un sens a chaque sous parties de l'equation
(et on obtient alors un edecomposition en structure interne) soit considere
cela comme artifice mathemaique et considerer alors que la reponse est autre
...

> Mais pourquoi qui que ce soit devrait-il perdre du temps à s'acharner
> à faire mettre debout un cheval qui l'est déjà?

parceque le cheval ne l'est pas.. il tremblote encore

> Je suis un simple vulgarisateur. Je ne fais que rediffuser des
> connaissance et conclusions qui ne le sont plus dans les ouvrages
> de référence standard.

reponse toujours destinee au groupe ...

dans les ouvrages de reference standard, comme il est facile de le lire un
peu partout, sur le net comme a flamarion, il n'est quasiment jamais fait
notion de reaite objective ou d'affirmation sur la perennite absolue de ce
qui est observe ... personne n'a jamais pu citer une seule phrase allant en
sens inverse

> La vulgarisation n'est pas une activité très lucrative, je vous
> l'assure.

certains qui font de la bonne vugarisation en vivent ...

> La non intégration de l'anomalie (par rapport à la RG) vérifiée
> des trajectoires de sondes spatiales.

comme tout le monde peut le voir en allant sur le site de la nasa, ces
defauts n'en sont pas et trouvent des explications parfaiement rationnels...

par exempel :
"Des chercheurs britanniques du Queen Mary and Westfield College viennent de
repérer un gros astéroïde en analysant de vieilles données de vol d'une
sonde américaine. En novembre et décembre 1992, la sonde Pionneer 10 a été
déviée de sa trajectoire pendant 25 jours par l'attraction gravitationnelle
d'un corps céleste inconnu. L'engin se trouvait alors au-delà de l'orbite de
Pluton, à 8,4 milliards de kilomètres de la Terre."

il serait interessant de chercher les vrais cas ou quelque chose echappe
vraiment a la RG !

> La simple prise de conscience que la réalité physique est sous-
> jacente à toutes les théories que nous avons élaborées à son
> sujet, et que nous aurions intérêt à finir de l'explorer, de
> manière à mettre nos théories au focus de manière définitive.

il est interessant de remarquer que la notion meme de "prise de conscience"
implique necesssairement une "interpretation" de l'esprit qui fait cette
prise de conscience.
de fait, on est deja psse dans le domaine de l'observable a ce moment et
non plus de l'information d'origine...
ensuite, il faut prendre conscience qu'une theorie n'a pas forcement de
realite sous jacente...
la realite existe et n'est accessibel qu'au travers de l'observable... le
but d'une theorie est de donner les bonne sinformations sur l'observable,
quitte a decrire quelquechose qui n'est pas la realite, faute de notre part
de pouvoir tout savoir ...

un exemple visuel simple est le probleme de trouver une courbe passant par
un nombre donne de point...
supposont que nous connaissions deux choses par exemple ... on represente
cela par deux points qui sont les observables...
par ces deux points, on peut faire passer une droite ou une parabole par
exemple ...
l'une peut etre vraiment la representation de la "realite" (on dira la
droite par exempel) alors que l'autre, qu donne pourtant bien le meme
resultat dans le domaine de l'observable, n'a pas de realite sous jacente
...

de meme, tant que nos connaissances seront incompletes (et il y a de fortes
chances qu'elles le restent indefiniment), on ne pourra jamais savoir si nos
theories sont celle sdecrivant vraiment la "realite sous jacente" ou des
theories dont le domaine d evalidite coincide simplement avec l'observable

@-

srp

non lue,
8 oct. 2002, 07:27:3008/10/2002
à
Pour les jeunes qui lisent ces lignes pour la première fois, ricky
est un triste individu qui me dénigre sur fsp depuis des mois,

destinées à paraître dans un dictionnaire du futur que vous
trouverez au lien mentionné ci-dessous.

--------------------------------------------------------
"École de pensée de Copenhague" :

--------------------------------------------------------
"Minus copenhagensis" :
--------------------------------------------------------
http://groups.google.com/groups?dq=&hl=fr&lr=&ie=UTF-8&threadm=3DA017E2.7F170F67%40globetrotter.net&prev=/groups%3Fhl%3Dfr%26group%3Dfr.sci.physique

newsgroups

non lue,
8 oct. 2002, 07:43:0608/10/2002
à

"srp" <s...@globetrotter.net> a écrit dans le message de news:
3DA1AA4B...@globetrotter.net...

> j'ai lu dans un autre message que vous disiez de moi :
>
> "Il faut le bouter hors d'ici, enduit de goudron et couvert de
> plumes, car ce n'est qu'un escroc stupide et sans scrupules."
>
> Que dire de plus!
>
> Fin de la conversation.

Et une dérobade de plus de l'ami michaud !
Tu es quelqu'un qui ne sait pas répondre aux questions les plus simples.
Malgré cela, tu as le culot de te mêler de sujets qui te
dépassent complètement, et une audace (les scientifiques ont
tort, j'ai raison) que seule une solide bêtise alliée à un ego
démesuré peuvent autoriser. S'il n'y avait que cela, tu serais
simplement un idiot parmi beaucoup d'autres. Mais il y a plus:
comme tu n'es absolument pas armé pour discuter de façon cohérente
de physique, tu as développé un ensemble de stratégies de contournement
d'une malhonnêteté ahurissante, et c'est pour cela que plus qu'un imbécile,
tu es un charlatan.


François.

ricky

non lue,
8 oct. 2002, 08:13:1208/10/2002
à
merci de prouver votre mauvaise fois !

j'ai bien preciser que ma reponse ne s'adressait pas a vous mais au forum

je constate que vous cherchez donc par tous les moyens d'empecher les personnes de s'exprimer, meme quand le message ne s'adresse pas a vous et ce dans
un fourm publique !

merci de votre demonstration


> Il tente obsessivement par tous les moyens de discuter avec
> moi

en aucun cas
comme demande, j'ai arrete de discuter avec vous depuis un certain temps ...
il faut arreter le cognac cher ami ..

par contre, je constate un besoin maladif chez vous de chercher une personne, comme vous le faits depuis des annees en tentant de les virer des
newsgroup que vous squatter. Aucune insinuation, juste un google ;-)


> malgré mon refus systématique, dans le but de me lasser pour
> éventuellement me faire taire sur ce forum,

c'est vous qui tentez de faire taire les autres, sur des reponses qui ne vous sont pas destinees en plus !
et vous ne refusez pas, vous tentez de provoquer !

> en insinuant que
> des résistances passées à d'autres présomptueux de son genre
> étaient une attitude systématique envers tout le monde.

il n'y a pas d'insinuation
www.google.fr le montre tres bien ... le nombre de reponse correspondants aux insultes anglaises et francaise est eloquant ;-)

@-
et toujours le meme SPAM ...

srp

non lue,
8 oct. 2002, 09:00:1108/10/2002
à
L'anonyme dont le pseudo est newsgroups, et qui est trop
m...... pour s'afficher à visage découvert a décidé de joindre
ses efforts à ceux de ricky pour tenter de me faire taire sur
ce forum.

Pour les jeunes qui lisent ces lignes pour la première fois, ricky
est un triste individu qui me dénigre sur fsp depuis des mois,
ainsi que tout le travail de vulgarisation fait par le Service
de Recherche Pédagogique pour l'éducation, parce que je refuse

systématiquement de discuter avec eux, car ils sont des
inconditionnels fanatisés de l'école de pensée de Copenhague,

car je suis un causaliste, comme de Broglie, Planck, Schrödinger,

Abraham et bien d'autres.

Ils tentent obsessivement par tous les moyens de discuter avec
moi malgré mon refus systématique, dans le but de me lasser pour
éventuellement me faire taire sur ce forum, en insinuant que

des résistances passées à d'autres présomptueux de son genre
étaient une attitude systématique envers tout le monde.

Comme vous pourrez le constater si vous relisez les enfilades dans

http://groups.google.com/groups?dq=&hl=fr&lr=&ie=UTF-8&threadm=3DA017E2.7F170F67%40globetrotter.net&prev=/groups%3Fhl%3Dfr%26group%3Dfr.sci.physique

--

srp

non lue,
8 oct. 2002, 09:03:1908/10/2002
à
Pour les jeunes qui lisent ces lignes pour la première fois, ricky
est un triste individu qui me dénigre sur fsp depuis des mois,
ainsi que tout le travail de vulgarisation fait par le Service
de Recherche Pédagogique pour l'éducation, parce que je refuse
systématiquement de discuter avec lui de physique, car je suis un
causaliste, comme de Broglie, Planck, Schrödinger, Abraham et bien
d'autres, et qu'il appartient, comme la majorité des physiciens
contemporains, à l'école de pensée de Copenhague.

Il intervient obsessivement sur mes messages, dans le but de
me lasser pour éventuellement me faire taire sur ce forum, en

insinuant que des résistances passées à d'autres présomptueux
de son genre étaient une attitude systématique envers tout le
monde.

Comme vous pourrez le constater si vous relisez les enfilades dans

ricky

non lue,
8 oct. 2002, 09:16:1008/10/2002
à
merci de vos efforts pour mettre un lien !

si vous pouviez maintenant mettre le lien directement en haut de la page, ou au second paragraphe aafin que vous puissiez quand meme ecrire le pseudo de
la personne visee, ce serait encore mieux ... la taille du message en serait d'autant plus courte sans en retirer le profond interet intrinseque
objectif, vu que c'est touours la meme chose

style :

personne visee = ...
lien vers le message = ...
spam : mon site qui vend toutes mes genales idees

voila, vous dites la meme chose en trois lignes :-)

merci de votre cooperation

gdlf

non lue,
8 oct. 2002, 09:45:0608/10/2002
à

"srp" <s...@globetrotter.net> a écrit

> Pour les jeunes qui lisent ces lignes pour la première fois, ricky
> est un triste individu qui me dénigre sur fsp depuis des mois,
> ainsi que tout le travail de vulgarisation fait par le Service
> de Recherche Pédagogique pour l'éducation, parce que je refuse
> systématiquement de discuter avec eux, car ils sont des
> inconditionnels fanatisés de l'école de pensée de Copenhague,
> car je suis un causaliste, comme de Broglie, Planck, Schrödinger,
> Abraham et bien d'autres.
>

Bien dit.

Il est temps que ces jeunes apprennent que le panchronisme est la seule
théorie physique contemporaine digne de ce nom.


srp

non lue,
8 oct. 2002, 10:39:5208/10/2002
à
La raison de mon mépris pour tous les tenants de l'école de
Copenhague, qui est devenu total depuis l'avénement de l'Internet,
est le constat que l'ensemble des physiciens tolèrent, parce
que ça les arrange bien, que des étroits d'esprit fanatiques,
le plus souvent anonymes, se chargent à leur place de harasser
et discréditer tous les opposants jusqu'à ce qu'ils abandonnent
à l'usure.

Quel déchéance pour l'orthodoxie en physique d'en être réduite
à se retrancher derrière de pareils minables! Il est urgent que
des physiciens responsables émergent de la génération montante,
et c'est pour en favoriser l'éclosion que je rediffuse certaines
découvertes qui sont systématiquement passées sous silence
dans les ouvrages de références dits "modernes".

François Müller

non lue,
8 oct. 2002, 11:33:1108/10/2002
à

> > --
> > André Michaud
> > Service de Recherche Pédagogique http://pages.globetrotter.net/srp/


C'est pas vrai. Encore lui. Et dire que je l'avais plonké.
On est encore passé du chat de Schrödinger à la jument de Michaud.


Jean Claude Calvez

non lue,
8 oct. 2002, 12:15:2808/10/2002
à

"srp" <s...@globetrotter.net> a écrit dans le message news:
3DA2C91D...@globetrotter.net...

> L'anonyme dont le pseudo est newsgroups, et qui est trop
> m...... pour s'afficher à visage découvert a décidé de joindre
> ses efforts à ceux de ricky pour tenter de me faire taire sur
> ce forum.
>
> Pour les jeunes qui lisent ces lignes pour la première fois, ricky
> est un triste individu qui me dénigre sur fsp depuis des mois,
> [cut]

Ce n'est pas le tout de faire des copier-coller, encore faut-il
les faire intelligemment : ricky n'est pas intervenu dans la
présente discussion, son nom n'a donc rien à faire ici.

De plus, il n'y a qu'un seul triste individu sur fsp : VOUS.

Enfin, foutez la paix aux "jeunes". Ils n'ont pas besoin
qu'un non-pédagogue de votre genre leur dise quoi
penser et quoi lire.

> --
> André Michaud
> Service de Recherche Pédagogique http://pages.globetrotter.net/srp/

Quelle pédagogie ????

--

Jean Claude Calvez
jeca...@wanadoo.fr


srp

non lue,
8 oct. 2002, 12:27:3608/10/2002
à
La raison de mon mépris pour tous les tenants de l'école de
Copenhague, qui est devenu total depuis l'avénement de l'Internet,
est le constat que l'ensemble des physiciens tolèrent, parce
que ça les arrange bien, que des étroits d'esprit fanatiques,
le plus souvent anonymes, se chargent à leur place de harasser
et discréditer tous les opposants jusqu'à ce qu'ils abandonnent
à l'usure.

Quelle déchéance pour l'orthodoxie en physique d'en être réduite

à se retrancher derrière de pareils minables! Il est urgent que
des physiciens responsables émergent de la génération montante,
et c'est pour en favoriser l'éclosion que je rediffuse certaines
découvertes qui sont systématiquement passées sous silence
dans les ouvrages de références dits "modernes".

Pour les jeunes qui lisent ces lignes pour la première fois,
ricky et quelques autres sont de tristes individus qui me dénigre
sur fsp depuis des mois, ainsi que tout le travail de vulgarisation

fait par le Service de Recherche Pédagogique pour l'éducation,

parce que je refuse systématiquement de discuter avec eux de
physique, car je suis un causaliste, comme de Broglie, Planck,
Schrödinger, Abraham et bien d'autres, et qu'il appartient,

comme la majorité des physiciens contemporains, à l'école de
pensée de Copenhague.

Il intervient obsessivement sur mes messages, dans le but de

http://groups.google.com/groups?dq=&hl=fr&lr=&ie=UTF-8&threadm=3DA017E2.7F170F67%40globetrotter.net&prev=/groups%3Fhl%3Dfr%26group%3Dfr.sci.physique

--

srp

non lue,
8 oct. 2002, 22:13:0508/10/2002
à
Sylvain a écrit :

Bon, j'ai enfin pu lire votre format .pdf

En passant, chapeau pour la clarté et la lisibilité de vos textes.

Voici mes remarques.

1) Au paragraphe 4, il y a présomption que "... le photon n'interagit
qu'avec des charges électriques"

Les expériences de l'équipe de McDonald au SLAC en 1997, qui
faisaient converger des jets concentrés de photons prouvent que
les photons peuvent interagir entre eux (des paires électron/positon
ont été créées de manière certaine au point de croisement)

Le photon est une particule électromagnétique, ce qui signifie
qu'il serait sensible à toutes les interaction électromagnétiques.

2) Il est correctement analysé que la direction du spin d'un
électron solitaire est indéterminée.

Le spin concerne l'aspect magnétique de la particule, sa direction
ne se détermine donc que relativement au spin d'autres particules.

Il est présumé dans le texte que "c'est l'expérimentateur qui,
pour observer ce sens, est libre d'en décider la direction comme
il veut."

Or les seuls cas possibles d'association d'électrons en doublets
sont ceux que les interactions locales provoquent lorsque les
électrons s'installent sur les orbitales des atomes, et lorsqu'ils
sont impliqués dans les liens covalents lors de la formation des
molécules. La direction est donc entièrement déterminée par les
interactions locales.

Il me semble que rien de ce qu'on peut penser ne peut influencer
la réalité. Il me semble que l'orientation relative du spin de
n'importe quel électron ne peut se déterminer qu'en conséquence
des interactions locales qu'il a à tout moment par rapport à
l'orientation de celui d'autres particules environnantes.

3)Il est présumé dans le texte que le principe d'exclusion de
Pauli n'implique pas qu'un doublet d'électrons puisse être dans
exactement le même état spatial lorsqu'ils sont associés par
spins antiparallèle. Or, à courte distance, l'interaction
magnétique domine, comme le montre les calculs de Heitler et
London, ce qui fait, selon moi, que la répulsion électrostatique
à si courte distance ne pourrait empêcher qu'ils se retrouvent
dans le même état statial.

Étant donné que l'expérience physique avec des électrons est
impossible pour les raisons d'interactions incontrôlables
que vous soulignez au début de votre texte, nous nous retrouvons
face à une expérience intellectuelle qui ne peut être concluante
en pratique.

Quant aux expériences faites avec des photons, Selleri rapporte,
entre autres difficultés majeures, que l'efficacité des détecteurs
ne dépasse pas les 20% en pratique, ce qu'il considère suffisemment
faible pour rendre les résultats inutilisables.

Au départ, de toute manière, si les photons sont sensibles
aux interactions électromagnétiques, comme les électrons, ce
que semble confirmer les expériences de l'équipe de McDonald,
toute les expériences EPR faites avec des photons souffrent
du même problème d'interactions incontrôlables que vous soulignez
pour les électrons.

Voilà. C'était mon avis.

Didier Lauwaert

non lue,
9 oct. 2002, 03:49:4609/10/2002
à
srp <s...@globetrotter.net> wrote in message news:<3DA37E9C...@globetrotter.net>...
> Sylvain a écrit :

> Les expériences de l'équipe de McDonald au SLAC en 1997, qui
> faisaient converger des jets concentrés de photons prouvent que
> les photons peuvent interagir entre eux (des paires électron/positon
> ont été créées de manière certaine au point de croisement)

Ah ? Ca été observé ? Intéressant ça.
En voyant les calculs de la théorie des champs
je pensais que c'était quelque peu hors de notre portée.
Comme quoi il ne faut jamais préjuger des progrès technologique :o)

Rappellons aussi que l'interaction à "quatre photon" (sans
création de paire) est théoriquement prévue
(mais encore plus difficile à observer).

snip

Concernant l'orientation du spin je suis d'accord.
La notion de spin de l'électron est quantique et
ne peut donc se traiter aussi simplement qu'on le
souhaiterait.

Sylvain

non lue,
9 oct. 2002, 16:22:3309/10/2002
à
1) Interactions du photon... bon je parlais des conditions courantes
où l'énergie n'est pas assez élevé pour que les photons aient une
chance significative d'interagir entre eux. Bien sûr à des énergies
élevées par photon il y a des risques d'interactions, mais pour la
lumière visible ces risques sont négligeables il me semble (sinon
l'image de la Lune serait complètement brouillée de par les
interactions de la lumière qui nous en vient avec celle du Soleil qui
la croise).

2) L'expérimentateur choisit la direction qu'il veut pour mesurer le
spin en agissant sur la forme des interactions locales auxquelles cet
électron sera soumis.

Dans le cas d'un atome d'hélium qui a une parfaite symétrie sphérique,
comment les interactions locales déterminent-elles la direction des
spins des électrons ?
A partir de là, en suivant le scénario que j'ai décrit qui ne rompt la
symétrie des interactions locales qu'au dernier moment (lors de la
mesure, et suivant la direction choisie), n'obtient-on pas les
résultats comme décrit ?

3) "pas exactement le même état spatial"
Pour rentrer dans les détails de ce que j'ai voulu dire, j'appelle
deux électrons dans le même état spatial si la fonction d'onde Psi des
positions respectives x,x' des deux électrons (distingués
conventionnellement suivant leur spin dans une direction donnée - déjà
je suppose ces spins opposés) est le carré tensoriel d'un état spatial
Phi, autrement dit de la forme
Psi(x,x')=Phi(x)Phi(x')
Phi étant donc l'état commun des deux électrons.
C'est bien cette notion de "même état" dont il est question dans le
principe d'exclusion
de Pauli (qui n'est comme on sait qu'une vague expression des
conséquences de l'antisymétrie de la fonction d'onde...).

Rien à voir avec l'idée que la distance entre les deux électrons doive
être nulle
Psi(x,x')=phi(x)delta(x-x')
où delta=masse de Dirac en 0 de l'espace à 3 dimensions.

Dans l'atome d'hélium donc, ces deux états sont différents mais
"proches" c'est-à-dire ont une bonne probabilité d'être égaux: il y a
une fonction Phi telle que Psi est proche du carré tensoriel de Phi au
sens de l'espace hilbertien, du genre que leur produit hermitien soit
comme 0.9.
Donc en ce sens, n'importe quelle interaction entre eux même
attractive les empêche de demeurer exactement dans le même état.
Mais peu importe.

>Étant donné que l'expérience physique avec des électrons est
>impossible pour les raisons d'interactions incontrôlables
>que vous soulignez au début de votre texte, nous nous retrouvons
>face à une expérience intellectuelle qui ne peut être concluante
>en pratique.

Qu'importe, du moment qu'on sait en théorie que cette expérience
serait possible si on y mettait les grands moyens (refroidissement du
rayonnement ambiant vers le zéro absolu).
Et ce qui est important aussi c'est que cette description n'est qu'une
manière de rendre intuitivement palpable (par un une visualisation de
son rapport plus simple à l'espace physique habituel) le même
phénomène de corrélation que celui qui a été vérifié expérimentalement
avec les photons: les paradoxes sont au fond identiques, la seule
différence est que leur compte-rendu est ici plus aisé pour la
compréhension.
Disons que la vérification expérimentale pour les photons sert de
justification à cette description pour les électrons, en sorte que
votre acceptation des faits avec les photons devrait vous amener à
accepter l'autre cas du même coup.
S'il y a une explication mécanique avec les photons, le dictionnaire
mathématique qui fait la correspondance entre les deux devrait la
traduire en une "explication mécanique" éventuellement très farfelue
mais concevable quand même avec les électrons (vous savez, les
transports de structures algébriques par des bijections qui en font
des isomorphismes...).
Et c'est une voie vers une telle explication que je vous demande.
En résumé, je voudrais que vous me montriez que l'ensemble des pistes
de recherche envisageables vers une telle explication n'est pas
l'ensemble vide, parce que moi je le vois vide...

A moins que vous estimiez que ces résultats qui sont des conséquences
logiques de la MQ, montrent par leur aberration que la MQ doit être
fausse ? (quand je dis fausse je veux dire que ses résultats seraient
faux, qu'importe qu'il y ait ou non possibilité de trouver une
explication derrière)

>Donc, lorsque je réfléchis aux particules et leurs interactions,
>je ne choisis pas entre les théories mathématiques, qui ne peuvent

>décrire que ce que leurs auteurs avaient eux-mêmes compris de la
>réalité physique. Je réfléchis aux particules elles-mêmes dans


>toute la mesure où j'ai réussi à les visualiser.

Vous définissez donc votre compréhension par une visualisation. Bien.
Voyons les conséquences qu'on peut en tirer (les conséquences que je
crois pouvoir tirer de
vos propos par l'absurde, non pas mes opinions).
Qu'est-ce qu'une visualisation ?
C'est une figure géométrique. Vous allez dire pas seulement, il y a
aussi la dynamique.
Qu'à cela ne tienne, un scénario dynamique qu'on imagine n'est rien
d'autre qu'une figure ou autre champ dans espace à quatre dimensions
feulleté par des hypersurfaces (chacune étant l'espace à trois
dimensions que vous visualisez à chaque instant).
A chaque point de cet espace à quatre dimension vous associez une
couleur, une dureté, une petite flèche ou n'importe quoi d'autre qui
se ramèneront toujours à des objets géométriques, quantitatifs ou
quelque chose de ce genre.

C'est donc un objet mathématique, une extension de notre bonne vieille
géométrie
euclidienne.
Le seul problème est de savoir laquelle, alors bien sûr ce ne sera pas
forcément une de celles auxquels les physiciens ont pensé jusqu'à
maintenant, n'empêche que c'en est une dans l'ensemble (définissable
mathématiquement: voir mon texte de logique dans ma page de géométrie)
des extensions possibles.

Bien sûr il y a une différence entre visualiser une figure et la
définir axiomatiquement. Il n'empêche que tout ce qu'on peut
visualiser est aussi mathématiquement définissable (de façon
éventuellement beaucoup plus lourde et impratiquable qu'une
visualisation directe je vous l'accorde).

>Je parle de la réalité physique, alors que vous voulez qu'on discute
>des descriptions déjà existantes de cette réalité, qui sont par
>définition incomplètes puisque nous n'avons encore compris toute
>la mécanique fondamentale.

Jamais dit ça. J'ai dit qu'il fallait choisir une théorie
mathématique. Je n'ai pas dit qu'il fallait la choisir parmi celles
qui ont déjà été étudiées.
Ou alors si vous parlez de compréhensions non mathématisables, alors
cela sort du domaine de la physique au sens de ce que nous pauvres
humains sur terre pourrons jamais comprendre et formuler (je fais
allusion ici à des expériences mystiques comme sont certaines NDE très
intéressantes que j'ai pu lire, ce qui nous emmênerait très loin du
sujet de cette discussion)

>Je comprend ce que vous voulez dire. Mais je suis incapable
>d'imaginer que toutes les contraintes inhérentes à la matière,
>comme vous dites, auraient déjà été entièrement comprises.

Non bien sûr. Mais certaines l'ont été à équivalence mathématique près
(de toute façon j'estime qu'on ne pourra jamais parler de quelque
chose mieux qu'à une équivalence mathématique près: dans le cas
contraire je vous défie de définir précisément le sens que vous
donneriez à ce cas contraire: une image visuelle dont les couleurs
seraient conformes aux véritables couleurs des particules ???)
(équivalence = traduction ou isomorphisme)

>Résultats, je suis toujours perplexe. Mon avis est, comme déjà
>mentionné, que tant que nous ne comprendrons pas à fond la
>mécanique électromagnétique des particules fondamentales,
>nous (moi, en tout cas) ne pourrons pas trancher le débat.

Ah bon ? Vous ne parlez que de votre incapacité à trancher le débat ?
Je croyais que que vous jugiez la compréhension des autres.

Quand on parle de trancher, on suppose que c'est parmi un ensemble de
voies de recherche envisageables. La question que je vous pose, c'est
: quelles hypothèses vous sembleraient envisageables ?
Une chose indubitable est que la MQ nous sort une loi de probabilité.
Cette loi peut être vraie ou fausse: envisagez-vous l'une, lautre
possibilité ou ces deux possibilités vous semblent-elles a priori
envisageables tant qu'on en saura pas plus ?

En l'occurence dans l'expérience EPR: on a en un lieu A une particule
et un observateur; loin de là en B une autre particule et un autre
observateur.
Le résultat de la mesure en A, qui se produit avant qu'une éventuelle
information lumineuse n'ait eu le temps de lui venir de B sur le choix
fait en B de la direction de la mesure, dépend-elle uniquement de
causes matérielles (qu'on ignore complètement, le problème n'est pas
là) qui sont localisées en A (dans le système (particule + observateur
en A) ? Ou bien subit-elle une influence réelle de l'autre plus vite
que la lumière ?
Qu'importe la nature éventuelle de cette influence.
Ma question est: soutenez-vous que oui, que non ou encore que le oui
et le non restent tous deux envisageables tant qu'on n'en saura pas
plus ?

Mes questions sont précises, sans pour autant présupposer quoi que ce
soit de la nature véritable des causes en jeu ni vous demander de la
connaître. C'est cela que j'appelais des "pistes de recherche". Mon
opinion personnelle est que l'ensemble des pistes de recherche
admissibles est vide. Si votre opinion est contraire, je vous prie de
m'indiquer quelles pistes de recherches seraient susceptibles de tenir
la route. Ou encore, rappel d'hypothèse envisageable, que les causes
soient locales mais que la réalité contredise cette loi de probabilité
issue de la MQ.

Merci de répondre en particulier à ces questions.
SP

srp

non lue,
9 oct. 2002, 18:54:0609/10/2002
à
Sylvain a écrit :

>
> 1) Interactions du photon... bon je parlais des conditions courantes
> où l'énergie n'est pas assez élevé pour que les photons aient une
> chance significative d'interagir entre eux. Bien sûr à des énergies
> élevées par photon il y a des risques d'interactions, mais pour la
> lumière visible ces risques sont négligeables il me semble

Très probable, surtout considérant les vitesses impliquées.

> (sinon l'image de la Lune serait complètement brouillée de par les
> interactions de la lumière qui nous en vient avec celle du Soleil
> qui la croise).

Tout à fait d'accord.

Mais ces interactions avec les particules des atomes que les photons
croisent dans l'appareillage de contrôle et d'interception ne peuvent
sûrement pas être ignorées, à mon sens, particulièrement sur
l'orientation de la polarité de la polarisation.

> 2) L'expérimentateur choisit la direction qu'il veut pour mesurer le
> spin en agissant sur la forme des interactions locales auxquelles cet
> électron sera soumis.
>
> Dans le cas d'un atome d'hélium qui a une parfaite symétrie sphérique,
> comment les interactions locales déterminent-elles la direction des
> spins des électrons ?

Je n'en sais rien. Je ne suis pas convaincu que la symétrie de
l'atome d'hélium soit si parfaitement sphérique. Symétrique, oui,
pour l'hélium 4, mais sphérique... je n'en sais rien.

> A partir de là, en suivant le scénario que j'ai décrit qui ne rompt
> la symétrie des interactions locales qu'au dernier moment (lors de
> la mesure, et suivant la direction choisie), n'obtient-on pas les
> résultats comme décrit ?

C'est ici que je n'arrive pas à conclure. La mesure ne peut se
faire qu'en capturant le photon. Le photon, par son inertie
extrêmement faible, me fait l'effet d'une plume qui serait
capturé par un cyclone. Comment sa polarité pourrait-elle ne
pas être affectée au dernier moment par les particules qu'elle
frôle en find de course?

Mon avis est que nous n'en savons pas assez pour conclure.

> 3) "pas exactement le même état spatial"
> Pour rentrer dans les détails de ce que j'ai voulu dire, j'appelle
> deux électrons dans le même état spatial si la fonction d'onde Psi
> des positions respectives x,x' des deux électrons (distingués
> conventionnellement suivant leur spin dans une direction donnée -
> déjà je suppose ces spins opposés) est le carré tensoriel d'un état
> spatial Phi, autrement dit de la forme
> Psi(x,x')=Phi(x)Phi(x')
> Phi étant donc l'état commun des deux électrons.
> C'est bien cette notion de "même état" dont il est question dans
> le principe d'exclusion de Pauli (qui n'est comme on sait qu'une
> vague expression des conséquences de l'antisymétrie de la fonction
> d'onde...).

C'est bien à la description de cet état que le principe d'exclusion
de Pauli réfère, mais l'état lui-même est physique.

Lorsque vous parlez du principe d'exclusion de Pauli,
moi, je ne considère que l'état physique réel d'une paire
d'électrons associés par spin antiparallèle, pas de leur
fonction d'onde. C'est un autre point qui me fait ne jamais
conclure sur toute cette affaire de paradoxe EPR. Le paradoxe
EPR, pour moi, est un paradoxe théorique. En réalité, il ne
peut pas y avoir paradoxe. Selon ma compréhension, chaque photon
qui circule interagit perpétuellement avec les autres particules
qu'il approche et croise sur sa trajectoire. La même chose pour
les électrons.

Je suis incapable de visualiser le trajet des particules à travers
la fonction d'onde, car cette fonction masque complètement le
fait que toute particule ne peut être en tout temps qu'un événement
électromagnétique localisé qui suit en tout temps sa trajectoire
de moindre action, trajectoire qui est déterminée à tout moment
par l'ensemble des interactions électromagnétiques entre cette
particules et les particules environnantes.

> Rien à voir avec l'idée que la distance entre les deux électrons
> doive être nulle Psi(x,x')=phi(x)delta(x-x')
> où delta=masse de Dirac en 0 de l'espace à 3 dimensions.

Je comprend l'idée.



> Dans l'atome d'hélium donc, ces deux états sont différents mais
> "proches" c'est-à-dire ont une bonne probabilité d'être égaux:
> il y a une fonction Phi telle que Psi est proche du carré tensoriel
> de Phi au sens de l'espace hilbertien, du genre que leur produit
> hermitien soit comme 0.9.
> Donc en ce sens, n'importe quelle interaction entre eux même
> attractive les empêche de demeurer exactement dans le même état.
> Mais peu importe.

Exact. peu importe.



> >Étant donné que l'expérience physique avec des électrons est
> >impossible pour les raisons d'interactions incontrôlables
> >que vous soulignez au début de votre texte, nous nous retrouvons
> >face à une expérience intellectuelle qui ne peut être concluante
> >en pratique.
>
> Qu'importe, du moment qu'on sait en théorie que cette expérience
> serait possible si on y mettait les grands moyens (refroidissement
> du rayonnement ambiant vers le zéro absolu).

Ça règlerait le problème pour les interactions avec les photons dans
l'enceinte, mais pas celui des interactions avec les particules des
atomes que l'électron croiseraient en chemin et en bout de course.

> Et ce qui est important aussi c'est que cette description n'est
> qu'une manière de rendre intuitivement palpable (par un une
> visualisation de son rapport plus simple à l'espace physique
> habituel) le même phénomène de corrélation que celui qui a été
> vérifié expérimentalement avec les photons:

C'est ici que j'ai des doutes, tout comme Selleri. Je ne crois
pas que la chose a été vérifiée de manière concluante.

> les paradoxes sont au fond identiques, la seule différence est que
> leur compte-rendu est ici plus aisé pour la compréhension.

Je vous l'accorde.

> Disons que la vérification expérimentale pour les photons sert de
> justification à cette description pour les électrons, en sorte que
> votre acceptation des faits avec les photons devrait vous amener à
> accepter l'autre cas du même coup.

C'est justement le noeud gordien, je ne crois pas que la vérification
expérimentale confirme le cas de manière certaine pour les photons.
J'ai les mêmes doutes et réserves que Selleri.

> S'il y a une explication mécanique avec les photons, le dictionnaire
> mathématique qui fait la correspondance entre les deux devrait la
> traduire en une "explication mécanique" éventuellement très farfelue
> mais concevable quand même avec les électrons (vous savez, les
> transports de structures algébriques par des bijections qui en font
> des isomorphismes...).

Je suis d'accord. Si la chose était possible pour les photons, je
suis d'accord qu'il ne serait pas exclus qu'elle le soit aussi
pour les électrons.

> Et c'est une voie vers une telle explication que je vous demande.
> En résumé, je voudrais que vous me montriez que l'ensemble des pistes
> de recherche envisageables vers une telle explication n'est pas
> l'ensemble vide, parce que moi je le vois vide...

Eh bien, à mon avis, cet ensemble est vide si on veut utiliser
la MQ pour résoudre le problème. Par compte, si on termine
l'exploration jusqu'à compréhension réelle de toute la mécanique
électromagnétique fondamentale des particules, je pense que nous
serons capables d'en extrapoler la véritable solution.


> A moins que vous estimiez que ces résultats qui sont des
> conséquences logiques de la MQ, montrent par leur aberration
> que la MQ doit être fausse ?

Je ne pense aucunement que la MQ soit fausse. Mais je pense que
ces résultats sont une conséquence de pousser la MQ plus loin
qu'une limite au-delà de laquelle elle dépasses ses propres
capacités prédictives.

La MQ et la fonction d'onde, par définition, sont incapables
de tenir compte des interactions électromagnétiques auxquelles
la particule en mouvement est soumise le long de sa trajectoire
de moindre action.

Je pense qu'on lui en demande tout simplement plus ici que ce
qu'elle peut offrir.

> (quand je dis fausse je veux dire que ses résultats seraient
> faux, qu'importe qu'il y ait ou non possibilité de trouver une
> explication derrière)

Mon avis est que, dans le cas EPR, ses résultats seraient faux,
pour la raison citée juste avant. Mais, encore, je pense que
nous ne le saurons de manière définitive que lorsque toute
la mécanique fondamentale des particules stables aura été
maîtrisée.



> >Donc, lorsque je réfléchis aux particules et leurs interactions,
> >je ne choisis pas entre les théories mathématiques, qui ne peuvent
> >décrire que ce que leurs auteurs avaient eux-mêmes compris de la
> >réalité physique. Je réfléchis aux particules elles-mêmes dans
> >toute la mesure où j'ai réussi à les visualiser.
>
> Vous définissez donc votre compréhension par une visualisation.

Exact.

> Bien. Voyons les conséquences qu'on peut en tirer (les conséquences
> que je crois pouvoir tirer de vos propos par l'absurde, non pas mes
> opinions).
> Qu'est-ce qu'une visualisation ?
> C'est une figure géométrique. Vous allez dire pas seulement, il y a
> aussi la dynamique.

Exact. La dynamique du mouvement interne de l'énergie qui constitue
chaque quanta d'énergie, et la dynamique des mouvements relatifs
des particules en fonction des interactions fondamentales.

> Qu'à cela ne tienne, un scénario dynamique qu'on imagine n'est rien
> d'autre qu'une figure ou autre champ dans espace à quatre dimensions
> feulleté par des hypersurfaces (chacune étant l'espace à trois
> dimensions que vous visualisez à chaque instant).

Pas à 4 dimensions, pas d'hypersurfaces. Oui, visualisation du
mouvement en continue.

> A chaque point de cet espace à quatre dimension vous associez une
> couleur, une dureté, une petite flèche ou n'importe quoi d'autre qui
> se ramèneront toujours à des objets géométriques, quantitatifs ou
> quelque chose de ce genre.

Pas à 4 dimensions, pas de couleur, pas de dureté, pas de flèche,
pas d'objet géométrique, oui quantitatif, quantique en fait.

Je n'associe rien à chaque point de l'espace. Pour moi n'existe
que des événements électromagnétiques discrets qui suivent en tout
temps leurs trajectoires de moindre action, qui dépend de leurs
interactions avec toutes les autres particules en mouvement.

> C'est donc un objet mathématique,

Absolument pas. À aucun niveau.

> une extension de notre bonne vieille géométrie euclidienne.

Absolument pas. Euclide comprendrait et adorerait, sans doute,
mais en perdrait sans doute ses dentiers d'étonnement.

> Le seul problème est de savoir laquelle, alors bien sûr ce ne
> sera pas forcément une de celles auxquels les physiciens ont
> pensé jusqu'à maintenant,

Il semblerait que non, sinon nous serions pas en train d'avoir
cette discussion.

> n'empêche que c'en est une dans l'ensemble (définissable
> mathématiquement: voir mon texte de logique dans ma page de
> géométrie) des extensions possibles.

Tout à fait. Tout ce qui est logique est définissable
mathématiquement.

> Bien sûr il y a une différence entre visualiser une figure et la
> définir axiomatiquement. Il n'empêche que tout ce qu'on peut
> visualiser est aussi mathématiquement définissable

Tout à fait mon avis.

> (de façon éventuellement beaucoup plus lourde et impratiquable
> qu'une visualisation directe je vous l'accorde).

Peut-être pas. Presque toutes les mathématiques déjà définies
sont valables, vous savez. Il reste peut-être juste une petite
mise au focus à faire, qui sait.

> >Je parle de la réalité physique, alors que vous voulez qu'on
> >discute des descriptions déjà existantes de cette réalité, qui
> >sont par définition incomplètes puisque nous n'avons encore
> >compris toute la mécanique fondamentale.
>
> Jamais dit ça. J'ai dit qu'il fallait choisir une théorie
> mathématique. Je n'ai pas dit qu'il fallait la choisir parmi
> celles qui ont déjà été étudiées.

Ok.

> Ou alors si vous parlez de compréhensions non mathématisables,

Du tout. Encore une fois, je pense que toute ce qui est logique
est mathématisable.

> alors cela sort du domaine de la physique au sens de ce que nous
> pauvres humains sur terre pourrons jamais comprendre et formuler
> (je fais allusion ici à des expériences mystiques comme sont
> certaines NDE très intéressantes que j'ai pu lire, ce qui nous
> emmênerait très loin du sujet de cette discussion)

Je ne crois ni à la magie, ni aux perceptions extra-sensorielles.

Je ne crois qu'à ce qui peut être physiquement vérifié de manière
non-équivoque.

> >Je comprend ce que vous voulez dire. Mais je suis incapable
> >d'imaginer que toutes les contraintes inhérentes à la matière,
> >comme vous dites, auraient déjà été entièrement comprises.
>
> Non bien sûr. Mais certaines l'ont été à équivalence mathématique
> près

Bien sûr.

> (de toute façon j'estime qu'on ne pourra jamais parler de
> quelque chose mieux qu'à une équivalence mathématique près: dans
> le cas contraire je vous défie de définir précisément le sens que
> vous donneriez à ce cas contraire: une image visuelle dont les
> couleurs seraient conformes aux véritables couleurs des particules
> ???) (équivalence = traduction ou isomorphisme)

Eh bien, je pense qu'on ne pourra pas décrire les choses
de manière plus précise qu'on les comprend par ailleurs.

La limite se trouve, à mon sens, à être le degré de finesse
de la compréhension que nous acquérons.

> >Résultats, je suis toujours perplexe. Mon avis est, comme déjà
> >mentionné, que tant que nous ne comprendrons pas à fond la
> >mécanique électromagnétique des particules fondamentales,
> >nous (moi, en tout cas) ne pourrons pas trancher le débat.
>
> Ah bon ? Vous ne parlez que de votre incapacité à trancher le
> débat ? Je croyais que que vous jugiez la compréhension des
> autres.

Je dis qu'à mon avis, nous n'en savons pas assez (que personne
n'en sait assez) pour trancher le débat. Tant que nous n'aurons
pas compris complètement la mécanique fondamentale, je pense que
nous en resterons à ce niveau (de ne pas pouvoir trancher).

Pourquoi pensez-vous que le paradoxe EPR et les inégalités de
Bell sont encore sujet à controverse après tant de décennies?



> Quand on parle de trancher, on suppose que c'est parmi un ensemble
> de voies de recherche envisageables.

Bien sûr. Mais comment prévoir les vois de recherches encore
inconnues qui deviendront envisageables lorsque nous en saurons
plus sur la mécanique fondamentale?

> La question que je vous pose, c'est : quelles hypothèses vous
> sembleraient envisageables ?

Je n'en sais rien. Nous verrons dans 10 ans, 20 ans, lorsque
la compréhension de la matière sera plus profonde.

> Une chose indubitable est que la MQ nous sort une loi de probabilité.
> Cette loi peut être vraie ou fausse: envisagez-vous l'une, lautre
> possibilité ou ces deux possibilités vous semblent-elles a priori
> envisageables tant qu'on en saura pas plus ?

Voilà! Les deux me semblent possibles, et je crois que nous n'en
savons pas assez pour trancher.



> En l'occurence dans l'expérience EPR: on a en un lieu A une particule
> et un observateur; loin de là en B une autre particule et un autre
> observateur.

Pour moi, les observateurs ne font pas partie de l'équation.
Lorsqu'ils "observent", l'événement est déjà terminé, parce que
ce qu'ils observent, ce sont les signaux des détecteurs qui leur
parviennent après que l'événement est terminé.

> Le résultat de la mesure en A, qui se produit avant qu'une éventuelle
> information lumineuse n'ait eu le temps de lui venir de B sur le choix
> fait en B de la direction de la mesure, dépend-elle uniquement de
> causes matérielles (qu'on ignore complètement, le problème n'est pas
> là) qui sont localisées en A (dans le système (particule + observateur
> en A) ? Ou bien subit-elle une influence réelle de l'autre plus vite
> que la lumière ?

Je pense que autant les signaux de B que ceux de A lui parviennent
après que l'événement est terminé. donc, ce qu'il conclue ne peut
en aucun cas influencer ce qui est déjà dans le passé. Rien ne
peut changer le passé.

> Qu'importe la nature éventuelle de cette influence.
> Ma question est: soutenez-vous que oui, que non ou encore que le oui
> et le non restent tous deux envisageables tant qu'on n'en saura pas
> plus ?

Dans le cas d'une influence potentielle de l'observateur sur le
résultat observé (obligatoirementà posteriori, ne l'oublions pas,
peu importe comment on considère la question) je dis que c'est
physiquement impossible. Je pense que rien ne peut changer le
passé.



> Mes questions sont précises, sans pour autant présupposer quoi que
> ce soit de la nature véritable des causes en jeu ni vous demander
> de la connaître. C'est cela que j'appelais des "pistes de recherche".

Elles sont précises. Je tente d'y répondre aussi précisément que
je les comprends.

> Mon opinion personnelle est que l'ensemble des pistes de recherche
> admissibles est vide.

Par le biais de la MQ et de la fonction d'onde. C'est mon opinion
aussi.

> Si votre opinion est contraire, je vous prie de m'indiquer quelles
> pistes de recherches seraient susceptibles de tenir la route.

Mon opinion est que ces pistes de recherche vont se faire jour
d'ici quelques années, lorsque nous comprendrons plus profondément
la nature des particules fondamentales et leurs interactions
réelles.

> Ou encore, rappel d'hypothèse envisageable, que les causes soient
> locales mais que la réalité contredise cette loi de probabilité
> issue de la MQ.

Ce que je pense ici, comme je l'ai déjà mentionné, est que la
MQ fait abstraction de trop de choses au niveau fondamental.
Je pense qu'on a tenté de gongler artificiellement la petite
grenouille MQ jusqu'à la rendre aussi grosse qu'une chèvre.

On a ensuite demandé à notre pseudo-chèvre de se mettre à manger
de l'herbe en nous imaginant que sous sa nouvelle apparence elle
serait capable de la digérer pour nous donner du lait. Le malheur
est que la grenouille qu'on a gonflé n'est pas un herbivore et
qu'elle est incapable de donner du lait.


> Merci de répondre en particulier à ces questions.

Je ne sais pas si mes réponses ont du sens pour vous, mais
j'ai fait de mon mieux pour être clair.

Sylvain

non lue,
10 oct. 2002, 10:43:2010/10/2002
à
srp <s...@globetrotter.net> wrote in message news:<3DA49D3A...@globetrotter.net>...

>Le paradoxe EPR, pour moi, est un paradoxe théorique.

On peut en faire des descriptions théoriques en MQ, mais il se réfère
à une expérience réelle sur laquelle la MQ fait des prédictions
précises.

>Je suis incapable de visualiser le trajet des particules à travers
>la fonction d'onde, car cette fonction masque complètement le
>fait que toute particule ne peut être en tout temps qu'un événement
>électromagnétique localisé qui suit en tout temps sa trajectoire
>de moindre action, trajectoire qui est déterminée à tout moment
>par l'ensemble des interactions électromagnétiques entre cette
>particules et les particules environnantes.

Votre conception de la réalité est conforme à celle de la mécanique
classique, ce qui nous ramène un siècle en arrière, du temps où on ne
connaissait pas la MQ et où donc tous ses développement étaient
inconcevables. Pourtant vous avez admis qu'il y a eu un progrès, que
cela a permis la compréhension de beaucoup de choses (en fait de
beaucoup plus de choses que les orbitales atomiques).
Mais finalement, y a-t-il la moindre chose pour laquelle la MQ a
apporté une explication (par exemple les orbitales atomiques !?) et
que vous êtes effectivement capable d'expliquer au moins aussi bien
sans les fonctions d'onde ?

Si toute votre oeuvre consiste à tout nier et détruire des
connaissances accumulées au cours du 20ème siècle, comment pourrait-on
reconstruire quelque chose ? Avez-vous la moindre piste à proposer
pour cela ?
S'il suffit de s'y mettre, pourquoi ne vous y êtes-vous pas mis
vous-même ?
Et si vos arguments sont valables, comment se fait-il qu'aucun jeune
ne vous ait suivi et ne soit effectivement parvenu à des explications
capables d'égaler en puissance explicative/prédictive la formulation
standard de la MQ avec son équation de Schrodinger qui n'est pas si
compliquée que cela) ?

Et les quelques grands physiciens que vous citez (Einstein,...) qui
étaient du même avis que vous à ce que vous dites, comment n'ont-ils
pas été capables de proposer la moindre explication alternative de ces
phénomènes sans s'appuyer sur les fonctions d'onde, qui rende compte
d'au moins autant de choses que l'a fait l'école de Copenhague ?

>Mon avis est que, dans le cas EPR, ses résultats seraient faux,
>pour la raison citée juste avant. Mais, encore, je pense que
>nous ne le saurons de manière définitive que lorsque toute
>la mécanique fondamentale des particules stables aura été
>maîtrisée.

Enfin une assertion précise.

J'ai assisté un jour à Grenoble à un séminaire sur l'expérience EPR.
Il disait en gros (je ne sais plus le détail) que les conditions de
validité de ces expériences n'étaient au départ pas remplies ce qui
permettait des explications classiques. Mais petit à petit en affinant
les expériences, telle ou telle "explication" classique (ad hoc)
tombait, en sorte que finalement on n'avait pas complètement rempli
tous les critères à la fois mais on n'en était plus très loin, et il
n'y avait donc plus de bonne raison physique pour que la réalité
complote ainsi à opérer des moyens tirées par les cheveux pour
produire les résultats que la MQ donne naturellement.
Qu'on pourra sans doute à l'avenir continuer à affiner l'expérience
pour remplir tous les critères mais qu'il est raisonnable de penser
que les résultats de la MQ seront toujours valables car concrètement
on ne voit pas au nom de quel mécanisme naturel ils deviendraient
subitement faux.

De toute façon qu'on soit d'accord ou pas, il est clair qu'il devrait
être possible à l'avenir de réaliser cette expérience qui tranchera le
débat.
Même si personne n'y comprend rien.

Que diriez-vous, plutôt que de passer votre temps à traiter tout le
monde de noms d'oiseaux, d'attendre plutôt tranquillement que cela
arrive, voire de participer activement à la la mise au point de cette
expérience d'enjeu si capital, puisqu'elle permettra enfin de conclure
qui a raison ?

>Pas à 4 dimensions, pas d'hypersurfaces. Oui, visualisation du
>mouvement en continue.

M'enfin, c'est mathématiquement la même chose !
Une visualisation en continu est un espace d'évènements où à chaque
évènement on associe l'instant où on le voit (ou l'imagine).
Ou si on préfère, c'est le fait d'associer à chaque instant
(l'ensemble des instants étant une ligne continue) une image en 3
dimensions: même si le mouvement est continu rien ne vous empêche de
faire une photo quand vous voulez. Et cette union disjointe d'espaces
à 3 dimensions le long d'un temps continu est bien mathématiquement un
espace à 4 dimensions.

>oui quantitatif

L'ensemble N des nombres entiers et l'ensemble R des nombre réels sont
des objets mathématiques. Quant aux quantités physiques (masses,
longueurs...), ce sont des éléments de droites vectorielles sur le
corps des nombres réels. Tout cela très mathématique (même si on n'a
pas l'habitude de présenter des calculs dimensionnés en cours de
mathématiques).

>quantique en fait

Qu'est-ce à dire ???

>Je n'associe rien à chaque point de l'espace. Pour moi n'existe
>que des événements électromagnétiques discrets qui suivent en tout
>temps leurs trajectoires de moindre action, qui dépend de leurs
>interactions avec toutes les autres particules en mouvement.

Ce sont donc des courbes dans l'espace à quatre dimensions. Les
courbes sont des ensembles continus de points...

>> C'est donc un objet mathématique,
>
>Absolument pas. À aucun niveau.

Vous n'avez donc aucune notion sur le sens du mot "mathématique".

>Pourquoi pensez-vous que le paradoxe EPR et les inégalités de
>Bell sont encore sujet à controverse après tant de décennies?

[Attention: ce que je vais répondre ici n'est que mon opinion
personnelle qui ne prétend pas refléter celle de la majorité des
physiciens]
Parce que c'est un résultat qui heurte la philosophie réaliste de
beaucoup de gens qui refusent d'admettre que la réalité physique (au
sens de ce qui est du domaine de la science physique, c'est-à-dire ce
qui est mathématisable) n'est pas indépendante.
Parce qu'il y a bien d'autres gens qui rechignent à accepter ce
verdict de la réalité physique elle-même qui a montré son incomplétude
à travers la manifestation des relations mathématiques qu'elle suit
(celles de la MQ) qui font elles-même la démonstration (à travers
leurs conséquences logiques) de leur propre incomplétude et plus
précisément de leur incomplétabilité (pire qu'avec le théorème de
Godel car là on peut toujours avancer en ajoutant des informations qui
manquent et il en manquera d'autres; ici par contre c'est un point
final sur la question de l'incertitude - ce qui n'empêche pas plein
d'autres découvertes dans d'autres directions bien sûr).

La différence, c'est que beaucoup de gens admettent les faits en
pratique sans s'occuper de leurs implications philosophiques, que cela
puisse ou non les heurter lorsqu'ils y réfléchissent.
D'autres (dont je fais partie comme beaucoup de croyants) trouvent ces
faits naturels parce qu'il est naturel que la réalité physique n'est
qu'une partie de la réalité et n'est pas indépendante.


>> La question que je vous pose, c'est : quelles hypothèses vous
>> sembleraient envisageables ?
>
>Je n'en sais rien. Nous verrons dans 10 ans, 20 ans, lorsque
>la compréhension de la matière sera plus profonde.

à comparer à:

> Mon avis est que, dans le cas EPR, ses résultats seraient faux,
>pour la raison citée juste avant. Mais, encore, je pense que
>nous ne le saurons de manière définitive que lorsque toute
>la mécanique fondamentale des particules stables aura été maîtrisée.

Voilà une belle incertitude quantique sur votre position.
A quoi peut rimer la virulence de vos critiques contre le reste du
monde si vous ne savez même pas où vous en êtes dans votre position ?

Entre non et peut-être il faut choisir.
Si vous êtes en train de réfléchir, allez mener vos réflexions dans
votre coin et revenez nous voir quand vous serez au clair sur la
limite de votre propre savoir.
Si vous ne savez même pas ce que vous savez, comment pourriez-vous
juger de ce que les autres ont le droit de savoir ?

Jusqu'à quand condamnerez-vous le monde entier de folie au nom de
l'affirmation tonitruante du fait que vous ne savez personnellement
rien à rien et surtout pas quelles sont les limites de ce que vous
savez, tout en démontrant que vous ignorez aussi les limites de ce que
savent les autres ?

Décidez-vous, oui ou non, de vous risquer à affirmer que les résultats
de la MQ sont faux (au sens de faux = parfois faux, par opposition à
vrai = toujours vrai ; et au sens expérimental puisque c'est un énoncé
d'expérience précise)
et qu'on s'en apercevra quand on fera une expérience plus fine de EPR
?

C'est tout ce que je voulais savoir. Rendez-vous donc le jour où cette
expérience sera réalisée.

>Pour moi, les observateurs ne font pas partie de l'équation.
>Lorsqu'ils "observent", l'événement est déjà terminé, parce que
>ce qu'ils observent, ce sont les signaux des détecteurs qui leur
>parviennent après que l'événement est terminé.

Remplacez "observateur" par "détecteur". Ca va mieux comme ça ?

>lorsque nous en saurons plus sur la mécanique fondamentale

A partir de quelles informations ? Développements purement théoriques
ou bien nouvelles expériences contredisant les théories ?
Par exemple, si sur l'expérience EPR la réalité s'accordait à
l'inégalité de Bell contre les résultats de la MQ, cela constituera un
résultat précis à examiner.

En attendant, la théorie a toujours été vérifiée par l'expérience. Si
la vraie théorie donne des résultats contraires à l'ancienne sur les
domaines expérimentaux accessibles, on devrait constater effectivement
un désaccord entre les théories actuelles et l'expérience.
Si le désaccord porte sur des domaines inexplorés, c'est un
développement des expériences qui permettra de découvrir ce désaccord.
S'il n'y a pas de désaccord observable, la vraie théorie est alors
toujours d'accord avec celle actuelle sur ce qui est accssible, en
sorte qu'elle ne serait qu'une reformulation des théories actuelles,
quelque chose de mathématiquement équivalent qu'on pourrait découvrir
par des moyens purement théoriques par développement à partir des
formules établies.

Laquelle de ces options prenez-vous donc ?

>Je pense que autant les signaux de B que ceux de A lui parviennent
>après que l'événement est terminé. donc, ce qu'il conclue ne peut
>en aucun cas influencer ce qui est déjà dans le passé. Rien ne
>peut changer le passé.

Voilà qui est une affirmation claire. N'implique-t-elle pas
logiquement l'inégalité de Bell, et donc la fausseté de la loi de
probabilité que donne la MQ sur ce sujet ?

>> Mon opinion personnelle est que l'ensemble des pistes de recherche
>> admissibles est vide.
>
>Par le biais de la MQ et de la fonction d'onde. C'est mon opinion
>aussi.

Encore une fois je pense que vous n'avez pas saisi ce que j'entendais
par "piste de recherche".
Vous vous référez à la manière de penser et de comprendre. Ce qui fait
planer la discussion dans de vagues philosophies dépourvues
d'affirmations concrètes à discuter, et où on pourra toujours
s'étriper indéfiniment à ne pas comprendre la pensée de l'autre.

Mais ce que j'espérais savoir de vous, c'est des affirmations précises
sur les résultats expérimentaux: contrediront-ils, oui ou non, les
lois de probabilité issues de la MQ ?

>Mon opinion est que ces pistes de recherche vont se faire jour
>d'ici quelques années, lorsque nous comprendrons plus profondément
>la nature des particules fondamentales et leurs interactions
>réelles.

Ben alors dépêchez-vous d'explorer ces pistes de recherche si
prometteuses et revenez nous voir quand vous les aurez trouvées.

srp

non lue,
10 oct. 2002, 14:04:2610/10/2002
à
Sylvain a écrit :

>
> srp <s...@globetrotter.net> wrote in message
> news:<3DA49D3A...@globetrotter.net>...
> >Le paradoxe EPR, pour moi, est un paradoxe théorique.
>
> On peut en faire des descriptions théoriques en MQ, mais il se
> réfère à une expérience réelle sur laquelle la MQ fait des
> prédictions précises.

... que l'analyse de Selleri nous révèle n'être pas confirmée
et pas confirmable expérimentalement, principalement dû au
degré de précision trop faible des meilleurs de nos instruments
de mesure.

> >Je suis incapable de visualiser le trajet des particules à travers
> >la fonction d'onde, car cette fonction masque complètement le
> >fait que toute particule ne peut être en tout temps qu'un événement
> >électromagnétique localisé qui suit en tout temps sa trajectoire
> >de moindre action, trajectoire qui est déterminée à tout moment
> >par l'ensemble des interactions électromagnétiques entre cette
> >particules et les particules environnantes.
>
> Votre conception de la réalité est conforme à celle de la mécanique
> classique,

Ma conception de la réalité n'a rien à voir avec la mécanique
classique.

> ce qui nous ramène un siècle en arrière, du temps où on ne
> connaissait pas la MQ et où donc tous ses développement étaient
> inconcevables. Pourtant vous avez admis qu'il y a eu un progrès,
> que cela a permis la compréhension de beaucoup de choses (en fait
> de beaucoup plus de choses que les orbitales atomiques).

Bien sûr. Beaucoup de choses ont été vérifiées expérimentalement
au cours du 20e siècle.

C'est pourquoi je pense qu'il est temps qu'une nouvelle synthèse
émerge, fondée sur ce nouvel ensemble plus complet.

> Mais finalement, y a-t-il la moindre chose pour laquelle la MQ a
> apporté une explication (par exemple les orbitales atomiques !?)

La MQ n'a pas apporté une explication pour les orbitales atomiques,
elle en permet seulement la mesure.

> et que vous êtes effectivement capable d'expliquer au moins aussi
> bien sans les fonctions d'onde ?

Mais bien sûr, comme quiconque comprend que les orbitales ne peuvent
correspondre qu'aux longueurs d'ondes intègres permises pour les
électrons dans les atomes.

> Si toute votre oeuvre consiste à tout nier et détruire des
> connaissances accumulées au cours du 20ème siècle,

De quelle oeuvre parlez-vous? Et qui parle de détruire? Je parle
au contraire de synthétiser en un nouvel ensemble plus complet
toutes les connaissances accumulées expérimentalement au niveau
fondamental, y compris surtout la découverte de Abraham et kaufmann
et la conclusion majeure de de Broglie sur la structure dynamique
probable du photon.

> comment pourrait-on reconstruire quelque chose ?

"On", je ne sais pas. La génération montante sûrement.

> Avez-vous la moindre piste à proposer pour cela ?

Bien sûr. J'y ai référé ad noseam ici depuis des mois.

> S'il suffit de s'y mettre, pourquoi ne vous y êtes-vous pas mis
> vous-même ?

Je m'y suis effectivement mis il y a déjà 40 ans.

> Et si vos arguments sont valables, comment se fait-il qu'aucun
> jeune ne vous ait suivi

Ben, il faut laisser le temps à certains d'entre eux d'en prendre
connaissance. Ça fait seulement 2 ans que je diffuse dans les
institutions d'Amérique du Nord.

Soyez patient.

De plus, il n'est pas question de "me suivre". Seulement pour eux
de prendre conscience de l'ensemble des constats expérimentaux
corréllés accumulés à date, et à partir de cette base, finir
le travail de vérification qui s'impose.

> et ne soit effectivement parvenu à des explications capables
> d'égaler en puissance explicative/prédictive la formulation

> standard de la MQ.

Curieusement, je ne vois aucune puissance explicative ni prédictive
à la MQ. Un simple outil statistique pour mesurer les niveaux
d'énergie des orbitales des atomes.

> avec son équation de Schrodinger qui n'est pas si compliquée que
> cela) ?

Pas si compliquée en effet, mais incapable de prendre en compte
les interactions électromagnétiques d'une particule en mouvement
avec les particules environnantes puisqu'elle ne tient même pas
compte de l'existence des particules.

> Et les quelques grands physiciens que vous citez (Einstein,...)
> qui étaient du même avis que vous à ce que vous dites,

Je n'ai jamais dit qu'Einstein était du même avis que moi. J'ai
dit qu'il était causaliste comme moi.

> comment n'ont-ils pas été capables de proposer la moindre
> explication alternative de ces phénomènes sans s'appuyer sur
> les fonctions d'onde, qui rende compte d'au moins autant de
> choses que l'a fait l'école de Copenhague ?

Au départ, Einstein est décédé une bonne dizaine d'années avant
que les quarks up et down ne soient découverts dans les nucléons.

Il lui était donc impossible de les prendre en compte.



> >Mon avis est que, dans le cas EPR, ses résultats seraient faux,
> >pour la raison citée juste avant. Mais, encore, je pense que
> >nous ne le saurons de manière définitive que lorsque toute
> >la mécanique fondamentale des particules stables aura été
> >maîtrisée.
>
> Enfin une assertion précise.
>
> J'ai assisté un jour à Grenoble à un séminaire sur l'expérience EPR.
> Il disait en gros (je ne sais plus le détail) que les conditions de
> validité de ces expériences n'étaient au départ pas remplies ce qui
> permettait des explications classiques.

Vous semblez toujours opposer la MQ à la mécanique classique.

Je n'au aucun argument à proposer sur ce terrain, puisque en ce qui
me concerne, les deux approches sont incomplètes.

> Mais petit à petit en affinant
> les expériences, telle ou telle "explication" classique (ad hoc)
> tombait, en sorte que finalement on n'avait pas complètement rempli
> tous les critères à la fois mais on n'en était plus très loin, et il
> n'y avait donc plus de bonne raison physique pour que la réalité
> complote ainsi à opérer des moyens tirées par les cheveux pour
> produire les résultats que la MQ donne naturellement.
> Qu'on pourra sans doute à l'avenir continuer à affiner l'expérience
> pour remplir tous les critères mais qu'il est raisonnable de penser
> que les résultats de la MQ seront toujours valables car concrètement
> on ne voit pas au nom de quel mécanisme naturel ils deviendraient
> subitement faux.

Encore une fois, votre argumentation est fondée sur "soit la
mécanique classique donne la réponse, soit la MQ donne la réponse".

Moi, je dis que nous n'en savons pas assez pour connaître la
réponse. Ni la MC, ni la MQ ne peuvent nous la fournir.

La MC a été conçue avant que la struture interne des atomes ne
soit comprise.

La MQ a été conçu avant que la structure interne des nucléons
ne soit comprise.

Les deux sont archaiques par rapport aux constats expérimentaux
accumulés à ce jour.

> De toute façon qu'on soit d'accord ou pas, il est clair qu'il
> devrait être possible à l'avenir de réaliser cette expérience
> qui tranchera le débat.

Personellement, j'ai les mêmes doutes que Selleri.

> Même si personne n'y comprend rien.
>
> Que diriez-vous, plutôt que de passer votre temps à traiter tout
> le monde de noms d'oiseaux,

Est-ce que je vous ai traité de noms d'oiseaux?

Si la réponse est non, pourquoi, d'après vous?

> d'attendre plutôt tranquillement que cela arrive,

Je pense que trop de temps a déjà été perdu. Je n'attends plus.

> voire de participer activement à la la mise au point de cette
> expérience d'enjeu si capital,

Je ne vous suis pas, là. Pourquoi cette expérience serait-elle
si capitale?

> puisqu'elle permettra enfin de conclure qui a raison ?

J'avoue que je ne vous suis pas. Quelle utilité pourrait avoir
une expérience à mes yeux dépassée qui cherche à démontrer
laquelle de deux approche incomplètes serait complète?

Nous savons que les deux sont incomplètes.

En ce qui me concerne, déterminer qui a raison ou tort n'a aucune
importance. Ce qu'il faut déterminer, est ce qui est vrai,
peu importe qui a raison ou tort.



> >Pas à 4 dimensions, pas d'hypersurfaces. Oui, visualisation du
> >mouvement en continue.
>
> M'enfin, c'est mathématiquement la même chose !

Absolument pas. Pas plus que l'image mentale non-verbale d'une
chaise n'a a voir avec le mot "chaise", et pas plus que la
véritable chaise physique n'a a voir avec l'image mentale
non-verbale que les perceptions de nos sens nous permette de
construire dans notre cerveau.

La réalité, c'est la chaise.

L'image mentale qu'on réussit à s'en construire, et qui ne peut
pas être plus précise que ce que nous nous laissons le temps de
percevoir, c'est la visualisation.

Le mot "chaise", qui est la description qu'on fait de l'image
mentale qu'on a réussi à construire, c'est les mathématiques.

Ces mathématiques ne peuvent décrire que ce que nous avons
réussi à mettre dans l'image mentale.

Si l'image mentale est incomplète, la description sera
incomplète.

En tout temps, ce qu'il faut vraiment comprendre, c'est la
véritable chaise qui existe physiquement.



> Une visualisation en continu est un espace d'évènements où à
> chaque évènement on associe l'instant où on le voit (ou l'imagine).

Oui. Et dans lequel on peut le faire évoluer relativement à
d'autres événements.

> Ou si on préfère, c'est le fait d'associer à chaque instant
> (l'ensemble des instants étant une ligne continue) une image en 3
> dimensions:

ou plus.

> même si le mouvement est continu rien ne vous empêche de faire une
> photo quand vous voulez.

Tout à fait.

> Et cette union disjointe d'espaces à 3 dimensions le long d'un
> temps continu est bien mathématiquement un espace à 4 dimensions.

Vous présumez qu'il s'agit d'un espace à 3 dimensions.

> >oui quantitatif
>
> L'ensemble N des nombres entiers et l'ensemble R des nombre réels
> sont des objets mathématiques. Quant aux quantités physiques (masses,
> longueurs...), ce sont des éléments de droites vectorielles sur le
> corps des nombres réels. Tout cela très mathématique (même si on n'a
> pas l'habitude de présenter des calculs dimensionnés en cours de
> mathématiques).

Oui, "quantitatif" porte cette signification pour vous, mais pour
moi, elle porte la signification suivante : quantique. Rien à voir
avec les mathématiques.



> >quantique en fait
>
> Qu'est-ce à dire ???

C'est à dire qu'au niveau fondamental tout ne peut être que
quantités d'énergies discrètes. Nous savons depuis Wien, Planck,
Einstein, Compton, Raman que les ondes de Maxwell n'existent pas.



> >Je n'associe rien à chaque point de l'espace. Pour moi n'existe
> >que des événements électromagnétiques discrets qui suivent en tout
> >temps leurs trajectoires de moindre action, qui dépend de leurs
> >interactions avec toutes les autres particules en mouvement.
>
> Ce sont donc des courbes dans l'espace à quatre dimensions.

Vous présumez que les trajectoires sont courbes ou droites. Elles
ne peuvent que suivre leurs trajet de moindre action, que ne
peuvent se résoudre que par des courbes complexes.

> Les courbes sont des ensembles continus de points...

Mathématiquement oui, mais en pratique il n'y a pas de points,
il ne peut y avoir que progression continu des événements
électromagnétiques discrets dans l'espace et le temps.

> >> C'est donc un objet mathématique,
> >
> >Absolument pas. À aucun niveau.
>
> Vous n'avez donc aucune notion sur le sens du mot "mathématique".

J'en ai une parfaite compréhension. Voir plus haut la métaphore
au sujet de la chaise.



> >Pourquoi pensez-vous que le paradoxe EPR et les inégalités de
> >Bell sont encore sujet à controverse après tant de décennies?
>
> [Attention: ce que je vais répondre ici n'est que mon opinion
> personnelle qui ne prétend pas refléter celle de la majorité des
> physiciens]
> Parce que c'est un résultat qui heurte la philosophie réaliste de
> beaucoup de gens qui refusent d'admettre que la réalité physique
> (au sens de ce qui est du domaine de la science physique,
> c'est-à-dire ce qui est mathématisable) n'est pas indépendante.

Ce résultat ne heuterait pas ma philosophie. S'il était positif,
cela signifierait simplement qu'il correspond à la réalité dont
je devrai tenir compte. S'il était négatif, cela signifierait
simplement qu'il correspond à la réalité dont je devrai tenir
compte.

S'il est invérifiable, ce que l'analyse de Selleri me démontre
à ma satisfaction, je n'ai pas à en tenir compte, ni à passer
de temps à tourner en rond autour.

> Parce qu'il y a bien d'autres gens qui rechignent à accepter ce
> verdict de la réalité physique elle-même qui a montré son
> incomplétude à travers la manifestation des relations mathématiques
> qu'elle suit (celles de la MQ) qui font elles-même la démonstration
> (à travers leurs conséquences logiques) de leur propre incomplétude
> et plus précisément de leur incomplétabilité (pire qu'avec le
> théorème de Godel car là on peut toujours avancer en ajoutant des
> informations qui manquent et il en manquera d'autres; ici par contre
> c'est un point final sur la question de l'incertitude - ce qui
> n'empêche pas plein d'autres découvertes dans d'autres directions
> bien sûr).

Je suis en désaccord total sur le point que la MQ est aussi
fondamentale que vous semblez le penser. À mes yeux, elle n'est
qu'une méthode mathématique comme le calcul différentiel et
intégral.


> La différence, c'est que beaucoup de gens admettent les faits en
> pratique sans s'occuper de leurs implications philosophiques, que
> cela puisse ou non les heurter lorsqu'ils y réfléchissent.
> D'autres (dont je fais partie comme beaucoup de croyants) trouvent
> ces faits naturels parce qu'il est naturel que la réalité physique
> n'est qu'une partie de la réalité et n'est pas indépendante.

Dans ma vision des choses, il ne peut exister qu'une seule
réalité physique objective (la vrai chaise de la métaphore ci-haut).

C'est à nous de faire en sorte de construire correctement l'image
non-verbale que nous décrivons ensuite avec nos mathématiques.

C'est pour aider à ce que cette construction reprenne que je
diffuse les recherches que je vous ai mentionées.

> >> La question que je vous pose, c'est : quelles hypothèses vous
> >> sembleraient envisageables ?
> >
> >Je n'en sais rien. Nous verrons dans 10 ans, 20 ans, lorsque
> >la compréhension de la matière sera plus profonde.
>
> à comparer à:

à ce que la philosophie de l'école de Copenhague autorise.



> > Mon avis est que, dans le cas EPR, ses résultats seraient faux,
> >pour la raison citée juste avant. Mais, encore, je pense que
> >nous ne le saurons de manière définitive que lorsque toute
> >la mécanique fondamentale des particules stables aura été
> >maîtrisée.
>
> Voilà une belle incertitude quantique sur votre position.
> A quoi peut rimer la virulence de vos critiques contre le reste
> du monde si vous ne savez même pas où vous en êtes dans votre
> position ?

Mes critiques ne sont pas virulentes lorsqu'on ne m'attaque pas.

Je sais exactement où j'en suis dans ma position.

> Entre non et peut-être il faut choisir.

Je tentais seulement de répondr à vos questions précises.

Ma position est que lorsque toute l'information expérimentale
accumulée à date aura été réévaluée et intégrée dans un nouvel
ensemble cohérent, nous aurons des réponses à des questions
qui à ce jour sont demeurées à controverse.

> Si vous êtes en train de réfléchir, allez mener vos réflexions
> dans votre coin

Vous vous opposez donc à ce que je donne ici des références
livresques vérifiables à des jeunes qui posent des questions?

> et revenez nous voir quand vous serez au clair sur la limite de
> votre propre savoir.

Je suis tout à fait au clair sur la limite de mon propre savoir,
et je suis déjà ici.

> Si vous ne savez même pas ce que vous savez, comment pourriez-vous
> juger de ce que les autres ont le droit de savoir ?

Les "autres", ont accès aux mêmes informations que moi. Je ne suis
pas responsable de ce qu'ils acceptent de considérer et de ce
qu'ils rejettent. Pas mon problème.



> Jusqu'à quand condamnerez-vous le monde entier de folie au nom de
> l'affirmation tonitruante du fait que vous ne savez personnellement
> rien à rien et surtout pas quelles sont les limites de ce que vous
> savez, tout en démontrant que vous ignorez aussi les limites de ce
> que savent les autres ?

Je ne condamne pas le monde entier de folie. Je diffuse des
références à des recherches qui ne sont citées dans aucun
ouvrage de référence récent, et qui sont en contradiction
flagrante avec la philosophie de l'école de pensée de
Copenhague.


> Décidez-vous, oui ou non, de vous risquer à affirmer que les
> résultats de la MQ sont faux (au sens de faux = parfois faux,
> par opposition à vrai = toujours vrai ;

Les résultats de la MQ sont toujours vrais pour le calcul des


niveaux d'énergie des orbitales dans les atomes.

> et au sens expérimental puisque c'est un énoncé d'expérience

> précise) et qu'on s'en apercevra quand on fera une expérience
> plus fine de EPR ?

Pour les "prédictions" de la MQ dans le cas de du paradoxe EPR,
je suis du même avis que Selleri.

> C'est tout ce que je voulais savoir. Rendez-vous donc le jour où
> cette expérience sera réalisée.

En ce qui me concerne, qu'elle ait lieu ou non, qu'elle soit
concluante ou non, elle ne changera rien à la réalité.

> >Pour moi, les observateurs ne font pas partie de l'équation.
> >Lorsqu'ils "observent", l'événement est déjà terminé, parce que
> >ce qu'ils observent, ce sont les signaux des détecteurs qui leur
> >parviennent après que l'événement est terminé.
>
> Remplacez "observateur" par "détecteur". Ca va mieux comme ça ?

Détecteur, ça va mieux, mais l'expérience réfère à des observateurs.



> >lorsque nous en saurons plus sur la mécanique fondamentale
>
> A partir de quelles informations ?

À partir de la mise en corrélation en seul tout cohérent de
l'ensemble des constats expérimentaux faits au sujet des
particules élémentaire fondamentales.

> Développements purement théoriques ou bien nouvelles expériences
> contredisant les théories ?

Ni l'un ni l'autre. Construction de l'édifice final à partir
des briques réelles qui sont maintenant à notre disposition.

> Par exemple, si sur l'expérience EPR la réalité s'accordait à
> l'inégalité de Bell contre les résultats de la MQ, cela constituera
> un résultat précis à examiner.

Voir plus haut. J'ai la même opinion que Selleri à ce sujet.

Invérifiable.

> En attendant, la théorie a toujours été vérifiée par l'expérience.

Pas selon Selleri.

> Si la vraie théorie donne des résultats contraires à l'ancienne
> sur les domaines expérimentaux accessibles, on devrait constater
> effectivement un désaccord entre les théories actuelles et
> l'expérience.

Toute théorie qui donne des résultats contraires à la réalité
expérimentale confirmée de manière non équivoque est fausse
ou incomplète par définition.

> Si le désaccord porte sur des domaines inexplorés, c'est un
> développement des expériences qui permettra de découvrir ce
> désaccord.

Aussi s'il porte sur un défaut d'intégration de données
expérimentales disponibles mais non intégrées. Ce qui est
notre problème actuellement.

> S'il n'y a pas de désaccord observable, la vraie théorie est alors
> toujours d'accord avec celle actuelle sur ce qui est accssible, en
> sorte qu'elle ne serait qu'une reformulation des théories actuelles,
> quelque chose de mathématiquement équivalent qu'on pourrait découvrir
> par des moyens purement théoriques par développement à partir des
> formules établies.
>
> Laquelle de ces options prenez-vous donc ?

Aucune de celles que vous proposez. Je prend celle que j'ai déjà
mentionnée:

Mise en corrélation en seul tout cohérent de l'ensemble des constats
expérimentaux faits au sujet des particules élémentaire fondamentales.



> >Je pense que autant les signaux de B que ceux de A lui parviennent
> >après que l'événement est terminé. donc, ce qu'il conclue ne peut
> >en aucun cas influencer ce qui est déjà dans le passé. Rien ne
> >peut changer le passé.
>
> Voilà qui est une affirmation claire. N'implique-t-elle pas
> logiquement l'inégalité de Bell, et donc la fausseté de la loi de
> probabilité que donne la MQ sur ce sujet ?

Pas du tout. Elle implique simplement à mon avis que le problème
semble mal posé, qu'on tente d'utiliser la MQ dans un contexte
où elle est inadéquoite.


> >> Mon opinion personnelle est que l'ensemble des pistes de recherche
> >> admissibles est vide.
> >
> >Par le biais de la MQ et de la fonction d'onde. C'est mon opinion
> >aussi.
>
> Encore une fois je pense que vous n'avez pas saisi ce que
> j'entendais par "piste de recherche".
> Vous vous référez à la manière de penser et de comprendre.

Du tout. Je réfère simplement à l'intégration de toutes les
données expérimentales accumulées en un seul tout cohérent.

> Ce qui fait planer la discussion dans de vagues philosophies
> dépourvues d'affirmations concrètes à discuter, et où on pourra
> toujours s'étriper indéfiniment à ne pas comprendre la pensée de
> l'autre.

Absolument pas. Il suffit de s'entendre sur des définitions
de termes communes.



> Mais ce que j'espérais savoir de vous, c'est des affirmations
> précises sur les résultats expérimentaux: contrediront-ils,
> oui ou non, les lois de probabilité issues de la MQ ?

Pour le paradoxe EPR, je pense, tout comme Selleri, que nous ne le
saurons jamais étant donné l'imprécision des appareils de mesure,
et parce que la fonction d'onde est incapable de prendre en compte
les interactions de la particule en mouvement avec les autres
particules environnantes.

Pour le calcul des niveaux d'énergie dans les atomes la MQ est
sans équivoque correcte.

> >Mon opinion est que ces pistes de recherche vont se faire jour
> >d'ici quelques années, lorsque nous comprendrons plus profondément
> >la nature des particules fondamentales et leurs interactions
> >réelles.
>
> Ben alors dépêchez-vous d'explorer ces pistes de recherche si
> prometteuses et revenez nous voir quand vous les aurez trouvées.

Mais je suis là, et je diffuse les pistes de recherche tous azimuts
depuis 2 ans déjà (pas ici, soyez rassuré).

Sylvain

non lue,
10 oct. 2002, 21:21:4310/10/2002
à
Petite question : de quand date l'analyse de Selleri que vous évoquez
?
Autre question: que pensez-vous des travaux sur la théorie des
ordinateurs quantiques ?
Si l'analyse de Selleri avait si bien montré l'impossibilité de
corréler les particules, nul ne se serait investi dans de telles
réflexions, non ?
Je soupçonne que son analyse s'applique aux conditions expérimentales
qui étaient disponibles à une certaine époque, et que depuis le
progrès technologique a permis de rendre possibles certaines choses
qui ne l'étaient pas. Je me trompe ?

>C'est pourquoi je pense qu'il est temps qu'une nouvelle synthèse
>émerge, fondée sur ce nouvel ensemble plus complet.

Les théories en vigueur que l'expérience a suggéré forment un ensemble
cohérent, bien simple en comparaison du très vaste éventail des
expériences qui les ont vérifiées et à leur degré de précision,
ensemble qui n'a donc pas besoin d'être synthétisé dans le fond (à
moins bien sûr de rentrer dans une théorie des cordes à 10 dimensions
ou autres sans intérêt pratique), seulement parfois dans la forme (ce
dont je m'occupe...)

>Mais bien sûr, comme quiconque comprend que les orbitales ne peuvent
>correspondre qu'aux longueurs d'ondes intègres permises pour les
>électrons dans les atomes.

Merci de donner la référence d'une présentation développée de cela,
car ces quelques mots ne suffisent pas à m'en faire une idée.

>De quelle oeuvre parlez-vous? Et qui parle de détruire? Je parle
>au contraire de synthétiser en un nouvel ensemble plus complet
>toutes les connaissances accumulées expérimentalement au niveau
>fondamental, y compris surtout la découverte de Abraham et kaufmann
>et la conclusion majeure de de Broglie sur la structure dynamique
>probable du photon.

Références ?
"structure du photon" hum hum. Le photon est actuellement connue comme
une particule élémentaire, et la théorie en vigueur qui la décrit est
l'électrodynamique quantique (ou encore pour aller plus loin, la
théorie de l'interaction électrofaible...)
Cette théorie donne de grandes précisions sur la mécanique des
particules électromagnétiques pour reprendre vos termes, précisions
confirmées par l'expérience à ce qu'il me semble.

Ma question est: Connaissez-vous bien cette théorie ?


>> Avez-vous la moindre piste à proposer pour cela ?
>
>Bien sûr. J'y ai référé ad noseam ici depuis des mois.

Ah bon ??? La seule chose que je vous ai vu référer ad noseam est la
définition de "minus copenhagensis".
Et l'affirmation qu'il y aurait une réalité sous-jacente à explorer,
affirmation beaucoup trop vague pour avoir la moindre utilité
effective pour indiquer la voie d'une telle recherche justement.

>De plus, il n'est pas question de "me suivre". Seulement pour eux
>de prendre conscience de l'ensemble des constats expérimentaux
>corréllés accumulés à date, et à partir de cette base, finir
>le travail de vérification qui s'impose.

Quels constats expérimentaux ? Merci de donner des références
précises.

>Curieusement, je ne vois aucune puissance explicative ni prédictive
>à la MQ. Un simple outil statistique pour mesurer les niveaux
>d'énergie des orbitales des atomes.

Lire cela donne l'impression que vous ne connaissez pas grand-chose à
la MQ, comme si vous en aviez lu un petit bouquin introductif quelque
part et que vous aviez supposé qu'il n'y avait rien d'autre après.

>> validité de ces expériences n'étaient au départ pas remplies ce qui
>> permettait des explications classiques.
>
>Vous semblez toujours opposer la MQ à la mécanique classique.

En remplaçant "classique" par "causaliste", ça va mieux ?

>Je ne vous suis pas, là. Pourquoi cette expérience serait-elle
>si capitale?

Parce qu'en elle la MQ entre en claire contradiction dans ses effets
avec ce que permettrait le causalisme, et permettrait donc de
départager.

(pour la chaise on ne s'entendra pas mais je suis las d'essayer
d'argumenter)

>Vous présumez qu'il s'agit d'un espace à 3 dimensions.

J'essaie de vous faire parler, et j'avais l'impression que telle
pouvait être votre pensée. Si ce n'est pas le cas, alors soit.

>C'est à dire qu'au niveau fondamental tout ne peut être que
>quantités d'énergies discrètes. Nous savons depuis Wien, Planck,
>Einstein, Compton, Raman que les ondes de Maxwell n'existent pas.

Donc que tout est finitiste ? Remarquez que si rien n'est étendu dans
l'espace, pas même les particules interactives, et que ces particules
ont une taille nulle sinon il faudrait expliquer la solidité interne
de leur épaisseur, alors les particules n'ont qu'une probabilité nulle
de se rencontrer pour réagir.
Les ondes de Maxwell, vous voulez dire le champ électromagnétique,
fonction qui à tout point de l'espace-temps associe les vecteurs E et
B (étant donné un référentiel galiléen bien sûr) ?
Je suis d'accord qu'elles ne sont qu'une ombre de la réalité telle que
la donne l'électrodynamique quantique, il n'empêche qu'il y a un
domaine d'approximation où ça reste valable.

>Vous présumez que les trajectoires sont courbes ou droites. Elles
>ne peuvent que suivre leurs trajet de moindre action, que ne
>peuvent se résoudre que par des courbes complexes.

Que peut être une courbe complexe qui ne soit pas une courbe ?

>Les résultats de la MQ sont toujours vrais pour le calcul des
>niveaux d'énergie des orbitales dans les atomes.

Quelle culture avez-vous concernant les domaines d'application de la
mécanique quantique ?
Connaissez-vous par exemple de quelle manière elle explique la
supraconductivité, la superfluidité, les lasers, et sûrement plein
d'autres choses dont je n'ai pas la liste...
Connaissez-vous le Modèle Standard ? Notamment les théories de jauge
qui sont ce par quoi les physiciens comprennent effectivement les
interactions nucléaires au-delà de la "petite histoire" des quarks
facile à raconter au public ?

>> En attendant, la théorie a toujours été vérifiée par l'expérience.
>
>Pas selon Selleri.

"vérifiée", je voulais dire pas contredite. Vous voulez dire qu'elle a
été contredite ?

>En ce qui me concerne, qu'elle ait lieu ou non, qu'elle soit
>concluante ou non, elle ne changera rien à la réalité.

Tiens c'est nouveau, je n'avais pas encore entendu dire que la réalité
pouvait être indépendante des conclusions issues des résultats
expérimentaux. On en apprend tous les jours.

>Mais je suis là, et je diffuse les pistes de recherche tous azimuts
>depuis 2 ans déjà (pas ici, soyez rassuré).

1) Diffuser des pistes de recherche n'a pas de sens si elles sont trop
vagues, seuls les résultats effectifs ont avantage à être diffusés.
Sinon ce n'est pas de la science mais du bavardage inutile. Mais je ne
sais pas de quoi il s'agit donc tout tient dans le 2):
2) Pour être recevables, ces informations (pistes de recherches +
résultats accumulés) dont vous affirmez ici l'existence devraient au
moins être gratuitement disponibles sur Internet, et je vous prie donc
d'en donner les URL.
Sinon (si ce sont des livres que vous prétendez vendre) je vous
expliquerai ici pourquoi je considèrerais cela comme de la pure
arnaque.

Merci.

srp

non lue,
11 oct. 2002, 08:40:3311/10/2002
à
Sylvain a écrit :

>
> Petite question : de quand date l'analyse de Selleri que vous évoquez
> ?
> Autre question: que pensez-vous des travaux sur la théorie des
> ordinateurs quantiques ?

À mon avis, ils sont sans issue.

> Si l'analyse de Selleri avait si bien montré l'impossibilité de
> corréler les particules, nul ne se serait investi dans de telles
> réflexions, non ?

Il n'a pas montré l'impossibilité de corréler des particules.
Il a analysé l'ensemble et montré que la précision des instruments
utilisables ne permettait pas de le vérifier de manière concluante.

> Je soupçonne que son analyse s'applique aux conditions expérimentales
> qui étaient disponibles à une certaine époque, et que depuis le
> progrès technologique a permis de rendre possibles certaines choses
> qui ne l'étaient pas. Je me trompe ?

Son ouvrage a été publié en 1986. Donc assez récent. Il est toujours
des nôtres, et n'a rien modifié pour les rééditions, donc, rien
n'a changé à ce jour.

Il est toujours disponible chez Flammarion. Je le recommande
à tous les jeunes.



> >C'est pourquoi je pense qu'il est temps qu'une nouvelle synthèse
> >émerge, fondée sur ce nouvel ensemble plus complet.
>
> Les théories en vigueur que l'expérience a suggéré forment un
> ensemble cohérent,

Absolument pas. La MQ et la GR sont totalement irréconciliables
depuis le début. Même après 30 ans, il est toujours impossible
de résoudre les équations de la chromodynamique quantique de
manière précise pour même un seul nucléon.

Belle cohérence!

> bien simple en comparaison du très vaste éventail des expériences
> qui les ont vérifiées et à leur degré de précision, ensemble qui
> n'a donc pas besoin d'être synthétisé dans le fond

Total désaccord.

> (à moins bien sûr de rentrer dans une théorie des cordes à 10
> dimensions ou autres sans intérêt pratique), seulement parfois
> dans la forme (ce dont je m'occupe...)

Cul de sac, à mon avis.

> >Mais bien sûr, comme quiconque comprend que les orbitales ne
> >peuvent correspondre qu'aux longueurs d'ondes intègres permises
> >pour les électrons dans les atomes.
>
> Merci de donner la référence d'une présentation développée de cela,
> car ces quelques mots ne suffisent pas à m'en faire une idée.

Voir les travaux de de Broglie et de Bohr juste avant que
Shrödinger ne produise son équation et Heisenberg sa méthode
statistique.



> >De quelle oeuvre parlez-vous? Et qui parle de détruire? Je parle
> >au contraire de synthétiser en un nouvel ensemble plus complet
> >toutes les connaissances accumulées expérimentalement au niveau
> >fondamental, y compris surtout la découverte de Abraham et kaufmann
> >et la conclusion majeure de de Broglie sur la structure dynamique
> >probable du photon.
>
> Références ?

Poincaré en traite sommairement des fondements dans "La science et
l'hypothèse" (page 247 dans l'édition de Flammarion) et dans
"La valeur de la science" (page 137 dans l'édition de Flammarion).

Un contributeur a déjà donné ici un lien vers le papier original
de Abraham qui en traite "Dynamik des Electrons"

http://134.76.163.65/servlet/digbib?template=view.html&id=52993&startpage=26&endpage=47&image-path=http://134.76.176.141/cgi-bin/letgifsfly.cgi&image-subpath=/2026&image-subpath=2026&pagenumber=26&imageset-id=2026

> "structure du photon" hum hum. Le photon est actuellement connue
> comme une particule élémentaire, et la théorie en vigueur qui la
> décrit est l'électrodynamique quantique (ou encore pour aller plus
> loin, la théorie de l'interaction électrofaible...)

L'électrodynamique quantique ne décrit pas le photon. Elle utilise
la méthode lagrangienne pour substituer la notion métaphorique de
photons virtuels à l'interaction coulombienne sous-jacente entre
les particules.

Rien à voir avec les vrai photons. Même commentaire pour
l'interaction électrofaible. Photons virtuels. Rien à voir
avec les vrai photons.

> Cette théorie donne de grandes précisions sur la mécanique des
> particules électromagnétiques pour reprendre vos termes,
> précisions confirmées par l'expérience à ce qu'il me semble.

Mathématiquement parlant, les calculs sont précis. J'en
j'en conviens.

> Ma question est: Connaissez-vous bien cette théorie ?

Je les comprend. Solutions mathématiques adéquates.



> >> Avez-vous la moindre piste à proposer pour cela ?
> >
> >Bien sûr. J'y ai référé ad noseam ici depuis des mois.
>
> Ah bon ??? La seule chose que je vous ai vu référer ad noseam est
> la définition de "minus copenhagensis".

Votre attention semble n'avoir été attirée que par mes bloquages
systématiques à des irrationels qui s'imaginaient pouvoir me faire
taire ici.

> Et l'affirmation qu'il y aurait une réalité sous-jacente à
> explorer, affirmation beaucoup trop vague pour avoir la moindre
> utilité effective pour indiquer la voie d'une telle recherche
> justement.

La réalité physique nous environne de toute part. Chacun fait
son choix à savoir s'il décide de croire qu'elle existe ou pas.

Les causalistes en ont de tout temps pris conscience et ont de
tout temps tenté d'en augmenter notre niveau de compréhension.



> >De plus, il n'est pas question de "me suivre". Seulement pour eux
> >de prendre conscience de l'ensemble des constats expérimentaux
> >corréllés accumulés à date, et à partir de cette base, finir
> >le travail de vérification qui s'impose.
>
> Quels constats expérimentaux ? Merci de donner des références
> précises.

Intégration de la découverte de Abraham et Kaufmann que les
quantités de mouvement sont totalement insensibles aux
interactions transversales. Voir références plus haut.

Intégration de la conclusions de de Broglie au sujet de la
structure dynamique interne probable du photon.

"La physique nouvelle et les quanta", Flammarion, page 277.

Intégration en un seul tout cohérent des interactions entre
les photons (réels), les électrons, les positons, les quarks
up, les quarks down, les neutrinos.



> >Curieusement, je ne vois aucune puissance explicative ni prédictive
> >à la MQ. Un simple outil statistique pour mesurer les niveaux
> >d'énergie des orbitales des atomes.
>
> Lire cela donne l'impression que vous ne connaissez pas grand-chose
> à la MQ,

Vous avez droit à votre opinion.

> comme si vous en aviez lu un petit bouquin introductif quelque
> part et que vous aviez supposé qu'il n'y avait rien d'autre après.

Vous avez droit à votre opinion.



> >> validité de ces expériences n'étaient au départ pas remplies ce
> >> qui permettait des explications classiques.
> >
> >Vous semblez toujours opposer la MQ à la mécanique classique.
>
> En remplaçant "classique" par "causaliste", ça va mieux ?

Les débats philosophiques ne m'intéressent que modérément.

Je suis causaliste. Je suis intéressé seulement à l'intégration
des constats expérimentaux en un seul tout cohérent.


> >Je ne vous suis pas, là. Pourquoi cette expérience serait-elle
> >si capitale?
>
> Parce qu'en elle la MQ entre en claire contradiction dans ses
> effets avec ce que permettrait le causalisme, et permettrait
> donc de départager.

À vrai dire, la MQ n'est en conflit avec rien du tout. C'est
l'interprétation de Copenhague qui l'érige en solution finale,
ce qui est impossible puisqu'elle ne peut intégrer les
niveaux d'énergie des quarks up et down.



> (pour la chaise on ne s'entendra pas mais je suis las d'essayer
> d'argumenter)

Si on ne s'entend pas sur la métaphore de la chaise, il n'existe
aucun moyen pour que nous nous comprenions.



> >Vous présumez qu'il s'agit d'un espace à 3 dimensions.
>
> J'essaie de vous faire parler, et j'avais l'impression que telle
> pouvait être votre pensée. Si ce n'est pas le cas, alors soit.

Ce n'est pas le cas.

Quant à me "faire parler", comme vous dites, vous perdez votre
temps. La simple mention des travaux de de Broglie fait baver
de mépris tous les physiciens qui fréquentent ce forum depuis
4 mois, certains jusqu'à l'obsession maladive (ceux que je
bloque systématiquement avec mon petit message sur les minus
copenhagensis).

Un élève de Wheeler à Princeton a même eu le front de qualifier
ici de Broglie de zozo et d'affirmer qu'il avait probablement
eu son Nobel pour des raisons politiques.

Alors, j'imagine parfaitement l'hystérie collective qui s'emparera
du lot si je fait mine de commencer à expliquer comment ses
conclusions peuvent permettre d'intégrer l'ensemble des constats
expérimentaux.

Très peu pour moi.



> >C'est à dire qu'au niveau fondamental tout ne peut être que
> >quantités d'énergies discrètes. Nous savons depuis Wien, Planck,
> >Einstein, Compton, Raman que les ondes de Maxwell n'existent pas.
>
> Donc que tout est finitiste ?

Comprend pas.

> Remarquez que si rien n'est étendu dans l'espace,

Jamais dit cela.

> pas même les particules interactives,

Nous savons que toutes les particules sont faites de quantités
d'énergie précises. Non?

> et que ces particules ont une taille nulle sinon il faudrait
> expliquer la solidité interne de leur épaisseur,

Aucune particule ne peut avoir une "taille nulle".

> alors les particules n'ont qu'une probabilité nulle de se rencontrer
> pour réagir.

Les abstractions mathématiques ne sont pas la réalité. Le mot
"chaise" n'est pas la chaise.

> Les ondes de Maxwell, vous voulez dire le champ électromagnétique,
> fonction qui à tout point de l'espace-temps associe les vecteurs E
> et B (étant donné un référentiel galiléen bien sûr) ?
> Je suis d'accord qu'elles ne sont qu'une ombre de la réalité telle
> que la donne l'électrodynamique quantique, il n'empêche qu'il y a
> un domaine d'approximation où ça reste valable.

Tout à fait valable au niveau macro. Mais au niveau micro, tout ne
peut être que quantique.

> >Vous présumez que les trajectoires sont courbes ou droites. Elles
> >ne peuvent que suivre leurs trajet de moindre action, que ne
> >peuvent se résoudre que par des courbes complexes.
>
> Que peut être une courbe complexe qui ne soit pas une courbe ?

Touché.



> >Les résultats de la MQ sont toujours vrais pour le calcul des
> >niveaux d'énergie des orbitales dans les atomes.
>
> Quelle culture avez-vous concernant les domaines d'application de
> la mécanique quantique ?
> Connaissez-vous par exemple de quelle manière elle explique la
> supraconductivité, la superfluidité, les lasers, et sûrement plein
> d'autres choses dont je n'ai pas la liste...

On a bien sûr tenté de tout lui faire expliqué. Les laser, par
exemple sont dûs à un constat d'Einstein qui date d'avant la MQ.

On a bien sûr tenté d'expliquer tous les constats expérimentaux
avec la MQ. Pourquoi en aurait-il été autrement?

> Connaissez-vous le Modèle Standard ? Notamment les théories de jauge
> qui sont ce par quoi les physiciens comprennent effectivement les
> interactions nucléaires au-delà de la "petite histoire" des quarks
> facile à raconter au public ?

Toujours pas d'intégration avec la séquence connue quantités mouvement,
photons, paires électron/positon.



> >> En attendant, la théorie a toujours été vérifiée par l'expérience.
> >
> >Pas selon Selleri.
>
> "vérifiée", je voulais dire pas contredite. Vous voulez dire qu'elle
> a été contredite ?

Je veux dire, comme Sellerie, que la sensibilité des équipements
utilisés n'a jamais permi de le vérifier expérimentalement de
manière concluante pour EPR, ce qui était, il me semble, le
suje de la discussion.

> >En ce qui me concerne, qu'elle ait lieu ou non, qu'elle soit
> >concluante ou non, elle ne changera rien à la réalité.
>
> Tiens c'est nouveau, je n'avais pas encore entendu dire que la
> réalité pouvait être indépendante des conclusions issues des
> résultats expérimentaux. On en apprend tous les jours.

Elle n'est pas indépendante des résultats expérimentaux vérifiables,
ce qui n'est toujours pas le cas pour EPR.



> >Mais je suis là, et je diffuse les pistes de recherche tous azimuts
> >depuis 2 ans déjà (pas ici, soyez rassuré).
>
> 1) Diffuser des pistes de recherche n'a pas de sens si elles sont
> trop vagues,

Elles sont parfaitement claire.

> seuls les résultats effectifs ont avantage à être diffusés.

Exact.

> Sinon ce n'est pas de la science mais du bavardage inutile.

Exact.

> Mais je ne sais pas de quoi il s'agit donc tout tient dans le 2):
> 2) Pour être recevables, ces informations (pistes de recherches +
> résultats accumulés) dont vous affirmez ici l'existence devraient
> au moins être gratuitement disponibles sur Internet,

Vous avez une étrange conception de la recevabilité.

> et je vous prie donc d'en donner les URL.

Pas de URL. J'ai retiré tout droit de regard à notre si
orgeuilleuse élite en physique fondamentale.

70 ans de perdus en futiles palabres m'on convaincu.

La génération montante se chargera de vous expliquer de quoi
il s'agit.

> Sinon (si ce sont des livres que vous prétendez vendre) je vous
> expliquerai ici pourquoi je considèrerais cela comme de la pure
> arnaque.

Est-ce la raison pour laquelle vous ne vous procurez pas le
magnifique ouvrage de Selleri, et que de toute évidence vous
n'avez pas lu celui (ceux, devrais-je dire) de de Broglie?
À les lire, j'ai trouvé qu'ils valaient chaque centime du faible
prix que j'ai du verser pour me les procurer. Je les conseille à
tous les jeunes.

Voyez-vous, ce que vous pensez est justement pourquoi la diffusion
auprès de la génération montante est si facile. Les jeunes n'ont
pas la prétention d'avoir tout vu, tout compris. Tout ce qui semble
avoir du sens les intéresse.

De plus, la diffusion que je fais est à titre gracieux. Toutes
les maigres ressources dont je dispose y sont consacrées.

ricky

non lue,
11 oct. 2002, 14:14:1811/10/2002
à
bonjour

> > Autre question: que pensez-vous des travaux sur la théorie des
> > ordinateurs quantiques ?
>
> À mon avis, ils sont sans issue.

pour sylvain

les premiers resultats positifs a partir d'ordinateurs quantique ont ete
obtenus recement
on a reussi a faire un equivalent d'une addition de vecteur...
cela correspond pour le moment a des capacites faibles, mais cela demontre
qu'on avance et que cette recherche est assez realiste

> Intégration de la conclusions de de Broglie au sujet de la
> structure dynamique interne probable du photon.
>
> "La physique nouvelle et les quanta", Flammarion, page 277.

on en profitera pour admirer dans ce livre toutes les precautions d'ecriture
montrant que pour ces physicien differncient bien les concepts qu'ils
utilisent d'une certitude objective

> À vrai dire, la MQ n'est en conflit avec rien du tout. C'est
> l'interprétation de Copenhague qui l'érige en solution finale,
> ce qui est impossible puisqu'elle ne peut intégrer les
> niveaux d'énergie des quarks up et down.

pour sylvain, l'interpretation de copenhague n'erige rien en solution
finale... elle interprete simplement une vision de la MQ ...
la mecanique Quantique ou "meca statistique" contient en son sein quel que
soit l'interpretation une contradiction possible avec la causalite telle
qu'on la connait jusqu'a present...

> Quant à me "faire parler", comme vous dites, vous perdez votre
> temps. La simple mention des travaux de de Broglie fait baver
> de mépris tous les physiciens qui fréquentent ce forum depuis
> 4 mois, certains jusqu'à l'obsession maladive (ceux que je
> bloque systématiquement avec mon petit message sur les minus
> copenhagensis).

au contraire

on admire de broglie, qui contrairement a certain haineux de ce forum, seuls
utilisateurs d'insulte divers, a toujours bien mis les precautions oratoirs
et de pensee dans ses ecrits... permettant de ne pas croire betement a une
realite objective demontree...

ce sont les deformations de certains qui font baver de mepris :-)

> Un élève de Wheeler à Princeton a même eu le front de qualifier
> ici de Broglie de zozo et d'affirmer qu'il avait probablement
> eu son Nobel pour des raisons politiques.

il faudra en profitr pour relire tout le message de la personne :-)

> Alors, j'imagine parfaitement l'hystérie collective qui s'emparera
> du lot si je fait mine de commencer à expliquer comment ses
> conclusions peuvent permettre d'intégrer l'ensemble des constats
> expérimentaux.

oui si c'est vous qui les deformez , en effet
mais il suffit, comme vous le conseillez , de lire son oeuvre pou voir
quelque choser de plus sain et de plus raisonne.

> Très peu pour moi.

on vous en remercie ;-)))

> Les abstractions mathématiques ne sont pas la réalité. Le mot
> "chaise" n'est pas la chaise.

tout comme le mot electron d'ailleurs ;-)

> On a bien sûr tenté de tout lui faire expliqué. Les laser, par
> exemple sont dûs à un constat d'Einstein qui date d'avant la MQ.

pour sylvain : a different niveaux...
deja, une theorie ne demontre pas la realite..
ensuite, einstein et la mq ont oborde le probleme differement ce qui permet
d'avoir divers theories qui recouvrent la notion de laser

> On a bien sûr tenté d'expliquer tous les constats expérimentaux
> avec la MQ. Pourquoi en aurait-il été autrement?

on n'a jamais "explique" quoi que ce soit avec une theorie
on tente juste de retrouver les observations

> > et je vous prie donc d'en donner les URL.
>
> Pas de URL. J'ai retiré tout droit de regard à notre si
> orgeuilleuse élite en physique fondamentale.

;-)))

> 70 ans de perdus en futiles palabres m'on convaincu.

c'et votre age sans doute ;-)

> La génération montante se chargera de vous expliquer de quoi
> il s'agit.

en effet, et pour le moemnt, elle est encore efficace et rationnelle..

> Est-ce la raison pour laquelle vous ne vous procurez pas le
> magnifique ouvrage de Selleri, et que de toute évidence vous
> n'avez pas lu celui (ceux, devrais-je dire) de de Broglie?
> À les lire, j'ai trouvé qu'ils valaient chaque centime du faible
> prix que j'ai du verser pour me les procurer. Je les conseille à
> tous les jeunes.

moi aussi
et de bien voir les termes choisis (surtotu de broglie, un maitre en la
matiere)

> Voyez-vous, ce que vous pensez est justement pourquoi la diffusion
> auprès de la génération montante est si facile. Les jeunes n'ont
> pas la prétention d'avoir tout vu, tout compris. Tout ce qui semble
> avoir du sens les intéresse.

comme "sembler voir du sens " est propre a chacun, en fait tout les
interesse

> De plus, la diffusion que je fais est à titre gracieux. Toutes
> les maigres ressources dont je dispose y sont consacrées.

lol comme dans votre site

@-


ricky

non lue,
11 oct. 2002, 14:30:1211/10/2002
à
bonjour

> Je soupçonne que son analyse s'applique aux conditions expérimentales
> qui étaient disponibles à une certaine époque, et que depuis le
> progrès technologique a permis de rendre possibles certaines choses
> qui ne l'étaient pas. Je me trompe ?

il n'y a pas que cela
selleri prone une vision de la physique, tout comme le faisaient l'ecole de
copenhague ou einstein ou les autres...
chaunes de ces visions permet d'eclairer certains cotes de la physique en en
fermant d'autres...

et d'ailleurs je ne peux m'empecher de citer deux lignes de selleri qu'un
certain interlocuteur a sans doute oublie de citer losqu'il parle de realite
objective :
"Il est impossible dans toute théorie future de faire abstraction de
l'observateur pour décrire le monde physique ", ce qi montre bien que meme
lui refuse de parler de realite "objective" et
"La définition d'une particule élémentaire aujourd'hui est liée à son
interaction. On ne parlera jamais d'une particule "en soi", mais en fonction
de l'action qu'elle crée dans telle ou telle condition"

"Le grand débat de la théorie quantique" de Franco Selleri

on voit bien que personne ne parle de "realite objective", meme pas les
personnes les plus admirees par srp ;-)

de meme, on voit aussi le refus de la notion de "mecanique" pour la MQ

> Références ?
> "structure du photon" hum hum.

en fait, selon certaines descriptions, on peut voir le photon comme une
composition de sous-structures...
seulement il ne s'agit que d'une decomposition mathematique permettant de
decrire plus precisement certains resultats d'experience sans toutefois
donner un sens physique a ces sous systemes

> Ah bon ??? La seule chose que je vous ai vu référer ad noseam est la
> définition de "minus copenhagensis".

:-)

> Et l'affirmation qu'il y aurait une réalité sous-jacente à explorer,
> affirmation beaucoup trop vague pour avoir la moindre utilité
> effective pour indiquer la voie d'une telle recherche justement.

tout a fait ;-)

> >Curieusement, je ne vois aucune puissance explicative ni prédictive
> >à la MQ. Un simple outil statistique pour mesurer les niveaux
> >d'énergie des orbitales des atomes.
>
> Lire cela donne l'impression que vous ne connaissez pas grand-chose à
> la MQ, comme si vous en aviez lu un petit bouquin introductif quelque
> part et que vous aviez supposé qu'il n'y avait rien d'autre après.

j'avoue que je suis en accord pour la partie "explication"... la MQ est
preductive mais n'explique pas au sens physique du terme ...

> >Vous semblez toujours opposer la MQ à la mécanique classique.
>
> En remplaçant "classique" par "causaliste", ça va mieux ?

et surtout le terme "mecanique" n'est pas adapte a cette theorie comme
l'admettent la majorite des physiciens ...

> Je suis d'accord qu'elles ne sont qu'une ombre de la réalité telle que
> la donne l'électrodynamique quantique, il n'empêche qu'il y a un
> domaine d'approximation où ça reste valable.

toujours se referer au domaine d'approximation (ou de validite) pour chaque
theorie :-)

> Tiens c'est nouveau, je n'avais pas encore entendu dire que la réalité
> pouvait être indépendante des conclusions issues des résultats
> expérimentaux. On en apprend tous les jours.

;-)

> 2) Pour être recevables, ces informations (pistes de recherches +
> résultats accumulés) dont vous affirmez ici l'existence devraient au
> moins être gratuitement disponibles sur Internet, et je vous prie donc
> d'en donner les URL.

on est beaucouip a attendre autre chose que "lisez en entier tel bouquin et
vous verrez" :-)
bonne chance pour avoir un reponse precise ;-)

@+
ricky


srp

non lue,
11 oct. 2002, 16:36:3611/10/2002
à
La raison de mon mépris pour tous les tenants de l'école de
Copenhague, qui est devenu total depuis l'avénement de l'Internet,
est le constat que l'ensemble des physiciens tolèrent, parce
que ça les arrange bien, que des étroits d'esprit fanatiques,
le plus souvent anonymes, se chargent à leur place de harasser
et discréditer tous les opposants jusqu'à ce qu'ils abandonnent
à l'usure.

Quelle déchéance pour l'orthodoxie en physique d'en être réduite
à se retrancher derrière de pareils minables! Il est urgent que
des physiciens responsables émergent de la génération montante,
et c'est pour en favoriser l'éclosion que je rediffuse certaines
découvertes qui sont systématiquement passées sous silence

dans les ouvrages de référence dits "modernes".

Pour les jeunes qui lisent ces lignes pour la première fois,

ricky et quelques autres sont de tristes individus qui me dénigrent
systématiquement sur fsp depuis des mois, ainsi que tout le travail

de vulgarisation fait par le Service de Recherche Pédagogique pour
l'éducation, parce que je refuse systématiquement de discuter avec
eux de physique, car je suis un causaliste, comme de Broglie,

Planck, Schrödinger, Abraham et bien d'autres, et qu'ils
appartiennent, comme la majorité des physiciens contemporains,
à l'école de pensée de Copenhague.

ricky intervient obsessivement sur mes messages, dans le but de

me lasser pour éventuellement me faire taire sur ce forum, en
insinuant que des résistances passées à d'autres présomptueux
de son genre étaient une attitude systématique envers tout le
monde.

Comme vous pourrez le constater si vous relisez les enfilades dans
lesquelles ces supposés "physiciens" sont impliqués, ils attaquent
systématiquement, très agressivement et souvent avec rage et mépris,
(particulièrement dans le cas de ricky) tous ceux qui tentent de
donner aux jeunes les explications connues des phénomènes physiques
qui sont plus claires et simples que ce que la mécanique quantique
peut offrir.

La raison de leur acharnement particulier à mon endroit est que je

les dénonce publiquement chaque fois qu'ils s'en prennent à moi,

en référant les lecteurs à l'analyse que Franco Selleri a fait du
problème dans "Le grand débat de la théorie quantique", Flammarion,
que je conseille à tous les jeunes de lire, et parce que j'ai remis
en circulation en Amérique du Nord certaines découvertes du passé
qu'ils ne mentionnent jamais dans les ouvrages de référence

qui sont mis à la disposition des jeunes dans les universités,

parce qu'elles contredisent leur dogmes fondamentaux.

Michel Talon, un élève de John Wheeler, qui ne se gêne pas pour

traiter Louis de Broglie et ses collègues de zozos sur ce forum,
nous a raconté ici même comment ses collègues ont réussi à faire
cesser tout enseignement causaliste à la suite de la retraite de
de Broglie, et comment au CERN, on avait fait fermer le dernier
projet de recherche qui ne cadrait pas avec la philosophie de
Copenhague.

Il y a cependant moyen d'apprendre la véritable physique
fondamentale, mais plus dans les facultés des universités, où
les programmes sont entièrement assujettis à la philosophie de
Copenhague depuis les années 60. Le seul moyen d'y parvenir
actuellement est de le faire hors du cadre académique.

À force de "discuter" avec ricky, j'ai fini par résumer, suite
à ses affirmations et confirmations, les principaux dogmes de

leur philosophie irrationnelle dans deux pseudo-définitions en

apparence caricaturales, mais absolument pas exagérées,
métaphoriquement destinées à paraître dans un dictionnaire du

futur, et que vous trouverez au lien mentionné ci-dessous.

--------------------------------------------------------
"École de pensée de Copenhague" :
--------------------------------------------------------
"Minus copenhagensis" :
--------------------------------------------------------

http://groups.google.com/groups?dq=&hl=fr&lr=&ie=UTF-8&threadm=3DA017E2.7F170F67%40globetrotter.net&prev=/groups%3Fhl%3Dfr%26group%3Dfr.sci.physique

--

Sylvain

non lue,
25 oct. 2002, 17:48:2725/10/2002
à
Merci pour tes réponses.

>j'avoue que je suis en accord pour la partie "explication"... la MQ
est

>predictive mais n'explique pas au sens physique du terme ...

Certes, pour ce point ma réponse était un peu rapide face à plusieurs
propos différents à la fois. Mon avis est qu'elle montre de par ses
prédictions objectives et vérifiées l'impossibilité d'une telle
"explication" de nature physique.
Et une démonstration d'insolubilité est une manière de résoudre la
question.
___________

Je conclurerai maintenant cette discussion que j'avais tentée avec
André Michaud sur le thème de l'expérience EPR.
Bien sûr je ne me faisais pas d'illusion sur son compte, j'avais bien
remarqué la situation que je vais récapituler.

C'est la réalité physique qui a imposé aux physiciens il y a bientôt
un sciècle d'adopter la théorie quantique pour la décrire, comme étant
de loin celle qui s'est avérée la mieux adaptée pour en rendre compte,
parmi toutes les formules qu'il était raisonnablement possible
d'élaborer.
Cette théorie a la propriété d'être en conflit direct avec les
conceptions philosophiques naturelles "réalistes" ou "causalistes" que
beaucoup de gens ont tendance à avoir a priori, liées à l'idée de
"réalité physique". A savoir l'idée que cette réalité physique serait
la substance ultime de l'existence, une réalité autonome et causale
(ou tout qualificatif qu'on préfèrera en précisant la signification
exacte à lui donner...).
Nombreux sont les physiciens qui s'en sont rendus compte.
Certains ont même tenté de résister à cette constatation, ont voulu
croire que ce n'était là qu'une étape provisoire qui serait bientôt
dépassée par la découverte de mécanismes plus profonds qui nous ferait
sortir de cette situation en explicitant des variables cachées...
Mais rien n'y fit, la théorie quantique tint bon, ses prédictions
s'avérant toujours plus précisément en accord avec l'expérience.

D'autres et non des moindres comme Feynman, ayant renoncé à la
contester face à ses succès objectifs, la qualifièrent
d'incompréhensible, comme une manière de qualifier d'irréductible ce
conflit entre ce qu'on sait sur la réalité d'une part, et d'autre part
le genre de conception réaliste a priori qu'on a tendance à se faire
dessus, qu'on aurait souhaité retrouver et qu'on est insatisfaits de
ne pas avoir finalement trouvé dans la théorie.
Ainsi ils voyaient toujours bien le paradoxe, et si on n'a rien trouvé
de plus ce n'est pas faute de s'être posé la question.
Mais la réalité physique n'ayant cure des conceptions philosophiques
de ceux qui croient en elle, la multiplication et l'élargissement
spectaculaires des confirmations de la théorie quantique qu'on y
trouva, contraignit finalement tous les physiciens honnêtes à
finalement admettre ces faits en pratique.

Maintenant, André Michaud n'a de cesse depuis longtemps de traiter de
fous, d'imbéciles et de malhonnêtes tous les physiciens ayant eu cette
honnêteté justement, comme s'ils n'avaient adopté volontairement ce
paradoxe comme un dogme arbitraire en interdisant au monde entier de
réfléchir autrement, alors qu'en réalité c'est l'examen logique qui
les a amenés à cette conclusion.
Il étale sa paranoïa dans les newsgroups, refusant toujours de se
remettre en question ou de s'aventurer à réfléchir et à répondre
sérieusement aux questions sensibles susceptibles de le déstabiliser.
Avant de m'y mêler, j'ai vu qu'objectivement j'étais dans le lot de
ceux qu'il insultait, puisque ses insultes s'adressaient, d'après leur
contenu, à tous ceux qui se rangeaient à la même constatation, ceux
qui ne s'obstinent pas à chercher encore à faire dire à la réalité
physique le réalisme qu'elle avait visiblement nié par sa confirmation
d'une théorie incompatible avec cette philosophie.

Malgré cela j'ai tenté de l'aborder par la voie de la raison, comme
pour essayer de lui faire enfin comprendre son erreur (ou celle de son
attitude) dans l'espoir de l'amener à se calmer et à cesser de
perturber/encombrer ce newsgroup de ses folies et arrogantes insultes.
Non que j'aie osé me croire plus fort à ce jeu que tous les autres qui
ont déjà tenté en vain de le faire jusqu'à présent. Simplement que
j'étais en forme et que j'avais envie d'essayer à mon tour.

Maintenant, j'envoie ce dernier message en retard, notamment le temps
de m'être informé plus précisément de l'état des connaissances
actuelles pour éviter le vague des pétitions de principe où on ne
saurait pas dire assez précisément où on en est.

J'ai donc cherché sur xxx.lpthe.jussieu.fr les articles sur les
expériences récentes d'EPR, et j'ai trouvé finalement
quant-ph/0007008
J'ai alors demandé à son auteur, Valerio Scarani, de me résumer la
situation actuelle sur le sujet.

Voici ce qu'il m'a répondu (avec sa permission de copie):

[PRA = Physical Review A ; PRL = Physical Review Letters]
------------

Pour les experiences EPR, la situation est:

L'echappatoire de localite a ete ferme a plusieurs reprises, soit par
changement aleatoire au dernier moment [Weihs et al, Phys Rev Lett
1998], soit par choix passif [en arrivant a un coupleur, c'est la
particule elle-meme qui "decide" de quel cote partir; p.ex. Gisin &
Zbinden, Phys. Lett. A 1999]. Dans ces experiences avec des photons
reste ouverte cependant l'echappatoire de detection. Celle-ci a ete
fermee dans un autre type d'exerience mettant en evidence des
correlations quantiques [Rowe et al, Nature 2001], mais dans ce cas
les particules etaient tres proches les unes des autres.

Pour resumer donc, les deux echappatoires [taux d'échec de détection /
direction de mesure fixée d'avance] ont ete fermees separement; reste
a les fermer dans une seule experience. Cela dit, l'echappatoire de
detection est consideree comme bizarre: comme disait John Bell, il est
difficile de croire que la physique quantique donne des predictions
parfaites pour des detecteurs mauvais, et qu'elle va s'ecrouler
miserablement des que les detecteurs deviendront bons!!

J'en profite pour te signaler que j'ai discute en detail tous ces
arguments dans un livre grand public, "Quand la matiere nous etonne",
a paraitre chez Vuibert en printemps 2003. J'espere que ce texte
fournira une bonne base pour des discussions entre non-specialistes:
il est important que plus de personnes entrent en contact avec les
phenomenes quantiques!
-----------
Mail suivant, en réponse à mon évocation de la page de John Baez sur
le sujet:
------------

Je veux bien repondre a ton mail, mais a moment donne si le theme
t'interesse vraiment je te conseille d'etudier le sujet toi-meme... ce
n'est pas pour rien que je ne participe a aucun newsgroup moi-meme!

Apres lecture diagonale, ce M. Baez (que je ne connaissais pas) semble
proner une certaine forme de "collapse" que les experiences d'Aspect
n'auraient pas exclu. Il semble ignorer l'experience Weihs et al.
1998, qui a mon avis (mais je n'ai pas verifie les chiffres) devrait
clore l'echappatoire qu'il propose. Au sujet des modeles de collapse
d'ailleurs, je pense que les experiences de Bell dites "en
configuration relativiste" realisees a Geneve, ainsi que toute la
discussion qui les a motivees, peuvent servir a refroidir certains
enthousiasmes: Zbinden et al, PRA 2001; Stefanov et al, PRL 2002. [ou
au format électronique: d'abord quant-ph/0009055, qui a ete ecrit
avant les experiences de Stefanov et al. Ensuite: Stefanov et al.,
quant-ph/0110117 (version longue: 0210015); Zbinden et al,
quant-ph/0002031 (version longue: 0007009).]

Avant que tu ne sombres dans des vaines discussions depassees sur les
variables cachees, je veux te signaler qu'il y a une vision "rajeunie"
des inegalites de Bell, consistant a les considerer comme des
indicateurs de "useful entanglement". Dans le contexte de la mecanique
quantique, la violation de l'inegalite de Bell s'ecrit
Tr(B*rho)>1, avec B = operateur de Bell.
Ce n'est pas une (in)equation "habituelle" en physique quantique,
theorie dans laquelle nous sommes plutot habitues a diagonaliser des
operateurs. En fait, personne n'aurait jamais eu l'idee d'ecrire cette
inequation sans la motivation originale de Bell. Et cependant, cette
inequation a beaucoup d'applications sans faire reference aucune aux
variables cachees. Notamment, il a ete demontre que si l'etat rho
satisfait cette inequation (cad, viole l'inegalite de Bell), alors:
(1) certains protocoles de crypto quantique sont surs (Scarani, Gisin
PRL 2001 - quant-ph/0101110 - version longue: 0104016);
(2) l'entanglement de rho est "distillable" (Acin et al, PRA a
paraitre, mais je conseille plutot quant-ph/0112102);
(3) on peut construire des protocoles de quantum communication
complexity (Brukner et al, PRL 2002).
Rien que ces indices montrent a mon avis que les inegalites de Bell
"signifient quelque chose", meme au dela de la raison pour laquelle
Bell lui-meme les avait concues. Cette direction me parait plus
fructueuse que celle d'aller chercher la petite bete pour ressusciter
les variables cachees!

Avec toutes ces references, toi et tes amis avez de la bonne lecture
pour un moment! Et vous verrez peut-etre qu'il y a encore pas mal a
faire et a comprendre, mais pas dans la direction de ressusciter les
variables cachees.

(...)
Non, je n'ai pas le temps d'ecrire a toutes les personnes qui
negligent les references d'autrui. Pour Internet, c'est sans espoir de
contrer toutes les betises qui y circulent; pour les articles soumis a
des revues, c'est la tache des referees que de signaler d'eventuelles
omissions.

Valerio

----------------

Je réponds maintenant à quelques passages d'AM. Je ne réponds pas par
d'autres questions car je ne compte pas continuer la discussion, en
tout cas pas avec lui.

>> Les théories en vigueur que l'expérience a suggéré forment un
>> ensemble cohérent,
>

>Absolument pas. La MQ et la GR sont totalement irréconciliables
>depuis le début. Même après 30 ans, il est toujours impossible
>de résoudre les équations de la chromodynamique quantique de
>manière précise pour même un seul nucléon.
>
>Belle cohérence!

Deux remarques bien différentes. Pour la première, il s'agit d'un
problème très lointain qui n'est pas une contradiction interne de la
MQ dans son domaine d'application (l'infiniment petit), et n'est pas
une contradiction de principe mais un problème contigent lié au nombre
de dimensions de l'espace-temps qui n'est pas celui qu'il faut pour
que l'invariance de jauge de la gravitation retombe sur ses pieds
(d'après ce que j'ai entendu dire).
Autant dire que ce problème ne se voit pas à l'oeil nu sur la
formulation de la MQ de base et que seuls les spécialistes de théorie
quantique des champs peuvent la discerner.
Ensuite, le domaine expérimental où cette confrontation serait
testable est bien au-delà du domaine où on est confronté aux problèmes
de divergence interne des calculs du Modèle Standard. Ce dernier est
l'application de la MQ à un certain ensemble de champs en interaction,
et les divergences sont liées à ces champs particuliers et à leurs
interactions sans que les principes de la MQ elle-même et les
paradoxes qui y sont liées ne soient en cause.
Ce qui nous amène au 2ème point: si on ne sait pas résoudre les
équations de la chromodynamique quantique, c'est parce que ces
équations sont difficiles, ce qui ne présume en rien d'une incohérence
de la théorie, si ce n'est qu'elle serait peut-être en manque de
reformulation qui permettrait de l'aborder numériquement par le bon
bout.
De même ce n'est pas parce que le mouvement des trois corps en
gravitation de Newton n'est pas résoluble par des formules exactes que
ce mouvement n'existe pas ni que la théorie de Newton qui en donne
l'équation serait fausse. La nature se moque de nos difficultés à la
suivre dans ses retranchements même si on en connaît la direction.

Alors bien sûr on ne sait pas encore tout et il reste des découvertes
à faire pour retrouver une cohérence théorique globale (théorie des
cordes...), mais ce qui coince en attendant n'a rien à voir avec les
paradoxes de la MQ qui étaient l'objet ici.

>L'électrodynamique quantique ne décrit pas le photon. Elle utilise
>la méthode lagrangienne pour substituer la notion métaphorique de
>photons virtuels à l'interaction coulombienne sous-jacente entre
>les particules.

Le terme de "photon virtuel" est peut-être métaphorique mais la
théorie mathématique à laquelle il se réfère est clairement définie.

>À vrai dire, la MQ n'est en conflit avec rien du tout.

D'accord. En effet, le causalisme n'est qu'un produit de l'imagination
de l'homme que rien ne fonde concrètement, en sorte que le conflit de
la MQ avec lui n'est pas un conflit avec quelque chose qui existe en
réalité.

>C'est l'interprétation de Copenhague qui l'érige en solution finale,

>ce qui est impossible puisqu'elle ne peut intégrer les
>niveaux d'énergie des quarks up et down.

Articulation complètement fantaisiste entre deux choses qui n'ont rien
à voir.

>Alors, j'imagine parfaitement l'hystérie collective qui s'emparera
>du lot si je fait mine de commencer à expliquer comment ses
>conclusions peuvent permettre d'intégrer l'ensemble des constats
>expérimentaux.

On arrive là en plein délire de science-fiction. Les gens ont une tête
pour réfléchir.
S'ils voient quelqu'un qui raconte des idioties ils le disent,
essaient de montrer l'erreur.
Ce n'est pas de l'hystérie que de ne pas se laisser convaincre par la
répétition des mêmes idioties et d'avoir quelque chose à y répondre,
du moins tant que ça a l'air de quelque chose de précis auquel il soit
possible de répondre.
Si AM avait quelque explication véritable à proposer, on pourrait
arrêter de le voir comme un idiot. Mais il sait, consciemment ou pas,
qu'il n'a que des bêtises à dire et qu'on le réfuterait aussitôt,
c'est pour cela qu'il n'ose pas les dire.

>Aucune particule ne peut avoir une "taille nulle".

Ce qui a une taille non nulle en un certain sens, est divisible. Mais
on ne peut pas couper un électron en deux par exemple. Je ne vois que
la fonction d'onde pour résoudre ce paradoxe.

>Si on ne s'entend pas sur la métaphore de la chaise, il n'existe
>aucun moyen pour que nous nous comprenions.

Mon avis n'était pas simplement "le contraire" du sien, le problème
est de préciser les choses plus en profondeur, ce que je ne vais pas
recommencer ici puisque je l'ai déjà fait dans mon texte
0.Introduction sur mon site.

>> Que peut être une courbe complexe qui ne soit pas une courbe ?
>

>Touché.

Quel art d'entretenir le mystère !

>(les pistes) sont parfaitement claire(s).

et clairement absentes de tout ce qu'il veut bien nous révéler de
manière ainsi accessible par internet (à part un site en langue
allemande non merci).

>> 2) Pour être recevables, ces informations (pistes de recherches +
>> résultats accumulés) dont vous affirmez ici l'existence devraient
>> au moins être gratuitement disponibles sur Internet,
>

>Vous avez une étrange conception de la recevabilité.

réponse de dinosaure qui refuse de voir le progrès technique évident à
tous points de vue des formes de l'édition dans lequel le monde est en
train d'entrer. Je pense au phénomène général bien connu du racket de
l'essentiel des droits d'auteur par les éditeurs, phénomène auquel ce
progrès s'apprête enfin à mettre un terme.

>Pas de URL. J'ai retiré tout droit de regard à notre si
>orgeuilleuse élite en physique fondamentale.

C'est donc le secret jalousement gardé d'une secte (à un seul individu
apparemment) qui condamne le monde entier pour ne pas avoir déjà
adhéré à ses si limpides révélations qu'elle a en même temps toujours
refusé de lui faire.

>La génération montante se chargera de vous expliquer de quoi il
s'agit.

On se demande comment elle pourrait ne serait-ce qu'être au courant
dans ces conditions.

>À les lire, j'ai trouvé qu'ils valaient chaque centime du faible
>prix que j'ai du verser pour me les procurer. Je les conseille à
>tous les jeunes.

Une démonstration d'une chose à laquelle on n'adhérait pas ne doit
être payable qu'après satisfaction. Car si on constate après achat de
livres, que cela ne démontre pas ce qu'un certain urluberlu prétendait
que cela démontre, ce qu'on savait déjà d'avance comme tout le monde
mais qu'on voulait simplement ne pas avoir l'air d'un grincheux qui ne
veut pas examiner les arguments de cet urluberlu, qui remboursera ?

--------------------
Enfin, commentaires sur les insultes paranoïaques qu'AM ne cesse de
proférer.

> "École de pensée de Copenhague" (...)
>tous les physiciens de la seconde moitié du 20e siècle ont adhéré
sans le moindre questionnement
(...)
>Le point le plus important, dans ce lavage de cerveau, c'est le
>problème de la compréhension.... Bohr, Heisenberg, Pauli et compagnie
disaient tous :
>" Ne cherchez pas à comprendre la mécanique quantique, elle est
presque totalement incompréhensible."

A mon avis, faut vraiment manquer de psychologie pour prendre ça au
premier degré.
Naturellement, sauf pour des étudiants qui seraient des nouilles de
toute manière (oui bon j'avoue qu'il peut y en avoir beaucoup mais
quand même), je ne vois pas comment on peut suivre un cours présentant
une théorie nouvelle accompagnée de cet avertissement sans chercher de
toute manière à comprendre ce cours. Cet avertissement n'est pas fait
pour être admis comme un dogme, et il ne peut pas l'être: ce n'est pas
un tel avertissement qui freinera la tentative des étudiants d'y
réfléchir, et je dirais même qu'ils doivent y réfléchir de toute
manière, ne serait-ce que pour être capables d'utiliser ce cours en
pratique.
Cet avertissement est là pour signaler qu'il n'y a pas à l'interpréter
comme (ni chercher derrière) l'expression d'une réalité physique
essentielle et causale. Il est là pour signaler aux étudiants que
s'ils cherchent et ne trouvent pas une telle interprétation c'est
normal, les autres l'avaient bien déjà remarqué, et que donc ne pas y
arriver n'est pas une raison pour se le reprocher comme si on était
fautif de n'avoir pas compris.
Et je ne trouve pas si difficile que cela de retrouver ces
constatations (ou si ce n'est redémontrer l'impossibilité, du moins
constater qu'on ne trouve pas de solution naturelle en essayant un
peu) par quelques expériences de pensée à partir de la connaissance du
fait que les prédictions de la théorie quantique ont été vérifiées.

Peut-être cette recommandation pouvait être ressentie comme un dogme
et une interdiction de réfléchir de par sa présentation et l'ambiance
d'un certain cours, mais ce genre d'aspect n'est alors plus du tout un
problème de fond.
Sinon, j'ai remarqué aussi l'ambiance de lavage de cerveau qui règne
en prépa, mais c'est un problème sociologico-politique qui n'a rien à
voir avec une quelconque affaire d'imposition d'une théorie ou
philosophie particulière.

>"Minus copenhagensis" :
>b) que si cette réalité physique existait tout de même, elle ne
pouvait être comprise
>que par Dieu (...) étant donné qu'ils se considéraient eux-mêmes
comme une sous-espèce
>d'homo sapiens sapiens qui était incapable, contrairement à l'espèce
mère, de
>raisonner logiquement

Il n'y a qu'une espèce d'hommes, tous capables de raisonner
logiquement et ainsi de constater, à partir d'un minimum d'habileté
d'esprit et s'ils veulent bien en faire l'effort, par quelle malice la
réalité fondamentale se dérobe à toute tentative de la saisir en
termes de causalité physique.

> contrairement aux preuves expérimentales déjà connues et au simple sens commun,
>toutes les particules étaient physiquement des paquets d'ondes;

Tout ce qui a été expérimenté est cohérent avec la théorie quantique.
Quant au sens commun, tout le monde n'a pas fait joujou avec des
particules quand il était petit pour savoir comment elles sont
physiquement.

>Le déclin de la confrérie fut fulgurant après que de nombreux
étudiants eurent pris
>connnaissance, dans des circonstances demeurées nébuleuses,
d'importantes découvertes

Si cela avait été le cas, ça se saurait et ils seraient nombreux à
faire chacun un petit site web qui présente ces découvertes, ce qui
est contradictoire avec l'affirmation plus haut du fait que ces
fantomatiques découvertes ne sont pas trouvables sur Internet.
A moins bien sûr qu'il ne fasse là qu'un bluff d'anticipation de ce
qu'il prétend qui arriverait à l'avenir, lorsque ce "dictionnaire du
futur" serait publié.
Mais on entre alors dans un pur délire de science-fiction.

------

En conclusion, je vois mal comment on pourrait arrêter l'encombrement
du newsgroup par ce genre de folies.
Ou peut-être une suggestion: s'il est difficile de censurer les
perturbateurs (en mettant leur identité dans un killfile ou je ne sais
quoi - il y en a déjà deux, am et rh), une bonne idée pourrait être
d'ouvrir un nouveau newsgroup qui serait spécialement dédié aux gens
de leur espèce et où ils pourraient librement échanger leurs idées
fumeuses sans déranger ceux qui préfèrent discuter des théories
communément acceptées.

Par exemple quelque chose comme "fr.sci.physique.alternatives".
Les suggestions sont ouvertes !

SP
http://spoirier.lautre.net/

srp

non lue,
26 oct. 2002, 00:37:4126/10/2002
à
Je me permet de contribuer mon grin de sel, puisqu'on mentionne mon
nom dans ce message.

Sylvain a écrit :


>
> Merci pour tes réponses.
>
> >j'avoue que je suis en accord pour la partie "explication"... la
> >MQ est predictive mais n'explique pas au sens physique du terme
> >...
>
> Certes, pour ce point ma réponse était un peu rapide face à plusieurs
> propos différents à la fois. Mon avis est qu'elle montre de par ses
> prédictions objectives et vérifiées l'impossibilité d'une telle
> "explication" de nature physique.
> Et une démonstration d'insolubilité est une manière de résoudre la
> question.
> ___________
>
> Je conclurerai maintenant cette discussion que j'avais tentée avec
> André Michaud sur le thème de l'expérience EPR.
> Bien sûr je ne me faisais pas d'illusion sur son compte, j'avais bien
> remarqué la situation que je vais récapituler.
>
> C'est la réalité physique qui a imposé aux physiciens il y a bientôt
> un sciècle d'adopter la théorie quantique pour la décrire, comme
> étant de loin celle qui s'est avérée la mieux adaptée pour en rendre
> compte, parmi toutes les formules qu'il était raisonnablement
> possible d'élaborer.

Erreur. Ce que la réalité physique a imposé, est le constat qu'au
niveau fondamental, toute l'énergie se manifestait seulement sous
forme de quanta discrets. Voilà ce que la réalité physique a imposé.

> Cette théorie a la propriété d'être en conflit direct avec les
> conceptions philosophiques naturelles "réalistes" ou "causalistes"
> que beaucoup de gens ont tendance à avoir a priori, liées à l'idée
> de "réalité physique". A savoir l'idée que cette réalité physique
> serait la substance ultime de l'existence, une réalité autonome et
> causale (ou tout qualificatif qu'on préfèrera en précisant la
> signification exacte à lui donner...).

C'est l'affirmation, depuis Heisenberg, que la mécanique quantique
est le fondement de la réalité qui est en conflit direct avec
l'idée mème de la causalité. Pas le fait que toute l'énergie
fondamentale ne se manifeste physiquement que sous forme
d'événements électromagnétiques discrets.

Le fait que l'énergie fondamentale ne se manifeste que sous forme
de quanta discret ne peut pas être en contradiction avec la réalité
physique puisqu'elle constitue la réalité physique proprement
dite.

> Nombreux sont les physiciens qui s'en sont rendus compte.
> Certains ont même tenté de résister à cette constatation, ont voulu
> croire que ce n'était là qu'une étape provisoire qui serait bientôt
> dépassée par la découverte de mécanismes plus profonds qui nous ferait
> sortir de cette situation en explicitant des variables cachées...
> Mais rien n'y fit, la théorie quantique tint bon, ses prédictions
> s'avérant toujours plus précisément en accord avec l'expérience.

Ce que "certains physiciens" ont espéré, était que la dérive de
l'école de Copenhague soit une étape provisoire, comme de Broglie
le mentionne dans un de ses ouvrages, mais rien n'y fit durant le
20e siècle, avec pour conséquence que toute recherche des fondements
physique a cessé depuis 70 ans.



> D'autres et non des moindres comme Feynman, ayant renoncé à la
> contester face à ses succès objectifs, la qualifièrent
> d'incompréhensible, comme une manière de qualifier d'irréductible
> ce conflit entre ce qu'on sait sur la réalité d'une part, et
> d'autre part le genre de conception réaliste a priori qu'on a
> tendance à se faire dessus, qu'on aurait souhaité retrouver et
> qu'on est insatisfaits de ne pas avoir finalement trouvé dans la
> théorie.

Ce que Feynman a réussi à faire, fut de masquer sous le voile
de ses photons virtuels, la réalité objective de l'interaction
coulombienne. Il n'a rien fait de plus que d'embrouiller encore
plus la piste menant à la reprise de la recherche des foncements
de la réalité physique objective.

À vrai dire, l'héritage de Copenhague fut la conquête du domaine
de la physique par des mathématiciens, qui se sont pris, demeurant
seuls en scène, pour de véritables physiciens depuis.

> Ainsi ils voyaient toujours bien le paradoxe, et si on n'a rien
> trouvé de plus ce n'est pas faute de s'être posé la question.

Vous parlez là des inconditionnels de l'école de Copenhague, les
seuls qui ont eu droit de parole depuis 50 ans.

> Mais la réalité physique n'ayant cure des conceptions philosophiques
> de ceux qui croient en elle, la multiplication et l'élargissement
> spectaculaires des confirmations de la théorie quantique qu'on y
> trouva, contraignit finalement tous les physiciens honnêtes à
> finalement admettre ces faits en pratique.

Elle convainqui en effet par "lavage de cerveau" (les mots mêmes
de Popper) toutes les générations depuis 50 ans. Comment aurait-il
pu en être autrement! Tout comme dans les églises, on n'enseigne
que la religion du maître des lieu, dans les facultés de physique,
on a fait de même avec la philosophie de l'école de pensée de
Copenhague.

Lorsque seulement une partie de la vérité est enseignée aux élèves,
ceux-ci n'ont aucune chance de porter un jugement éclairé.



> Maintenant, André Michaud n'a de cesse depuis longtemps de traiter
> de fous, d'imbéciles et de malhonnêtes

Là, vous déraillez complètement. Jamais je n'ai dit une telle chose.

Les seuls individus que j'ai traité avec mépris lors de conversations
sont ceux qui ne connaissent pas le sens du mot courtoisie, et ceux
qui approuvaient silencieusement ou directement leur démarche en
arrière plan.

J'ai le plus profond respect de tous les autres.

> tous les physiciens ayant eu cette honnêteté justement,

Voir plus haut. Depuis 50 ans, on ne permet pas aux étudiants de
porter un jugement éclairé. On les endoctrine, tout simplement.

> comme s'ils n'avaient adopté volontairement ce
> paradoxe comme un dogme arbitraire en interdisant au monde entier de
> réfléchir autrement, alors qu'en réalité c'est l'examen logique qui
> les a amenés à cette conclusion.

Bien sûr, l'examen logique de seulement une partie de la vérité.

Mais je ne vous ai pas vu répondre aux objections de Selleri, et
je suis certain qu'on se garde bien de faire la promotion de son
analyse dans les facultés de physique, tout comme on a jeté aux
orties toute la pensée de de Broglie.

En fait d'honnêteté intellectuelle, je n'ai pas de leçon à recevoir
de vous.

> Il étale sa paranoïa dans les newsgroups, refusant toujours de se
> remettre en question ou de s'aventurer à réfléchir et à répondre
> sérieusement aux questions sensibles susceptibles de le déstabiliser.

Mais c'est tout réfléchi, mon beau, et depuis longtemps. 70 ans de
palabres futiles de la part de Einstein et de Broglie entre-autres
suffisent à tirer la conclusion que toute nouvelle entreprise du
genre est vouée à l'échec.

Profitez bien des dernieres années de l'hégémonie de la philosophie
de votre école de Copenhague. Toute ce que vous pourrez dire ne
changera rien à ce qui va arriver.

> Avant de m'y mêler, j'ai vu qu'objectivement j'étais dans le lot de
> ceux qu'il insultait, puisque ses insultes s'adressaient, d'après
> leur contenu, à tous ceux qui se rangeaient à la même constatation,
> ceux qui ne s'obstinent pas à chercher encore à faire dire à la
> réalité physique le réalisme qu'elle avait visiblement nié par sa
> confirmation d'une théorie incompatible avec cette philosophie.

Mes "insultes", comme vous dites, ne s'adressaient qu'aux petits
m...... anonymes qui tentaient de me faire sur ce forum par leurs
insultes. Pour le reste, si vous trouvez que de combatre publiquement
une philosophie que je trouve irrationnelle est une insulte qui vous
est portée personnellement, c'est qu'il y a un problème de mise
en perspective de votre part.

Encore une fois, je n'ai pas de leçon à recevoir de quelqu'un qui
se taisait (et ainsi approuvait tacitement) face à des agressions
comme celles auxquelles ces m...... anonymes se sont adonnées ici
pendant des années envers tous ceux qui ne se "conformaient" pas.
Leur échec avec moi semble avoir calmé leurs ardeurs envers tous les
autres.

Peut-être les dissidents oseront-ils progressivement refaire
surface. Ce sera avec ma bénédiction, soyez-en certain. Ce
forum n'est pas modéré. Vive la liberté de parole.

> Malgré cela j'ai tenté de l'aborder par la voie de la raison,

Je vous ai répondu de la même manière, en vous amenant les objections
de Selleri. Résultat: silence et défilement.

> comme pour essayer de lui faire enfin comprendre son erreur (ou
> celle de son attitude) dans l'espoir de l'amener à se calmer et à
> cesser de perturber/encombrer ce newsgroup de ses folies et
> arrogantes insultes.

Ben, quand on ne m'insulte pas, je ne perturbe pas.

Mais rien ne m'empêchera de dénoncer l'irrationalité de la
philosophie de l'école de Copenhague lorsque des dérives
excessives se font jour sur ce forum à l'attention de jeunes
lecteurs, comme s'il s'agissait de vérités de l'évangile.

> Non que j'aie osé me croire plus fort à ce jeu que tous les autres
> qui ont déjà tenté en vain de le faire jusqu'à présent. Simplement
> que j'étais en forme et que j'avais envie d'essayer à mon tour.

Soit dit sans vous offenser, je vous voyais venir à 100 km.

MaIS Pourquoi pas mettre votre plus gros canon en batterie ?

Il doit bien exister dans la loge francophone de votre confrérie
des "grands maîtres de la physique" quelqu'un qui soit assez
d'envergure pour me coincer, non?

Pourquoi ne pas nous l'amener ici ? J'ai quelques questions à lui
poser, en espérant qu'il ne se défilera pas lui aussi.

Eh ben voilà! C'est exactement les conclusions de Selleri. Encore
rien de définitivement concluant. Rien qui ne confirme la théorie
de Bohr au sujet du paradoxe EPR.

> Cela dit, l'echappatoire de detection est consideree comme bizarre:
> comme disait John Bell, il est difficile de croire que la physique
> quantique donne des predictions parfaites pour des detecteurs
> mauvais, et qu'elle va s'ecrouler miserablement des que les
> detecteurs deviendront bons!!

C'est beau d'avoir la foi! Votre problème, est que les détecteurs
ne peuvent pas devenir meilleurs. Vous oubliez qu'ils sont faits
des mêmes particules électromagnétiques, qui sont du même ordre
de grandeur, que les particules que vous tentez de mesurer.

Gardant ce fait en mémoire, je vous suggère de relire Heisenberg,
votre maître à penser, pour comprendre pourquoi il est impossible
que vos détecteurs deviennent "bons" un jour.

Vous avez déjà perdu. Votre seul problème est que vous n'en avez
pas encore pris conscience.

fda

non lue,
26 oct. 2002, 04:31:5426/10/2002
à

Sylvain wrote:

> Par exemple quelque chose comme "fr.sci.physique.alternatives".
> Les suggestions sont ouvertes !

Ce serait une plutôt bonne idée. Avec réponse là-bas et là-bas seulement
à toutes les contributions de ce genre faites ici. Ca pourrait archer si
chacun se donne le mot.

Mais c'est à appliquer avec doigté, car il est parfois difficile de
distinguer entre tel ou tel délire particulier et le simple contresens
involontaire de quelqu'un qui a mal saisi tel ou tel concept et vient
ici à la pêche aux infos... ce lieu étant après tout un peu fait pour
cela :o)

Au fait, quid de contresens courants concernant non pas la physique,
mais l'interrogation de la physique sur un sujet qu'il faut poser à une
autre discipline ? Au lycée, une question m'empêchait de dormir et ma
prof de physique ne semblait pas savoir y répondre : "Le mélange du bleu
et du jaune donne du vert, qui se trouve entre les deux dans le spectre.
Bien. Mais pourquoi le mélange du bleu et du rouge donne-t-il du violet
qui ne se trouve *pas* entre les deux ?". Je m'étais mis à échafauder
quelques loufoquerie bizarre invoquant des battements ou je ne sais
quoi. La vraie réponse arriva deux ans plus tard par où je ne
l'attendais pas, à savoir par le cours de sciences naturelles. Pensons
que la même chose peut arriver à beaucoup de gens, et que la FAQ doit
peut-être ne pas les oublier non plus.

Moky

non lue,
26 oct. 2002, 11:37:3026/10/2002
à
Et si on arrêtait de se battre à propos de l'interprétation à donner à
la MQ ? De toute façon, on sait que les équations sont justes : QED
donne la structure hyperfine de l'hydrogène avec plus de 20 décimales
exactes ! Il est absolument impossible de nier la véracité d'une telle
théorie. Quelle que soit LA réalité, en ce qui concerne sa partie
mesurable, elle doit se réduire à la MQ dans la limite où l'on se
place dans son domaine d'application.

En ce qui concerne Copenhage, son interprétaion est de toute façon
fausse et inutile à discuter. Elle se base sur une théorie non
relativiste de la MQ, nous avons à présent de bien meilleures théorie.
C'est à partir d'elles qu'il faut discuter et il se fait qu'en matière
de contradiction avec le sens commun, elles sont encore pires que la
MQ proprement dite.

Pour André :

Feynmann ne voile rien du tout sous ses photons virtuels. Et en
particulier pas l'interaction coulombienne. Car celle-ci n'existe plus
! Fini ! Coulomb est faux, Maxwell avait tors. Ce ne sont que des
approximations de la réalité. Celle que QED nous donne est bien plus
performante.
Les photons virtuels ne sont jamais que des fonctions de Green qui
arrivent dans le développement perturbatif des intégrales de chemin.
Comme ces fonctions apparaîssent sous forme de propagateurs de
Feynmann entre deux points de l'espace qui ne sont ni le point de
départ ni le point d'arrivé du propagateur total, il nous a paru
logique de les appeler "photon" aussi car ils font intervenir le même
genre d'intégrales. Mais comme ces "photons" n'apparaîssent que entre
les endroits où l'on mesure la position de la particule, on a cru bon
de les appeler "virtuels".
Si ça dérange André de voir des photons virtuels et qu'il préfère
voir la physique telle qu'elle est, il ne lui reste plus qu'à calculer
des amplitudes sans recours au support visuel des diagrammes de
Feynmann.

Disons les choses autrement. L'opérateur d'évolution en TQC
(théorie quantique des champs) est en gros l'exponentielle du
hamiltonien. Dans ce hamiltonien figure des champs qui correspondent à
des particules réelles a priori et des produits de ces champs qui font
les interactions.
Lors du développement de l'expo (que l'on met en braket pour
calculer des amplitudes de transitions), il apparaît des termes qui
sont exactement des propagateurs de particules entre des points
intermédiaires.
Extraordinaire côïncidence ! Alors autant en profiter et appeler
ces propagateurs "particules virtuelles" parcque mathématiquement ces
termes ont la même forme.

Ensuite, faut ouvrir les yeux : les mathématiciens ne sont pas du
tout seuls maître à bord. Ce n'est pas parcqu'André ne comprend pas la
mathématique liée à la physique qu'il doit croire qu'il n'y a qu'elle.
En effet, la majorité des théorèmes liés par exemple aux intégrales de
chemin de Feynmann ne sont pas encore démontrés de façon rigoureuse.
Ca ne nous empêche pas de les utiliser depuis plus de 40ans. Si il
fallait attendre que les mathématiciens s'occupent de la physique, on
en serait encore à se demander si l'hydrogène a un niveau fondamental.
Je ne rigole pas : il y a moins de dix ans qu'on a démontré
l'existence d'un niveau fondamental pour l'hydrogène non relativiste !

A bientôt,
Moky (fana de phys math)

srp

non lue,
26 oct. 2002, 11:46:0626/10/2002
à
Sylvain a écrit :

srp (cité hors contexte):

> >Alors, j'imagine parfaitement l'hystérie collective qui s'emparera
> >du lot si je fait mine de commencer à expliquer comment ses
> >conclusions peuvent permettre d'intégrer l'ensemble des constats
> >expérimentaux.
>
> On arrive là en plein délire de science-fiction. Les gens ont une
> tête pour réfléchir.
> S'ils voient quelqu'un qui raconte des idioties ils le disent,
> essaient de montrer l'erreur.
> Ce n'est pas de l'hystérie que de ne pas se laisser convaincre par
> la répétition des mêmes idioties et d'avoir quelque chose à y
> répondre, du moins tant que ça a l'air de quelque chose de précis
> auquel il soit possible de répondre.
> Si AM avait quelque explication véritable à proposer, on pourrait
> arrêter de le voir comme un idiot. Mais il sait, consciemment ou
> pas, qu'il n'a que des bêtises à dire et qu'on le réfuterait
> aussitôt, c'est pour cela qu'il n'ose pas les dire.

Petit commentaire à transmettre au présomptueux auteur de ces
lignes :

Ce que j'ai a dire, je le dis sans retenue dans des centaines
d'institutions secondaires, collégiales et universitaires d'Amérique
du nord, et de plus en plus ailleurs sur la planète.

Voilà à quel point j'ose proposer une véritable explication.

De plus, L'immense majorité des quelques 900 centres de recherche à
haute énergie de la planète en ont reçu une copie. Je suis certain
qu'après avoir peiné à terminer le 4e chapitre, et ne comprenant pas
un traître mot du 5e, la grande majorité de ces grands spécialistes,
tous des "gens qui ont une tête pour réfléchir" auront fait du
papier-cul avec.

Ne vous en déplaise, je parle maintenant avec des milliers de voix
et à visage découvert dans toutes les institutions qui ont intégré
cet ouvrage dans leur bibliothèque. Et nous continuons la difusion.

Votre dilemme est que malgré que toute cette information soit de
plus en plus à la disposition de la génération montante, l'incroyable
orgueuil de l'élite en place l'empêche de même reconnaître son
existence et de la prendre en compte. Votre problème.

La diffusion continue donc avec votre bénédiction par abstention.

>
> >(les pistes) sont parfaitement claire(s).
>
> et clairement absentes de tout ce qu'il veut bien nous révéler de
> manière ainsi accessible par internet (à part un site en langue
> allemande non merci).

Le papier original de Abraham vous est donc tout aussi inaccessible
que le miens, même lorsqu'on vous le met sous le nez, puisque on
dirait que vous ne maîtrisez sa langue. Donc, selon vous, si ce
n'est pas en français, ce n'est pas de la science ?

Je m'en doutais, voyez-vous. Malheureusement, l'orgeuil et la
présomption d'omniscience de ce qui nous sert d'élite en physique
fondamentale a fait en sorte qu'il n'existe pas de traduction
française.

Un clou de plus dans le cercueil de l'école de Copenhague.



> >> 2) Pour être recevables, ces informations (pistes de recherches +
> >> résultats accumulés) dont vous affirmez ici l'existence devraient
> >> au moins être gratuitement disponibles sur Internet,
> >
> >Vous avez une étrange conception de la recevabilité.
>
> réponse de dinosaure qui refuse de voir le progrès technique évident
> à tous points de vue des formes de l'édition dans lequel le monde
> est en train d'entrer. Je pense au phénomène général bien connu du
> racket de l'essentiel des droits d'auteur par les éditeurs, phénomène
> auquel ce progrès s'apprête enfin à mettre un terme.

Réponse d'envieux qui omet de mentionner l'impossibilité de publier
quoi que ce soit dans tous les journaux formels dont les comités
d'évaluation sont tous inféodés à l'idéologie de la philosophie
de l'école de Copenhague, et qui rejettent automatiquement tout
ce qui n'est pas conforme à l'orthodoxie.

Vous et vos "éminents" collègues avez oublié le pouvoir de l'écrit.

Lorsqu'un jeune, sans préjugé, met la main sur in livre qu'il
trouve intéressant, il tend à le lire au complet, et à y revenir
pour se remettre en mémoire les aspects qui lui échappent
momentannément.

A chaque fois, un nouveau clou est planté dans le couvercle du
cercueil de la philosophie de l'école de Copenhague.

> >Pas de URL. J'ai retiré tout droit de regard à notre si
> >orgeuilleuse élite en physique fondamentale.
>
> C'est donc le secret jalousement gardé d'une secte (à un seul
> individu apparemment) qui condamne le monde entier pour ne pas
> avoir déjà adhéré à ses si limpides révélations qu'elle a en même
> temps toujours refusé de lui faire.

Au contraire, il n'y a aucun secret. je parle à visage découvert
avec les milliers de voix que constituent autant de copies de mon
ouvrage, qui est maintenant disséminé assez largement, à la portée
de la génération montante. Et nous continuons la diffusion.

> >La génération montante se chargera de vous expliquer de quoi il
> s'agit.
>
> On se demande comment elle pourrait ne serait-ce qu'être au courant
> dans ces conditions.

Eh ben, voyez plus haut. Je pense qu'il faudrait être vraiment
stupide pour ne pas comprendre ce qui est en train d'arriver.



> >À les lire, j'ai trouvé qu'ils valaient chaque centime du faible
> >prix que j'ai du verser pour me les procurer. Je les conseille à
> >tous les jeunes.
>
> Une démonstration d'une chose à laquelle on n'adhérait pas ne doit
> être payable qu'après satisfaction. Car si on constate après achat
> de livres, que cela ne démontre pas ce qu'un certain urluberlu
> prétendait que cela démontre, ce qu'on savait déjà d'avance comme
> tout le monde mais qu'on voulait simplement ne pas avoir l'air d'un
> grincheux qui ne veut pas examiner les arguments de cet urluberlu,
> qui remboursera ?

La méthode est sans faille, voyez-vous. Les têtes sont tellement
enflées d'orgueil chez notre "élite" copenhagiste en physique
fondamentale, que le moyen le plus simple de leur retirer tout
droit de regard, était de placer une condition d'accès, que leur
besoin viscéral de montrer à leur collègues à quel point ils ont
la gueule fine, leur interdisait de remplir sans perdre la face.

Assorti, bien sûr, d'une distribution gratuite sélective en
milieu académique, à la portée de la génération montante (en
cours depuis 2 ans).

Pourquoi j'en parle maintenant ouvertement, vous direz-vous.
Simple. Plus personne ne peut maintenant défaire ce qui a déjà
été fait.

Un autre clou dans le couvercle du cercueil de l'idéologie de
Copenhague.

> --------------------
> Enfin, commentaires sur les insultes paranoïaques qu'AM ne cesse de
> proférer.
>
> > "École de pensée de Copenhague" (...)
> >tous les physiciens de la seconde moitié du 20e siècle ont adhéré
> sans le moindre questionnement

Conclusion confirmée par Selleri dans sont livre "Le grand débat de
la théorie quantique", Flammarion.

> (...)
> >Le point le plus important, dans ce lavage de cerveau, c'est le
> >problème de la compréhension.... Bohr, Heisenberg, Pauli et
> >compagnie disaient tous :
> >" Ne cherchez pas à comprendre la mécanique quantique, elle est
> > presque totalement incompréhensible."

Citation des paroles de Karl Popper dans sa préface au livre de
Selleri.



> A mon avis, faut vraiment manquer de psychologie pour prendre ça au
> premier degré.

Au contraire. Le constat est sans appel pour qui lit attentivement
l'analyse de Selleri, et se donne la peine de faire quelques
vérifications personnelles dans le milieu.

> Naturellement, sauf pour des étudiants qui seraient des nouilles de
> toute manière (oui bon j'avoue qu'il peut y en avoir beaucoup mais
> quand même), je ne vois pas comment on peut suivre un cours présentant
> une théorie nouvelle accompagnée de cet avertissement sans chercher de
> toute manière à comprendre ce cours.

C'est le dilemme de tous les étudiants, dont aucun ne sont des
"nouilles", ne vous en déplaise, lorsqu'on ne leur donne pas d'autre
choix que d'adhérer ou de couler leur cours.

> Cet avertissement n'est pas fait pour être admis comme un dogme,

Pas un dogme. Un simple constat.

> et il ne peut pas l'être: ce n'est pas
> un tel avertissement qui freinera la tentative des étudiants d'y
> réfléchir, et je dirais même qu'ils doivent y réfléchir de toute
> manière, ne serait-ce que pour être capables d'utiliser ce cours
> en pratique.

Y aurait pas de problème si on leur enseignait aussi la véritable
physique en parallèle.

> Cet avertissement est là pour signaler qu'il n'y a pas à l'interpréter
> comme (ni chercher derrière) l'expression d'une réalité physique
> essentielle et causale.

Il est là pour signaler que l'enseignement de la physique se fait
par lavage du cerveau en convaincant tous les étudiant qu'il est
impossible de comprendre les lois fondamentales de la physique.

Connerie.

> Il est là pour signaler aux étudiants que
> s'ils cherchent et ne trouvent pas une telle interprétation c'est
> normal,

Ben c'est normal, vu que la MQ ne tient compte que d'un ensemble
trop restreint des constats expérimentaux.

> les autres l'avaient bien déjà remarqué, et que donc ne pas y
> arriver n'est pas une raison pour se le reprocher comme si on était
> fautif de n'avoir pas compris.

Lorsque quelqu'un baisse les bras et abandonne, ce n'est jamais
la faute des autres.

> Et je ne trouve pas si difficile que cela de retrouver ces
> constatations (ou si ce n'est redémontrer l'impossibilité, du moins
> constater qu'on ne trouve pas de solution naturelle en essayant un
> peu) par quelques expériences de pensée à partir de la connaissance
> du fait que les prédictions de la théorie quantique ont été
> vérifiées.

Quiconque n'a pas la présomption de penser que tout a déjà été
découvert, n'a aucune peine à comprendre pourquoi les explications
disponibles ne sont pas encore totalement claires.

> Peut-être cette recommandation pouvait être ressentie comme un dogme
> et une interdiction de réfléchir de par sa présentation et l'ambiance
> d'un certain cours,

C'est exactement ce qu'elle signifie, et c'est un reflet exact de
la réalité.

> mais ce genre d'aspect n'est alors plus du tout un problème de fond.
> Sinon, j'ai remarqué aussi l'ambiance de lavage de cerveau qui règne
> en prépa,

Tient donc!

> mais c'est un problème sociologico-politique qui n'a rien à
> voir avec une quelconque affaire d'imposition d'une théorie ou
> philosophie particulière.

Tient donc! Ça a à voir avec quoi alors!

D'après vous, on procède à ce lavage de cerveau pour ne pas
contraindre les étudiants à mieux comprendre qu'il est impossible
de comprendre? ... ou l'inverse, peut-être?

Pas très convaincant, votre argument.

> >"Minus copenhagensis" :
> >b) que si cette réalité physique existait tout de même, elle ne
> > pouvait être comprise que par Dieu (...) étant donné qu'ils se
> > considéraient eux-mêmes comme une sous-espèce d'homo sapiens
> > sapiens qui était incapable, contrairement à l'espèce mère, de
> >raisonner logiquement
>
> Il n'y a qu'une espèce d'hommes, tous capables de raisonner
> logiquement et ainsi de constater, à partir d'un minimum d'habileté
> d'esprit et s'ils veulent bien en faire l'effort, par quelle malice
> la réalité fondamentale se dérobe à toute tentative de la saisir en
> termes de causalité physique.

Sans vouloir vous offenser : connerie. Les hommes (et femmes) normaux
de notre espèce ne s'imaginent pas que la réalité fondamentale est
malicieuse et se dérobe volontairement à toute tentative de la
saisir en terme de causalité physique.

Ils attendent patiemment que ce qui nous sert d'élite en physique
fondamentale se lèvent de sur leur cul et se remettent au travail
pour leur apporter un peu plus de lumière sur les causes
fondamentales.

Malheureusement pour eux, ils ignorent que cette supposée "élite"
a abandonné la partie il y a 70 ans, et endocrine les générations
successives de physiciens à penser la même chose depuis ce temps.

J'ai décidé de jeter du sable dans l'engrenage, et croyez-moi
je m'y suis mis à tour de bras depuis 2 ans.



> > contrairement aux preuves expérimentales déjà connues et au
> > simple sens commun, toutes les particules étaient physiquement
> > des paquets d'ondes;
>
> Tout ce qui a été expérimenté est cohérent avec la théorie
> quantique.

Bien, indiquez nous donc comment l'insensibilité des quantités de
mouvement aux interactions transversales a été intégré à la théorie
quantique ? Nous vous écoutons.

> Quant au sens commun, tout le monde n'a pas fait joujou avec des
> particules quand il était petit pour savoir comment elles sont
> physiquement.

Sauf votre respect, personne, même dans notre si omnisciente
"élite" en physique fondamentale n'en sait encore assez pour
tirer quelque conclusion que ce soit, surtout pas celle qu'il
est impossible de les comprendre, que cette supposée "élite"
véhicule comme un dogme depuis 70 ans.



> >Le déclin de la confrérie fut fulgurant après que de nombreux
> >étudiants eurent pris connnaissance, dans des circonstances
> demeurées nébuleuses, d'importantes découvertes
>
> Si cela avait été le cas, ça se saurait et ils seraient nombreux
> à faire chacun un petit site web qui présente ces découvertes,

Mais il existe ce site web. Malheureusement, vous ne lisez pas
la langue de son auteur, qui n'est malheureusement plus des
nôtres. D'après ce que je connais de son caractère, je suis
certain qu'il aurait procédé exactement comme je suis en train
de le faire pour briser le cercle vicieux de la philosophie
défaitiste de l'école de Copenhague.

> ce qui est contradictoire avec l'affirmation plus haut du fait que
> ces fantomatiques découvertes ne sont pas trouvables sur Internet.

Mais elles sont tout à fait disponibles dans les textes originaux
des véritable auteurs. De Broglie est toujours publié, quoique
jamais traduit en anglais. Abraham est toujours disponible, quoique
seulement en allemand. Poincaré et les autres sont tous disponibles
en français.

> A moins bien sûr qu'il ne fasse là qu'un bluff d'anticipation de ce
> qu'il prétend qui arriverait à l'avenir, lorsque ce "dictionnaire du
> futur" serait publié.

Aucun bluff. Rien ne peut changer la réalité. La formulation du
futur dictionnaire sera probablement différente, mais les jours
de l'école de Copenhague sont comptés.

> Mais on entre alors dans un pur délire de science-fiction.

Merci pour votre imprimatur par incrédulité et abstention.

La diffusion continue.



> ------
>
> En conclusion, je vois mal comment on pourrait arrêter l'encombrement
> du newsgroup par ce genre de folies.

Enfin un peu de réalisme. Vous ne pouvez pas.

Le temps où l'élite avait droit de regard sur absolument tout ce
qui est diffusé est bien révolu.

> Ou peut-être une suggestion: s'il est difficile de censurer les
> perturbateurs (en mettant leur identité dans un killfile ou je ne
> sais quoi - il y en a déjà deux, am et rh), une bonne idée pourrait
> être d'ouvrir un nouveau newsgroup qui serait spécialement dédié
> aux gens de leur espèce et où ils pourraient librement échanger
> leurs idées fumeuses sans déranger ceux qui préfèrent discuter
> des théories communément acceptées.
>
> Par exemple quelque chose comme "fr.sci.physique.alternatives".
> Les suggestions sont ouvertes !

Vous pouvez toujours rêver. Depuis l'avénement de l'Internet, plus
personne ne peut restreindre le libre droit de parole qui est garanti
comme un droit fondamental par l'article 19 de la Déclaration
universelle des droits de l'Homme.

srp

non lue,
26 oct. 2002, 12:21:1526/10/2002
à
Moky a écrit :


> Et si on arrêtait de se battre à propos de l'interprétation à donner à
> la MQ ? De toute façon, on sait que les équations sont justes : QED
> donne la structure hyperfine de l'hydrogène avec plus de 20 décimales
> exactes !

Pas vraiment, c'est pas la QED qui donne cette structure, mais la MQ.

> Il est absolument impossible de nier la véracité d'une telle
> théorie.

La MQ n'est pas une théorie. C'est une simple méhtode ce calcul
des niveaux d'énergie dans l'atome d'hydrogène.

> Quelle que soit LA réalité, en ce qui concerne sa partie
> mesurable, elle doit se réduire à la MQ dans la limite où l'on
> se place dans son domaine d'application.

Pas plus qu'un véritable cheval ne peut se réduire à sa description
dans un dictionaire.



> En ce qui concerne Copenhage, son interprétaion est de toute façon
> fausse et inutile à discuter.

C'est pourquoi j'ai entrepris de lui scier les jambes sans plus de
cérémonies. Le temps des palabres futiles est révolu.

Place à l'action. Assez de temps a été perdu.

> Elle se base sur une théorie non relativiste de la MQ, nous avons à
> présent de bien meilleures théorie.

Toutes fondées sur le dogme de la philosophie de Copenhague voulant
que la réalité physique objective sous-jacente à toutes nos
descriptions n'existe pas.

> C'est à partir d'elles qu'il faut discuter et il se fait qu'en matière
> de contradiction avec le sens commun, elles sont encore pires que la
> MQ proprement dite.

Je suis en désaccord. C'est à partir de l'ensemble des constats
expérimentaux vérifiés qu'il faut discuter et tout rebâtir.



> Pour André :
>
> Feynmann ne voile rien du tout sous ses photons virtuels. Et en
> particulier pas l'interaction coulombienne.

Bien au contraire. Je vous suggère de lire sont papier original:

Space-Time Approach to Quantum Electrodynamics, Phys. Rev. 76, 769
(1949).

C'est Feynman lui-même qui les a nommé photons virtuels (ou vitual
quanta) et il indique explicitement qu'ils sont une métaphore
mathématique de l'interaction coulombienne sous-jacente, une
métaphore qui permet d'utiliser la méthode lagrangienne, plus
facile que la méthode hamiltonienne.

Il ne nie donc pas l'existence de l'interaction coulombienne. La
QED permet simplement d'en calculer plus facilement l'intensité
à des distances déterminées entre les particules en train
d'interagir.

> Car celle-ci n'existe plus ! Fini ! Coulomb est faux, Maxwell
> avait tors.

Eh ben, c'était pas l'avis de Feynman, ni de la majorité des
physiciens, même de nos jours.

> Ce ne sont que des approximations de la réalité. Celle que QED
> nous donne est bien plus performante.

Ce sont les photons virtuels qui sont des approximations, au
dire même de Feynman. Vous avez tout faux.

> Les photons virtuels ne sont jamais que des fonctions de Green qui
> arrivent dans le développement perturbatif des intégrales de chemin.
> Comme ces fonctions apparaîssent sous forme de propagateurs de
> Feynmann entre deux points de l'espace qui ne sont ni le point de
> départ ni le point d'arrivé du propagateur total, il nous a paru
> logique de les appeler "photon" aussi car ils font intervenir le même
> genre d'intégrales. Mais comme ces "photons" n'apparaîssent que entre
> les endroits où l'on mesure la position de la particule, on a cru bon
> de les appeler "virtuels".

Difficile de les appeler autrement, puisque c'est Feynman lui-même
qui les a nommé ainsi afin de pouvoir en discuter dans ses articles.

> Si ça dérange André de voir des photons virtuels et qu'il préfère
> voir la physique telle qu'elle est, il ne lui reste plus qu'à calculer
> des amplitudes sans recours au support visuel des diagrammes de
> Feynmann.

Ça ne me dérange pas du tout que les photons virtuels soient utilisés
dans les calculs. Ce qui me dérange est qu'on tente de faire croire
aux étudiants qu'ils sont autre chose qu'une métaphore mathématique
de la réalité sous-jacente.



> Disons les choses autrement. L'opérateur d'évolution en TQC
> (théorie quantique des champs) est en gros l'exponentielle du
> hamiltonien.

La théorie quantique des champs est la conséquence de la résistance
des physiciens de l'époque à abandonner l'idée des champs de la
théorie de Maxwell, or, vous avez dit un peu plus haute que Maxwell
avait tort.

Paradoxe!

D'autre part, nous savons depuis Wien, Planck, Einstein, Compton,
Raman, que les champs de cette nature n'existent tout simplement
pas. Par conséquent, la TQC ne peut être qu'une théorie mathématique
d'estimation globale sans fondement réel dans la réalité physique
objective au niveau micro, niveau auquel nous savons par preuve
expérimentale (Wien, Planck, Einstein, Compton, Raman) que toute
l'énergie se présente sous forme de quanta discrets au niveau
micro.

> Dans ce hamiltonien figure des champs qui correspondent à des
> particules réelles a priori et des produits de ces champs qui font
> les interactions.

Mathématiquement parlant, bien sûr. Mais ça ne change rien à la
nature quantique discrète confirmée de toute l'énergie au niveau
fondamental.

> Lors du développement de l'expo (que l'on met en braket pour
> calculer des amplitudes de transitions), il apparaît des termes qui
> sont exactement des propagateurs de particules entre des points
> intermédiaires.

Mathématiquement, c'est sans problème.

> Extraordinaire côïncidence ! Alors autant en profiter et appeler
> ces propagateurs "particules virtuelles" parcque mathématiquement ces
> termes ont la même forme.

Tout à fait. Sans problème.


> Ensuite, faut ouvrir les yeux : les mathématiciens ne sont pas du
> tout seuls maître à bord. Ce n'est pas parcqu'André ne comprend pas
> la mathématique liée à la physique qu'il doit croire qu'il n'y a
> qu'elle.

Je comprend parfaitement le rôle des mathématique en physique
fondamentale.

> En effet, la majorité des théorèmes liés par exemple aux intégrales
> de chemin de Feynmann ne sont pas encore démontrés de façon
> rigoureuse.

Tient donc! Pourtant, Boileau (je crois) n'a-t-il pas écrit :
"Ce qui se conçoit bien s'exprime clairement, et les mots pour
le dire viennent aisément"

Y aurait-il un os dans la "théorie" ?

> Ca ne nous empêche pas de les utiliser depuis plus de 40ans.

Bien sûr. Pourquoi nous en priver ? Mais ça ne change rien au fait
que plus personne en physique fondamentale ne cherche à comprendre
plus avant la réalité physique objective sous-jacente, puisque
plus personne dans ce milieu ne croit à son existence.

> Si il fallait attendre que les mathématiciens s'occupent de la
> physique, on en serait encore à se demander si l'hydrogène a un
> niveau fondamental.
> Je ne rigole pas : il y a moins de dix ans qu'on a démontré
> l'existence d'un niveau fondamental pour l'hydrogène non
> relativiste !

Eh ben, Je vous suggère de lire les papiers de Bohr des années 20.

C'est curieux tout de même qu'il ait déjà été au courant, lui, il
y a près de 80 ans, du niveau fondamental de l'hydrogène non
relativiste, vous ne trouvez pas, et qu'il ait même fondé toute
sa recherche sur cette connaissance ?

ricky

non lue,
26 oct. 2002, 17:48:0726/10/2002
à
hello

> La MQ n'est pas une théorie. C'est une simple méhtode ce calcul
> des niveaux d'énergie dans l'atome d'hydrogène.

ah je comprend mieux vos raisonnements :-)))
la MQ n'est pas une theorie , c'est juste une methode de calcul ;-)))

> Pas plus qu'un véritable cheval ne peut se réduire à sa description
> dans un dictionaire.

non mais la vision de ce cheval le peut

> C'est pourquoi j'ai entrepris de lui scier les jambes sans plus de
> cérémonies. Le temps des palabres futiles est révolu.

ah oui on scie les jambes sans toucher a une scie et sans parler ! la
methode pedagogique srp ;-)

> Place à l'action. Assez de temps a été perdu.

ah oui l'action ! celle de faire du copier coller ;-) ca fait vachement
avancer la physique ca...
oui je sais "pour les jeunes, etc etc ..." ... tu peut completer ;-)

> Toutes fondées sur le dogme de la philosophie de Copenhague voulant
> que la réalité physique objective sous-jacente à toutes nos
> descriptions n'existe pas.

voulant qu'il n'y ait pas de realite sous jacente a une theorie, ce qui est
tres different de ce que vous ecrivez.. mais bon, je sais que vous n'avez
pas encore appris a lire visiblement ;-)

> Ce sont les photons virtuels qui sont des approximations, au
> dire même de Feynman. Vous avez tout faux.

io n'a jamais ecrit cela ! il a ecrit que c'etait une facon de voir les
choses, une metaphore et en aucun cas une aprocimation de quoi que ce
soit...

> Ça ne me dérange pas du tout que les photons virtuels soient utilisés
> dans les calculs. Ce qui me dérange est qu'on tente de faire croire
> aux étudiants qu'ils sont autre chose qu'une métaphore mathématique
> de la réalité sous-jacente.

personne ne le fait
a par qu'il n'y a pas de realite sous jacente a une thoerie, mais une
realite qui se represente par de multiples observables chacun pouvant etre
decris par une theorie...

> D'autre part, nous savons depuis Wien, Planck, Einstein, Compton,
> Raman, que les champs de cette nature n'existent tout simplement
> pas.

vu qu'aucun physicien n'a jamais ecrit qu'un champ "existait", c'est normal
!

> Par conséquent, la TQC ne peut être qu'une théorie mathématique
> d'estimation globale sans fondement réel dans la réalité physique
> objective au niveau micro, niveau auquel nous savons par preuve
> expérimentale (Wien, Planck, Einstein, Compton, Raman) que toute
> l'énergie se présente sous forme de quanta discrets au niveau
> micro.

niveau auquel nous savons que nouspouvons le voir experimentalement sous
cette forme comme einstein , compton et autres l'on toujours dit (nul par
srp n'a ete fichu de trouver le mot de realite objetcive chez aucun de ces
physiciens !!!)
oui je sais "le jeunes, copier coller "

> C'est curieux tout de même qu'il ait déjà été au courant, lui, il
> y a près de 80 ans, du niveau fondamental de l'hydrogène non
> relativiste, vous ne trouvez pas, et qu'il ait même fondé toute
> sa recherche sur cette connaissance ?

relisez ses papier et vous verrez qu'il n'etait pas "au courant " mais qu'il
theorisait cela ... marrant d'essayer de faire dire aux autres ce qu'on veut
imposer ! pedagogie sans doute

@-
oui je sais "les jeunes, etc etc "
les pauvres ;-)


fda

non lue,
26 oct. 2002, 21:16:3326/10/2002
à

ricky wrote:

> ah je comprend mieux vos raisonnements :-)))
> la MQ n'est pas une theorie , c'est juste une methode de calcul ;-)))

C'est aussi ce que dit Feynman tout au début de son cours de MQ. En
substance : "j'ai un peu honte de vous présenter cela, car en physique,
il faut en principe comprendre ce que l'on fait et ce qui se passe; il
se trouve qu'avec la MQ nous ne pouvons tout simplement pas; tout ce que
nous pouvons faire, c'est tourner la manivelle à calculs et observer que
cela nous donne le bon résultat".

ricky

non lue,
26 oct. 2002, 20:16:5726/10/2002
à
bonjour

> C'est aussi ce que dit Feynman tout au début de son cours de MQ.

pas exactement !

En
> substance : "j'ai un peu honte de vous présenter cela, car en physique,
> il faut en principe comprendre ce que l'on fait et ce qui se passe; il
> se trouve qu'avec la MQ nous ne pouvons tout simplement pas; tout ce que
> nous pouvons faire, c'est tourner la manivelle à calculs et observer que
> cela nous donne le bon résultat".

oui d'ou le nom de THEORIE PROBABILISTE qui est son vrai nom :-)
la MQ est une theorie qui s'appuie sur un outil mathematique independant de
toute description interne de l'objet... elle ne cherche pas a comprendre ce
qu'est l'objet , mais elle encadre une facon de trouver l'observable a
partir de donnees externes ...
de fait, on utilise la manivelle a calcul pour resoudre les problemes qui
sont decrit par la theorie !

une theorie n'est pas forcement explicative du probleme lui meme... elle
peut se contenter de l 'encadrer ...

@+
ricky


fda

non lue,
26 oct. 2002, 22:55:4226/10/2002
à

ricky wrote:

> une theorie n'est pas forcement explicative du probleme lui meme... elle
> peut se contenter de l 'encadrer ...

C'était le cas des lois de Kepler. Par chance, on n'en est pas resté là :o)

Moky

non lue,
27 oct. 2002, 03:03:4427/10/2002
à
Redéfinissons la physique pour éviter les malentendus. Je peux
toujours me dire que la nature ne suit aucune loi et que Dieu choisi
au cas par cas les résultats des expériences. Cette théorie n'est pas
compliquée et rend parfaitement compte des TOUTES les expériences
passées ou à venir, et avec beaucoup plus de décimales exacte que 20,
et bonne chance pour me montrer expérimentalement qu'elle est fausse.
Pourtant, c'est une théorie peu usitée parmi les physiciens. Il n'est
donc pas suffisant de se dire qu'il faut une théorie qui rend
l'expérience.
La physqique cherche donc à trouver des modèles qui rendent compte
des expériences sans postuler l'existence de Dieu. Avec ça, il y a
encore des milliard de possibilités. On choisit "la plus simple". pour
l'instant la théorie la plus simple qui rend compte de toutes les
expériences, c'est le modèle standart avec la RG. Etant donné qu'on
croit en l'existence d'une théorie unifiée de la RG et des trois
autres interactions, on poursuit des recherches intensives pour la
trouver depuis pas mal de décénies. Et aujourd'hui, on a pas mal de
candidats. Mais tant qu'on ne l'a pas, il est inutile de se demander
si elle décrit une réalité 'objective' : les équations qu'on a pour
l'instant ne décrivent pas la réalité, mais une approximation qu'on
espère être bonne.

Quand je dis que la 'vraie' théorie doit se ramener à la MQ,
j'entend : les équations doivent se ramener à celles de la MQ ; leur
interprétation peut être bien différente.

Ce n'est pas tellement vrai que l'on nie la réalité sous-jacente à
nos théories : la MQ est extrêmement difficile à interpréter et,
souvent, on préfère suivre les équations comme guide pour construire
nos théories, sans s'occuper des interprétations à leur donner.

En ce qui concerne l'interaction Colombienne, je crois m'être mal
exprimé. Quand je dis qu'elle est fausse, je veux dire qu'elle n'est
qu'une approximation de la réalité. Il y a bien des cas où la théorie
de Coulomb donne de moins bons résultats que QED, mais aucun cas où
c'est l'inverse. Maintenant, si on veut utiliser l'expression
"interaction coulombienne" pour désigner tout ce qui est
électromagnétique, on peut effectivement dir que QED décrit
l'interaction coulombienne. Mais il n'en reste pas moins que QED n'est
pas une couche supplémentaire sur la théorie de Coulomb.


Bohr n'a pas calculé le niveau fondamental de l'hydrogène non
relativiste !
En fait, le hamiltonien étant un opérateur non borné (dans un espace
de Hilbert), l'existence d'une valeur propre minimale est
cauchemardesque à démontrer rigoureusement.
En particulier, lorsqu'on développe les solutions de l'eq de
schrodinger en ondes planes (ce qui est fait dans quasi tout les
bouquins de MQ), on a déjà tout faux car l'espace de Hilbert qui
contient les fonctions d'ondes est l'espace L² des fonctions de carré
sommable. Or, une onde plane n'est pas de carré sommable => en fait
les ondes planes ne forment pas une base des états physique car elle
ne sont même pas des états physique !
Aussi lourd que cela puisse paraître, il est effectivement vrai que
depuis 70 ans, presque AUCUN bouquin de physique n'a été écrit sans
fautes. La méthode de Feynman était appelée "intuitive" pour cette
raison : elle était encore moins rigoureuse que le reste de la
physique.

Ce n'est pas la MQ qui donne la structure hyperfine de l'hydrogène
mais bien la QED pcq il faut tenir compte des corrections radiatives
qui sont des effets relativistes. La MQ n'est pas une théorie
relativiste.

Et pour finir, la TQC n'est pas basée sur Maxwell. En fait, l'idée
est de trouver une équation d'onde relativiste (c'est à dire
invariante sous grosso modo le groupe de Lorentz) dont les solutions
seraient les fonctions d'onde de la MQ mais en mieux. Pour des tas de
raisons, il faut que les solutions forment des représentations du
groupe de Lorentz. Dès lors, sans chercher dans les cas pathologiques,
il n'y a que deux équations possibles : Klein-Gordon et Dirac.
Seulement, en étudiant les solutions, il est vite apparu qu'une
interprétation de ces solutions en tant qu'amplitude de probabilité ne
fonctionnerait pas. Mais heureusement, le formalisme de seconde
quantification donne quand même un contenu intuitif à ce qu'il se
passe.
Toute l'armada mathématique amène obligatoirement des
antiparticules, des particules virtuelles et des masses et charges
infinies ;il faut vivre avec jusqu'à trouver mieux.
Lorsqu'on essaye d'appliquer cette théorie au lagrangien de
Maxwelle, rien ne marche à cause de l'invariance de jauge. QED est
loin, mais très loin d'être "basée" sur Maxwell. En particulier, les
"champs" de la TQC n'ont rien à voir avec ceux de Maxwell, en tout cas
en ce qui concerne leur interprétation.
La théorie des supercordes n'est pas basée sur la RG : c'était un
bonus tout à fait inatendu de retrouver la gravitation à parir des
supersymétries.

Alors, j'en viens à poser prudement une question : qu'est-ce
qu'André sait de la physique ? As-t-il lu et compris la formulation de
la MQ avec les notations de Dirac, sait-il comment la TQC se développe
? Comment les particules virtuelles surgissent du développement
perturbatif des intégrales de chemin, a-t-il compris la manière dont
on calcule la structure hyperfine de l'hydroigène et tout et tout ???

Bonne journée,

fda

non lue,
27 oct. 2002, 05:27:2627/10/2002
à

Moky wrote:

> Je peux
> toujours me dire que la nature ne suit aucune loi et que Dieu choisi
> au cas par cas les résultats des expériences. Cette théorie n'est pas
> compliquée et rend parfaitement compte des TOUTES les expériences
> passées ou à venir, et avec beaucoup plus de décimales exacte que 20,
> et bonne chance pour me montrer expérimentalement qu'elle est fausse.

Laplace a répondu à cette objection il y a presque deux siècles :

- "Pourquoi, monsieur de Laplace, n'acceptez-vous pas de prendre en
compte l'hypothèse de l'existence de Dieu ? Elle permet d'expliquer
absolument TOUT"

- "C'est bien le problème, monsieur. Elle permet sans doute d'expliquer
TOUT, mais elle ne permet de prédire RIEN. Et, en ce qui me concerne,
c'est à la prévision des conséquences que je m'intéresse".


> Ce n'est pas tellement vrai que l'on nie la réalité sous-jacente à
> nos théories : la MQ est extrêmement difficile à interpréter et,
> souvent, on préfère suivre les équations comme guide pour construire
> nos théories, sans s'occuper des interprétations à leur donner.

Feynman indiquait dans son cours qu'il ne savait justement pas quelle
interprétation plausible en donner. Evidemment, la physique a dû
progresser depuis, mais on ne voit pas venir grand chose. Lui se
contentait d'un : "Tout ce que nous pouvons dire, c'est que la nature se
comporte comme cela, et qu'il faut faire avec".


> Alors, j'en viens à poser prudement une question : qu'est-ce

> que xxxx sait de la physique ? As-t-il lu et compris [etc]

Faute ! Nous sommes ici pour parler de physique et non de tel ou tel
intervenant du forum, sinon on n'en sortira jamais. En revanche, si vous
abstenir de ces digressions vous laisse du temps libre, alors révisez un
peu votre grammaire et votre orthographe. Tout le monde vous en sera
reconnaissant.

srp

non lue,
27 oct. 2002, 08:52:1927/10/2002
à
Pour les jeunes qui lisent ces lignes pour la première fois,
ricky est un triste individu qui m'a dénigré systématiquement
sur fsp pendant des mois, ainsi que tout le travail de vulgarisation
fait par le Service de Recherche Pédagogique pour l'éducation, parce
que je refuse systématiquement de discuter avec lui de physique,
car je suis un causaliste, comme de Broglie, Planck, Schrödinger,
Abraham et bien d'autres, et qu'ils appartiennent, comme la
majorité des physiciens contemporains, à l'école de pensée de
Copenhague.

ricky est intervenu obsessivement sur mes messages pendant des
mois, dans le but de me lasser pour éventuellement me faire

taire sur ce forum, en insinuant que des résistances passées
à d'autres présomptueux de son genre étaient une attitude

systématique de ma part envers tout le monde, et semble vouloir
s'y remettre.

Comme vous pourrez le constater si vous relisez les enfilades passées
dans lesquelles ces supposés "physiciens" se sont impliqués, ils

attaquent systématiquement, très agressivement et souvent avec rage
et mépris, (particulièrement dans le cas de ricky) tous ceux qui
tentent de donner aux jeunes les explications connues des phénomènes
physiques qui sont plus claires et simples que ce que la mécanique
quantique peut offrir.

La raison de leur acharnement particulier à mon endroit est que je

les ai dénoncé publiquement chaque fois qu'ils s'en sont pris à moi,

en référant les lecteurs à l'analyse que Franco Selleri a fait du
problème dans "Le grand débat de la théorie quantique", Flammarion,
que je conseille à tous les jeunes de lire, et parce que j'ai remis
en circulation en Amérique du Nord certaines découvertes du passé

qui ne sont jamais mentionnées dans les ouvrages de référence

qui sont mis à la disposition des jeunes dans les universités,
parce qu'elles contredisent leur dogmes fondamentaux.

Michel Talon, un élève de John Wheeler, qui ne s'est pas gêné

pour traiter Louis de Broglie et ses collègues de zozos sur ce
forum, nous a raconté ici même comment ses collègues ont réussi à

faire cesser tout enseignement causaliste au dernier endroit oû
un tel enseignement était donné sur la planète, à la suite de la

retraite de de Broglie, et comment au CERN, on avait fait fermer
le dernier projet de recherche qui ne cadrait pas avec la
philosophie de Copenhague.

Il y a cependant moyen d'apprendre la véritable physique
fondamentale, mais plus dans les facultés des universités, où
les programmes sont entièrement assujettis à la philosophie de
Copenhague depuis les années 60. Le seul moyen d'y parvenir
actuellement est de le faire hors du cadre académique.

À force de "discuter" avec ricky, j'ai fini par résumer, suite
à ses affirmations et confirmations, les principaux dogmes de
leur philosophie irrationnelle dans deux pseudo-définitions en
apparence caricaturales, mais absolument pas exagérées,
métaphoriquement destinées à paraître dans un dictionnaire du
futur, et que vous trouverez au lien mentionné ci-dessous.
--------------------------------------------------------
"École de pensée de Copenhague" :
--------------------------------------------------------
"Minus copenhagensis" :
--------------------------------------------------------

http://groups.google.com/groups?dq=&hl=fr&lr=&ie=UTF-8&threadm=3DA017E2.7F170F67%40globetrotter.net&prev=/groups%3Fhl%3Dfr%26group%3Dfr.sci.physique

--

srp

non lue,
27 oct. 2002, 09:16:1227/10/2002
à
Un dernier petit message à "Sylvain" de la part de "l'idiot" et
"hurluberlu" que vous avez "tenté de raisonner" pour "lui faire
voir son erreur".

Je vous disais que "je vous voyais venir à 100 km", mais je n'avais
pas terminé ma phrase. Je la complète donc ici pour l'édification
des jeunes qui s'interrogent sur notre "entretien".

Je vous voyais venir à 100 km avec votre intervention obséquieuse
dans un fil où ce m...... anonyme de ricky tentait de me faire
taire.

Même au "meilleurs de votre forme", vous ne faisiez pas le poids.

Votre dernier message est tout à fait typique des planqués
à la sauce de Copenhague qui finissent toujours par révéler
leur vraie nature en se retranchant dans l'insulte lorsqu'ils
perdent une discussion logique.

Le papier de Abraham est hors de la portée de tous les tenants
de l'école de Copenhague pour la simple raison qu'il exige un
effort de raisonnement et d'intégration, une faculté que tous
les inconditionnels de Copenhague ont dû abandonner au tout
début de leurs cours d'initiation à la mécanique quantique
saveur Copenhague, sous peine de couler leur cours.

Autrement dit, il faut se lever de sur son cul pour tenter de
comprendre quelque chose de nouveau, ce qui est hors de votre
portée et de celle de tous vos confrères minus copenhagensis,
et non attendre de vous faire biberonner et torcher comme le
font tous les incapables diplômés qui parasitent la physique
fondamentale actuellement.

Heureusement, les jeunes qui n'ont pas encore abandonné sont
toujours capable d'une telle intégration.

ricky

non lue,
27 oct. 2002, 10:12:3627/10/2002
à
bonjour

> - "C'est bien le problème, monsieur. Elle permet sans doute d'expliquer
> TOUT, mais elle ne permet de prédire RIEN. Et, en ce qui me concerne,
> c'est à la prévision des conséquences que je m'intéresse".

belle reponse de ce garcon ;-)
apres tout, il a bien resume le but de la physique ! que ce soit dieu ou
autre chose, ce qui compte c'est que cela nous fasse avancer dans nos
previsions ...

@+
ricky


ricky

non lue,
27 oct. 2002, 10:15:0227/10/2002
à
bonjour

> C'était le cas des lois de Kepler. Par chance, on n'en est pas resté là
:o)

:-)
c'etait quand meme un bon debut ;-)

@+
ricky


Moky

non lue,
27 oct. 2002, 10:45:0327/10/2002
à
J'ai dit un jour : "Dans ce hamiltonien figure des champs qui

correspondent à des particules réelles a priori et des produits de ces
champs qui font les interactions."

Srp m'a répondu :" Mathématiquement parlant, bien sûr. Mais ça ne


change rien à la nature quantique discrète confirmée de toute
l'énergie au niveau fondamental."

Alors je dit :
QED fait partie de la TQC et les TQC ne nient absolument pas la nature
discrète des quantas d'énergie. Ce n'est pas parcqu'un photon est
décrit par un champ vectoriel que sont interaction avec la matière ne
peut pas se faire de façon discrète.

Me trompe-je ?

D'ailleurs, si on croit à la nature discrète de la matière dans le
niveau le plus fondamental, pourquoi les expériences de Young
fonctionnent-elles avec des électrons et des protons,... ?
Il n'y a rien à y faire : l'effet photoélectrique PROUVE que la
lumière (et les électrons,...) ne sont PAS des ondes.
Young PROUVE que ces mêmes objets ne sont PAS des particules non
plus.
La MQ résoud astucieusement le paradoxe avec des fonctions d'ondes
et tout le bataclan. Alors soit on croit les équations et on ferme les
yeux sur les choses contreintuitives et on développe toutes les TQC.
Soit on essaye de comprendre ce qu'il se passe avant d'aller plus
loin, et on se retrouve 80 ans plus tard à encore essayer
d'interpreter Young/photoélectrique mais on n'avance pas.

Désolé de poster mes réflexions dans autant de messages disparates,
mais ainsi viennent-elles dans ma tête,

srp

non lue,
27 oct. 2002, 11:14:5327/10/2002
à
Moky a écrit :

>
> J'ai dit un jour : "Dans ce hamiltonien figure des champs qui
> correspondent à des particules réelles a priori et des produits de ces
> champs qui font les interactions."
>
> Srp m'a répondu :" Mathématiquement parlant, bien sûr. Mais ça ne
> change rien à la nature quantique discrète confirmée de toute
> l'énergie au niveau fondamental."
>
> Alors je dit :
> QED fait partie de la TQC et les TQC ne nient absolument pas la nature
> discrète des quantas d'énergie. Ce n'est pas parcqu'un photon est
> décrit par un champ vectoriel que sont interaction avec la matière ne
> peut pas se faire de façon discrète.
>
> Me trompe-je ?

Ce qui est illogique avec la TQC est la notion que des paires de
particules puissent apparaître spontannément hors du néan. Prémisse
invalide : théorie invalide.



> D'ailleurs, si on croit à la nature discrète de la matière dans le
> niveau le plus fondamental, pourquoi les expériences de Young
> fonctionnent-elles avec des électrons et des protons,... ?

Parce que les électrons et les protons existent vraiment, et qu'ils
ne peuvent pas passer du point A au point B sans parcourir une
trajectoire précise de moindre action entre les deux points.

Et contrairement à ce que vous avez écrit dans un autre message,
il est faux qu'il a été démontré qu'un électron solitaire passe
par les deux fentes de young en même temps. Il est compris et
vérifié expérimentalement que chaque électron (ou photon) individuel
passe par une ou l'autre des fentes.

Même John Wheeler, un des principaux gourous de l'école de
Copenhague en convient. (Magazine DISCOVER de juin 2002, page 46.

> Il n'y a rien à y faire : l'effet photoélectrique PROUVE que la
> lumière (et les électrons,...) ne sont PAS des ondes.

Exact.

> Young PROUVE que ces mêmes objets ne sont PAS des particules non
> plus.

Faux. L'expérience de young montre seulement que les photons qui
sont émis par la cible suite à l'arrivée d'innombrables particules
(photons ou électrons) par les fentes, le font selon un "pattern"
cohérent de franges d'interférences.

On observe le même genre de comportement organisé lorsqu'on projette
de la limaille de fer sur une feuille placée au dessus d'un aimant.
Personne n'en a conclu que la limaille se comportait comme des ondes.

Mon avis est qu'on comprendra le phénomène des franges de diffraction
seulement lorsque la mécanique électromagnétique des particules
fondamentales sera entièrement comprise.

> La MQ résoud astucieusement le paradoxe avec des fonctions d'ondes
> et tout le bataclan. Alors soit on croit les équations et on ferme
> les yeux sur les choses contreintuitives et on développe toutes les
> TQC.

Solution que je rejette.

> Soit on essaye de comprendre ce qu'il se passe avant d'aller plus
> loin, et on se retrouve 80 ans plus tard à encore essayer
> d'interpreter Young/photoélectrique mais on n'avance pas.

Exact. Si les inconditionnels de l'école de Copenhague ne nous
avaient pas fait perdre 70 ans de recherche, j'estime qu'on aurait
la réponse depuis environ 40 ans.

Malheureusement, il nous faudra attendre que la recherche reprenne
avec la prochaine génération.



> Désolé de poster mes réflexions dans autant de messages
> disparates, mais ainsi viennent-elles dans ma tête,

Sans problème. Je ne suis pas certain que j'aurai connaissance de
tous vos messages, cependant.



> Moky (fana de phys math)

--

fda

non lue,
27 oct. 2002, 16:42:4227/10/2002
à

> Ce qui est illogique avec la TQC est la notion que des paires de
> particules puissent apparaître spontannément hors du néan. Prémisse
> invalide : théorie invalide.

Dommage que ce soit observé. La réalité est décidément bien têtue.


> L'expérience de young montre seulement que les photons qui
> sont émis par la cible suite à l'arrivée d'innombrables particules
> (photons ou électrons) par les fentes, le font selon un "pattern"
> cohérent de franges d'interférences.

Y compris quand ces particules sont émises *une par une* excusez du peu.

On attend une explication ne faisant pas intervenir la MQ et ses amplitudes.
Pour le moment, on ne voit pas venir grand chose, à part de vagues discours.

srp

non lue,
27 oct. 2002, 15:04:4127/10/2002
à
fda a écrit :

>
> > Ce qui est illogique avec la TQC est la notion que des paires de
> > particules puissent apparaître spontannément hors du néan. Prémisse
> > invalide : théorie invalide.
>
> Dommage que ce soit observé. La réalité est décidément bien têtue.

Faux.

La seule manière connue pour produire une paire de particules
élémentaires est le découplage de photons d'énergie suffisante
déjà existants.

Prouvé expérimentalement par Joliot-Curie dans les années 30
à partir de photons gamma d'un seuil d'énergie de 1.022 MeV qui
frôlent des noyaux d'atomes lourds.

Prouvé expérimentalement pour des photons d'énergie d'au moins
1.022 MeV convergeant vers une zone suffisemment restreinte pour
interagir entre-eux, par l'équipe de McDonald à l'accélérateur
du SLAC en 1997.

Rien d'autre. Aucune particule ne peut être créée à partir
de rien.



> > L'expérience de young montre seulement que les photons qui
> > sont émis par la cible suite à l'arrivée d'innombrables particules
> > (photons ou électrons) par les fentes, le font selon un "pattern"
> > cohérent de franges d'interférences.
>
> Y compris quand ces particules sont émises *une par une* excusez
> du peu.

Oui, et alors!

Même chose pour des grains de limaille projetés un à un sur la
feuille qui couvre un aimant.

Cela ne fait toujours pas de la limaille un phénomène ondulatoire.

Et alors!



> On attend une explication ne faisant pas intervenir la MQ et ses
> amplitudes.
> Pour le moment, on ne voit pas venir grand chose, à part de vagues
> discours.

Déjà répondu:

Mon avis est qu'on comprendra le phénomène des franges de diffraction
seulement lorsque la mécanique électromagnétique des particules
fondamentales sera entièrement comprise.

Vivement que tous les minus copenhagensis soient mis à la retraite
pour que la recherche puisse reprendre.

fda

non lue,
27 oct. 2002, 17:14:0727/10/2002
à

srp wrote:

>>>Ce qui est illogique avec la TQC est la notion que des paires de
>>>particules puissent apparaître spontannément hors du néan. Prémisse
>>>invalide : théorie invalide.
>>
>>Dommage que ce soit observé. La réalité est décidément bien têtue.
>
> Faux.
>
> La seule manière connue pour produire une paire de particules
> élémentaires est le découplage de photons d'énergie suffisante
> déjà existants.

Bref, faux, mais vrai. Et à part ça, ce sont les autres qui selon vous
manquent de logique.

> Rien d'autre. Aucune particule ne peut être créée à partir
> de rien.

L'effet Casimir, ça vous dit quelque chose ?


>>>L'expérience de young montre seulement que les photons qui
>>>sont émis par la cible suite à l'arrivée d'innombrables particules
>>>(photons ou électrons) par les fentes, le font selon un "pattern"
>>>cohérent de franges d'interférences.
>>
>>Y compris quand ces particules sont émises *une par une* excusez
>>du peu.
>
> Oui, et alors!

Alors on se demande dans ce cas précis qu'est-ce qui interfère avec
quoi, pas vrai ?

> Même chose pour des grains de limaille projetés un à un sur la
> feuille qui couvre un aimant.

Phénomène sans rapport : Pour se mettre dans les mêms conditions
d'expérience, il faudrait enlever chaque grain de limaille avant
d'envoyer le suivant. Et là, je demande à voir où vous les verriez
apparaître, vos lignes de force, qui sont dues essentiellement à des
phénomènes d'attraction *locale* des grains de limaille entre eux - sans
correspondance aucune dans le cas des fentes d'Young.

srp

non lue,
27 oct. 2002, 16:31:4327/10/2002
à
fda a écrit :

>
> srp wrote:
>
> >>>Ce qui est illogique avec la TQC est la notion que des paires de
> >>>particules puissent apparaître spontannément hors du néan. Prémisse
> >>>invalide : théorie invalide.
> >>
> >>Dommage que ce soit observé. La réalité est décidément bien têtue.
> >
> > Faux.
> >
> > La seule manière connue pour produire une paire de particules
> > élémentaires est le découplage de photons d'énergie suffisante
> > déjà existants.
>
> Bref, faux, mais vrai. Et à part ça, ce sont les autres qui selon
> vous manquent de logique.

Faux que des paires de particules peuvent apparaître spontannément
du néan. Les seuls cas possibles le sont à partir du découplage de
photons déjà existants.

Vous n'avez qu'à lire les travaux de Joliot-Curie et les résultats
de McDonald. (Je sais: Trop difficile et hors de votre portée.)

Pas mon problème.

> > Rien d'autre. Aucune particule ne peut être créée à partir
> > de rien.
>
> L'effet Casimir, ça vous dit quelque chose ?

Déjà répondu:

L'effet Casimir s'explique très simplement par les interactions
électromagnétiques normales. Loi de l'inverse du carré de la
distance pour l'interaction coulombienne et loi de l'inverse du
cube de la distance pour l'interaction magnétostatique entre les
aspects magnétiques des particules, évidemment plus marquées
à courte distance.

Aucun besoin d'avoir recours à des "fluctuations quantiques
d'énergie" qui se matérialiseraient spontannément sortant de
nulle part, lubie jamais démontrée des inconditionnels de
l'école de Copenhague qu'ils tentent de prouver depuis 70 ans.

Je vous laisse la magie. Je préfère la physique.

> >>>L'expérience de young montre seulement que les photons qui
> >>>sont émis par la cible suite à l'arrivée d'innombrables particules
> >>>(photons ou électrons) par les fentes, le font selon un "pattern"
> >>>cohérent de franges d'interférences.
> >>
> >>Y compris quand ces particules sont émises *une par une* excusez
> >>du peu.
> >
> > Oui, et alors!
>
> Alors on se demande dans ce cas précis qu'est-ce qui interfère avec
> quoi, pas vrai ?

Déjà répondu.



> > Même chose pour des grains de limaille projetés un à un sur la
> > feuille qui couvre un aimant.
>
> Phénomène sans rapport :

Faux. Les deux phénomènes sont essentiellement des phénomènes
d'interaction électromagnétique. Toutes les particules élémentaires
dont l'existence physique a été prouvée par collision avec d'autres
particules sont de nature électromagétique.

> Pour se mettre dans les mêms conditions d'expérience, il faudrait
> enlever chaque grain de limaille avant d'envoyer le suivant.

Dans ce cas, dans l'expérience de young, il faudrait aussi enlever
chaque photon ou électron détecté avant d'envoyer le suivant.

Je demande alors à voir où vous les verriez apparaître, vos franges
d'interférence.

> Et là, je demande à voir où vous les verriez apparaître, vos lignes
> de force, qui sont dues essentiellement à des phénomènes d'attraction *locale* des grains de limaille entre eux -

- et avec le champ magnétique qui résulte de l'alignement parallèle
des spins de certains électrons de l'aimant.

> sans correspondance aucune dans le cas des fentes d'Young.

Déjà répondu. Relire le message précédent.

ricky

non lue,
27 oct. 2002, 17:05:3427/10/2002
à
bonjour

> Ce qui est illogique avec la TQC est la notion que des paires de
> particules puissent apparaître spontannément hors du néan. Prémisse
> invalide : théorie invalide.

les premices ne sont pas invalides, les divers theories le permettant sans
mettre a mal l'observable ... pire, grace a cela, on arrive a predir des
phenomenes ce qui est la base de la physique

il faut simplement que le principe d'equilibre soit respecter.
en gros, le temps d'existence avant anihilation de cette paire est
inversemenbt proportionnel a l'energie de cette paire

donc la theorie n'a pas ete prouvee invalide ...

> par les deux fentes de young en même temps. Il est compris et
> vérifié expérimentalement que chaque électron (ou photon) individuel
> passe par une ou l'autre des fentes.

absolument pas...
il a ete demontrer que l'electron, comme le photon est autre chose que
simplement cela...

> Même John Wheeler, un des principaux gourous de l'école de
> Copenhague en convient. (Magazine DISCOVER de juin 2002, page 46.

il n'y a pas de gourou d'une ecole de pensee... un grand pedagogue devrait
savoir cela, a moins d'etre gourou lui meme...

> Faux. L'expérience de young montre seulement que les photons qui
> sont émis par la cible suite à l'arrivée d'innombrables particules
> (photons ou électrons) par les fentes, le font selon un "pattern"
> cohérent de franges d'interférences.

non cela est seulement dans le cas de la fente unique
dans le cas des deux fentes, on montre bien qu'il y a interference...

et le comportement different des deux cas montre que votre interlocuteur a
raison

> On observe le même genre de comportement organisé lorsqu'on projette
> de la limaille de fer sur une feuille placée au dessus d'un aimant.
> Personne n'en a conclu que la limaille se comportait comme des ondes.

n'importe quoi , cette comparaison est sans rapport .. un pedagogue de votre
classe confondre le support (le champ de l'aimant) et le test (la limaille)
....
cette comparaison prouve votre raisonnement circulaire classique : pour etre
valide, il presuppose qu'on pense le photon particulaire !! et il demontrz
alors que le photon est particulaire :-)

1) la limaille de fer suit un champ dont on montre qu'il est bien une onde !
or les photons sont l'image de ce champ et non de la limaille !
2) la limaille se comporte toujours de la meme facon, qu'on la projette par
un trou, deux trous ou en vrac, pas les photons

comparaison n'est pas raison !

> Exact. Si les inconditionnels de l'école de Copenhague ne nous
> avaient pas fait perdre 70 ans de recherche, j'estime qu'on aurait
> la réponse depuis environ 40 ans.

oui vous etes tres doues pour estimer, inventer , ecrire tout ce qui n'est
pas verifiable
ce qui est verifiable c'est qu'il n'y a pas eu perte de temps

> Malheureusement, il nous faudra attendre que la recherche reprenne
> avec la prochaine génération.

bien sur

@-


fda

non lue,
27 oct. 2002, 20:24:4027/10/2002
à

srp wrote:

>>>Même chose pour des grains de limaille projetés un à un sur la
>>>feuille qui couvre un aimant.
>>
>>Phénomène sans rapport :
>
> Faux. Les deux phénomènes sont essentiellement des phénomènes
> d'interaction électromagnétique. Toutes les particules élémentaires
> dont l'existence physique a été prouvée par collision avec d'autres
> particules sont de nature électromagétique.

Vos grains de limaille s'influencent magnétiquement les uns les autres,
ce qui les amène fort naturellement à se regrouper le long de ce qui
vous semble être des ligne de force (en fait, la *position* de ces
lignes n'a rien de prévisible - à la différence de leur allure générale
- et dépend essentiellement de l'endroit où sont tombés les premiers
grains de limaille.

*Rien à voir* donc avec les franges d'interférences des photons. Au
demeurant, il suffit de faire l'expérience pour constater que les grains
ne tombent pas spontanément sur ces lignes : ils viennent s'y placer, et
d'autant plus docilement qu'on a le bon goût de tapoter le carton pour
cela. faires la même chose avec des photons on vous souhaite bien du
plaisir :o)))

>>Pour se mettre dans les mêms conditions d'expérience, il faudrait
>>enlever chaque grain de limaille avant d'envoyer le suivant.
>
> Dans ce cas, dans l'expérience de young, il faudrait aussi enlever
> chaque photon ou électron détecté avant d'envoyer le suivant.

C'est bien ce qui se passe. Pour votre gouverne, quand un photon est
détecté, il n'existe plus en tant que photon. On pouvait espérer que
vous saviez au moins ça.

srp

non lue,
27 oct. 2002, 19:52:0327/10/2002
à
fda a écrit :

>
> srp wrote:
>
> >>>Même chose pour des grains de limaille projetés un à un sur la
> >>>feuille qui couvre un aimant.
> >>
> >>Phénomène sans rapport :
> >
> > Faux. Les deux phénomènes sont essentiellement des phénomènes
> > d'interaction électromagnétique. Toutes les particules élémentaires
> > dont l'existence physique a été prouvée par collision avec d'autres
> > particules sont de nature électromagétique.
>
> Vos grains de limaille s'influencent magnétiquement les uns les
> autres, ce qui les amène fort naturellement à se regrouper le long
> de ce qui vous semble être des ligne de force (en fait, la *position*
> de ces lignes n'a rien de prévisible - à la différence de leur allure
> générale - et dépend essentiellement de l'endroit où sont tombés les
> premiers grains de limaille.
>
> *Rien à voir* donc avec les franges d'interférences des photons. Au
> demeurant, il suffit de faire l'expérience pour constater que les
> grains ne tombent pas spontanément sur ces lignes : ils viennent s'y
> placer, et d'autant plus docilement qu'on a le bon goût de tapoter le
> carton pour cela. faires la même chose avec des photons on vous
> souhaite bien du plaisir :o)))

Vous pensez donc que les photons se comportent comme des grains de
limaille de fer? On aurait pu s'attendre à un niveau de connaissance
un peu plus profond d'un prétendu "physicien".


> >>Pour se mettre dans les mêms conditions d'expérience, il faudrait
> >>enlever chaque grain de limaille avant d'envoyer le suivant.
> >
> > Dans ce cas, dans l'expérience de young, il faudrait aussi enlever
> > chaque photon ou électron détecté avant d'envoyer le suivant.
>
> C'est bien ce qui se passe. Pour votre gouverne, quand un photon est
> détecté, il n'existe plus en tant que photon. On pouvait espérer que
> vous saviez au moins ça.

On aurait pu penser qu'un prétendu "physicien" était au courant
que les électrons qui sont capturés dans l'expérience de Young ne
cessent pas d'exister. Mais bien sûr, quand on croit à la magie,
tout est possible, n'est-ce pas?

Vous prétendez donc avoir compris à fond toute la mécanique
électromagnétique des particules qui permettrait de l'expliquer.

Alors, nous vous écoutons.

Et vos "fluctuations quantiques d'énergie" pour l'effet Casimir,
parlez-nous donc de la création spontannée et magique des paires
à partir du vide.

J'imagine que vous croyez aussi aux monopoles magnétiques et
aux trous noirs.

Pourquoi ne pas nous en glisser un petit aperçu.

srp

non lue,
27 oct. 2002, 19:55:1227/10/2002
à
Pour les jeunes qui lisent ces lignes pour la première fois,
ricky est un triste individu qui m'a dénigré systématiquement
sur fsp pendant des mois, ainsi que tout le travail de vulgarisation
fait par le Service de Recherche Pédagogique pour l'éducation, parce
que je refuse systématiquement de discuter avec lui de physique,
car je suis un causaliste, comme de Broglie, Planck, Schrödinger,
Abraham et bien d'autres, et qu'ils appartiennent, comme la
majorité des physiciens contemporains, à l'école de pensée de
Copenhague.

Il est intervenu obsessivement sur mes messages pendant des

http://groups.google.com/groups?dq=&hl=fr&lr=&ie=UTF-8&threadm=3DA017E2.7F170F67%40globetrotter.net&prev=/groups%3Fhl%3Dfr%26group%3Dfr.sci.physique

--

fda

non lue,
27 oct. 2002, 22:22:3627/10/2002
à
srp wrote:

>>>>>Même chose pour des grains de limaille projetés un à un sur la
>>>>>feuille qui couvre un aimant.
>>>>
>>>>Phénomène sans rapport :
>>>
>>>Faux. Les deux phénomènes sont essentiellement des phénomènes
>>>d'interaction électromagnétique. Toutes les particules élémentaires
>>>dont l'existence physique a été prouvée par collision avec d'autres
>>>particules sont de nature électromagétique.
>>
>>Vos grains de limaille s'influencent magnétiquement les uns les
>>autres, ce qui les amène fort naturellement à se regrouper le long
>>de ce qui vous semble être des ligne de force (en fait, la *position*
>>de ces lignes n'a rien de prévisible - à la différence de leur allure
>>générale - et dépend essentiellement de l'endroit où sont tombés les
>>premiers grains de limaille.
>>
>>*Rien à voir* donc avec les franges d'interférences des photons. Au
>>demeurant, il suffit de faire l'expérience pour constater que les
>>grains ne tombent pas spontanément sur ces lignes : ils viennent s'y
>>placer, et d'autant plus docilement qu'on a le bon goût de tapoter le
>>carton pour cela. faires la même chose avec des photons on vous
>>souhaite bien du plaisir :o)))
>
>
> Vous pensez donc que les photons se comportent comme des grains de
> limaille de fer? On aurait pu s'attendre à un niveau de connaissance
> un peu plus profond d'un prétendu "physicien".

Non, c'est vous qui le pensez : "Même chose pour les grains de
limaille", c'est vous. "Phénomène sans rapport", c'est moi. "Rien à
voir", c'est moi aussi. Vous ne savez donc pas lire ? On aurait pu
s'attendre à un niveau de connaissance un peu plus profond sur un forum.


>>'est bien ce qui se passe. Pour votre gouverne, quand un photon est
>>détecté, il n'existe plus en tant que photon. On pouvait espérer que
>>vous saviez au moins ça.
>
> On aurait pu penser qu'un prétendu "physicien" était au courant
> que les électrons qui sont capturés dans l'expérience de Young ne
> cessent pas d'exister.

Ben voyons ! Où sont-ils, d'après vous, les photons, après avoir frappé
le détecteur, qu'on s'amuse un peu ? Du côté de Sirius ? Où se
décident-ils brusquement à faire du sur place en attendant docilement
que les autres photons arrivent derrière eux ? On a ses diplômes de
physique dans des pochettes-surprise, au Canada ?

> J'imagine que vous croyez aussi aux monopoles magnétiques et
> aux trous noirs.

Il n'est pas question de "croire", mais de constater qu'en effet puisque
la gravitation dévie la lumière (peut-être n'êtes-vous pas au courant?),
on ne voit pas bien pourquoi à partir d'un certain seuil elle ne se
mettrait pas à la retenir. Les croyances, c'est vous, pas moi.

srp

non lue,
27 oct. 2002, 21:54:1027/10/2002
à
fda a écrit :

Pas du tout. C'est vous qui avez dit: "faires la même chose avec des
photons", c'est pas moi. Vous pensez donc que les photons se comportent
comme des grains de limaille de fer.

> >>'est bien ce qui se passe. Pour votre gouverne, quand un photon est
> >>détecté, il n'existe plus en tant que photon. On pouvait espérer que
> >>vous saviez au moins ça.
> >
> > On aurait pu penser qu'un prétendu "physicien" était au courant
> > que les électrons qui sont capturés dans l'expérience de Young ne
> > cessent pas d'exister.
>
> Ben voyons ! Où sont-ils, d'après vous, les photons, après avoir
> frappé le détecteur, qu'on s'amuse un peu ? Du côté de Sirius ? Où
> se décident-ils brusquement à faire du sur place en attendant
> docilement que les autres photons arrivent derrière eux ? On a ses
> diplômes de physique dans des pochettes-surprise, au Canada ?

Un peu dur de la feuille le mec. Ça fait 3 fois que je lui répète

qu'on comprendra le phénomène des franges de diffraction seulement

lorsque la mécanique électromagnétique des particules fondamentales
sera entièrement comprise.

C'est pourquoi il faut mettre à la retraite au plus vite tous les
petits minus copenhagensis qui empêchent le progrès pour que la
recherche puisse reprendre.

C'est clair non!

> > J'imagine que vous croyez aussi aux monopoles magnétiques et
> > aux trous noirs.
>
> Il n'est pas question de "croire", mais de constater qu'en effet
> puisque la gravitation dévie la lumière (peut-être n'êtes-vous pas
> au courant?), on ne voit pas bien pourquoi à partir d'un certain
> seuil elle ne se mettrait pas à la retenir. Les croyances, c'est
> vous, pas moi.

Connerie irrationnelle.

Parlez-nous des monopoles magnétiques, qu'on rigole un peu.

fda

non lue,
28 oct. 2002, 03:10:3428/10/2002
à

srp wrote:

>>Non, c'est vous qui le pensez : "Même chose pour les grains de
>>limaille", c'est vous. "Phénomène sans rapport", c'est moi. "Rien à
>>voir", c'est moi aussi. Vous ne savez donc pas lire ? On aurait pu
>>s'attendre à un niveau de connaissance un peu plus profond sur un
>>forum.
>
> Pas du tout. C'est vous qui avez dit: "faires la même chose avec des
> photons", c'est pas moi. Vous pensez donc que les photons se comportent
> comme des grains de limaille de fer.

Vous me confirmez que l'usage du "donc" ne devrait pas être autorisé en
dessous d'un certain quotient intellectuel.

srp

non lue,
28 oct. 2002, 07:00:5928/10/2002
à
fda a écrit :

Vous êtes vraiment stupide alors, c'était pas seulement un air
que vous vous donniez pour la galerie ?

Bon, enlevons donc le "donc" pour vous simplifier la tâche. Vous
pensez [] que les photons se comportent comme des grains de limaille
de fer puisque vous avez dit "faites la même chose avec des photons".

En fait, j'ai répondu clairement à toutes vos questions.

Je résume donc:

La création spontannée de paires de particules à partir du néan
total est une connerie irrationnelle avec laquelle se bercent
tous les minus copenhagensis pour s'endormir chaque soir. Tous
les autres savent, parce que la chose a été confirmée
expérimentalement par Joliot-Curie dans les années 30, et l'équipe
de McDonald en 1997, que les paires électron/positon ne peuvent être
produites que par découplage de photons DÉJÀ EXISTANTS d'énergie
1.022 MeV ou plus.

Même si la limaille forme un pattern cohérent sur la feuille au
dessus de l'aimant, personne n'a été assez stupide pour penser
que la limaille se comportait comme une onde.

Dans le cas de l'expérience de Young, malgré les preuves béton
que nous avons depuis 100 ans que les photons sont des particules
électromagnétiques discrètes (Wien-Planck-Einstein-Compton-Raman),
les minus copenhagensis ont continué à croire que les photons
se comportent comme des ondes, trop bouchés qu'ils étaient et
sont encore pour accepter l'évidence, connerie qu'ils continuent
à enseigner et à marteler comme un dogme par lavage de cerveau
chez génération après génération d'étudiants en physique depuis
70 ans.

La solution:

Puisque tous les minus copenhagensis sont stupides par
autodéconnexion du lobe frontal au point qu'il a été impossible
de leur faire comprendre depuis 70 ans qu'il faut reprendre la
recherche pour finir de comprendre la mécanique électromagnétique
des particules qui nous donnerait l'explication que tout le monde
cherche (sauf les minus copenhagensis, bien sûr), il faut donc que
tous les petits minus copenhagensis soient mis à la retraite au plus
tôt pour que de véritables physiciens puissent reprendre la recherche
en remplacement de la bande d'incompétents diplômés à la sauce
Copenhague qui empêchent tout progrès depuis 70 ans.

C'est clair non?

Mais je suis certain que stupide comme vous avez démontré l'être,
vous n'aurez toujours pas compris. Pas mon problème.

Didier Lauwaert

non lue,
28 oct. 2002, 10:15:2428/10/2002
à
fda <vacances@bali> wrote in message news:<3DBCF0FA.9000608@bali>...

> Vous me confirmez que l'usage du "donc" ne devrait pas être autorisé en
> dessous d'un certain quotient intellectuel.

:-)

Salut, je me permet d'intervenir
(tu ne m'en voudras pas ;-)

C'est amusant ta discussion avec srp.

Il compare les grains de limaille aux franges d'interférence,
tu analyses l'aspect expérimental avec les grains
puis tu ironises sur ses remarques en lui proposant
(c'est amusant) de faire le même avec des photons.
Et... paf ! Il t'accuse de confondre grains de limaille et photons !

C'est trop fort !!!

Mieux encore (et même à tomber le c... par terre) :

Tu affirmes, que les photons disparaissent après détection
et il te réplique que... tu affirmes que les électrons
disparaissent après interaction !!!!!
(et il t'insulte par la même occasion, cela ne me surprend pas
que tu t'énerves quelque peu ;-),
d'autant que se faire traiter de "soit disant physicien" par
quelqu'un qui s'y connait si mal en physique, c'est vexant :-)))

Fumant, non ?
(à mon avis il croit que tes smiley sont des signes de ponctuations)

Et ce n'est qu'un extrait infime.

Franchement, avec un tel dialogue de sourd, comment peux-tu
encore argumenter ?
Le dialogue comporte des remarques intéressante (des deux cotés
d'ailleurs, faut bien le dire) pris message par message. Mais globalement,
le fil est abérant !!!!!
On pourrait objecter que ceux qui lisent ce fil
peuvent au moins se faire une idée des intervenants
(après y avoir réfléchi, c'est à dire après avoir
repris son soufle après avoir beaucoup ri ;-)
malheureusement ce fil est franchement trop lourd !

Je te conseille le truc de la grosse majorité des participants
---> le kill file :-)))
Vous n'êtes plus quelque trois ou quatre à lire ses messages
(sur fsp du moins).

ricky

non lue,
28 oct. 2002, 10:27:5828/10/2002
à
bonjour


> Il compare les grains de limaille aux franges d'interférence,
> tu analyses l'aspect expérimental avec les grains
> puis tu ironises sur ses remarques en lui proposant
> (c'est amusant) de faire le même avec des photons.
> Et... paf ! Il t'accuse de confondre grains de limaille et photons !
>
> C'est trop fort !!!

tu as remarque aussi ? :-)
y a du melange ;-)))

> d'autant que se faire traiter de "soit disant physicien" par
> quelqu'un qui s'y connait si mal en physique, c'est vexant :-)))

ce sreait plutot amusant souvent si on n'etait pas sur un forum non ? ;-)

> Vous n'êtes plus quelque trois ou quatre à lire ses messages
> (sur fsp du moins).

il a promis de ne polluer qu'un forum a la fois ;-)

euh attention a fsp, on risque de croire qu'il est aussi abonne a
fr.soc.politique ;-)))

et la c'est pas la meme ambiance si j'ai bien tout suivi ... meme avec srp,
c'est un monde de douceur ici, comme quoi la science rend quand meme plus
aimable que la politique ;-))))

@+
ricky

Moky

non lue,
28 oct. 2002, 12:25:4528/10/2002
à
>
> > Young PROUVE que ces mêmes objets ne sont PAS des particules non
> > plus.
>
> Faux. L'expérience de young montre seulement que les photons qui
> sont émis par la cible suite à l'arrivée d'innombrables particules
> (photons ou électrons) par les fentes, le font selon un "pattern"
> cohérent de franges d'interférences.
>
> On observe le même genre de comportement organisé lorsqu'on projette
> de la limaille de fer sur une feuille placée au dessus d'un aimant.
> Personne n'en a conclu que la limaille se comportait comme des ondes.
>
> Mon avis est qu'on comprendra le phénomène des franges de diffraction
> seulement lorsque la mécanique électromagnétique des particules
> fondamentales sera entièrement comprise.
>

J'ai pourtant fort l'impression qu'on soit parvenu à trouver des
franges d'interférence avec des électrons envoyés un par un. Je lit
dans mon cours de MQ : "Récemment en 1989, Tonomura et son équipe et
son équipe au laboratoire Hitachi du Japon..." Le reste est de la
description du montage expérimental. Toujours est-il qu'on a des
photos successives avec 10, 100,3000,20000,70000 impacts d'électrons.
Sur la figure à 10 électrons, on n'a aucune structure nette, sur celle
à 70000 on a clairement les interférences. Je crois que c'est fini,
sans espoir, de se dire que l'électron passe par un trou ou l'autre.
Il est expérimentalement quasi sans appel de se dire au minimum que
l'électron interfère avec lui-même (je ne dit pas pour autant qu'il
passe par les deux en même temps quoi que je le pense).
La référence donnée est : A. Tonomura, J. Endo, T. Matsuda et H.
Ezawa,
Demonstration of single-electron buildup of an interference pattern,
American Journal of Physics 57 (1989)117.


Et si je citait Feynman qui dit explicitement qu'il est faux de
croire que l'électron passe SOIT par l'un SOIT par l'autre, je suppose
que ça ne te convaincra pas plus et tu aurais raison : les citations
sont des arguments d'autorité. Mais bon...


Il faut par ailleurs nuancer le fait que la TQC permet de créer des
particules du "néant". Le seul néant qui existe en TQC est le vecteur
appelé ‘vide' qui n'est que le vecteur propre du hamiltonien avec la
plus basse valeur propre. Bon ça n'a pas beaucoup d'importance. Il y a
deux types de particules ‘créées' :
1) Celles qui sont créées à partir d'énergie (cinétique par exemple),
avec conservation de tout les bons nombres quantique. Celles-là
restent et on peut avoir ainsi multiplication du nombre de particules
sans que cela ne choque vraiment.
2) Les particules ‘virtuelles'. Celles-là, à part peut-être dans
certains cas, je ne suis plus sûr, ne respectent pas la condition
p²=m², sont effectivement créées à partir de ‘rien', mais elles ne
sont pas mesurables en ce sens qu'elles disparaissent dès qu'on essaye
de les voir. Ce sont, techniquement parlant, des lignes internes aux
graphes de Feynman. Mais ces particules qui partent du vide retournent
au vide, sans être mesurables, ce sont en fait plutôt des
intermédiaires de calcul.
Mais ces particules sont intéressantes à considérer car elles
permettent une chouette interprétation physique de la normalisation de
la charge en termes de polarisation du vide.


Ne t'inquiète donc pas, srp, on n'a pas perdu 70 ans de physique ;
peut-être 70 ans de philo des sciences. Mais les théories qui ont été
construites depuis les débuts de la MQ sont trop bonnes (sur le plan
expérimental) pour ne pas être au minimum une bonne avancée dans la
recherche des lois de la nature. La prédiction de l'existence du boson
de Higgs est superbe, la prédiction des anti-particules, du quark
top,… On ne peut pas nier que la réalité se trouve plus proche du
modèle standard que de Newton et Maxwell.
Et sur le plan théorique, elles sont trop belles pour être fausse.
Mais ça c'est pas un argument qui fait mouche sur les ‘non initiés'.


Bonne soirée,

Moky

non lue,
28 oct. 2002, 12:48:4728/10/2002
à
Tout ces discours dont embrouillés. Je ne sais pas qui à dit quoi...
Mais voici mes réflexions :

...mécanique électromagnétique des particules fondamentales
sera entièrement comprise.


Les particules élémentaires ne font pas d'électromagnétisme. La
mécanique des particules élémentaires doit être comprise au minimum au
niveau de l'unification électrofaible (c'est la plus profonde théorie
ayant de bonne confirmations expérimentales en la matière).


...J'imagine que vous croyez aussi aux monopoles magnétiques et
aux trous noirs.

Moi, je n'afirmerait rien à propos des monopôles magnétiques :
certaines théories de grande unification les plus en vogue
(électrofaible + forte) ont des solutions qui sont des monopôles
magnétiques. D'autre part, à ce qu'il paraît, on en a besoin pour
expliquer la période d'inflation post-big bang. Quant aux trous noirs,
... je ne suis pas sûr qu'on en ait 'vu' (si ce mot à une sens), mais
je ne compterais pas sur le fait qu'ils n'existent pas.


> >>>Faux. Les deux phénomènes sont essentiellement des phénomènes
> >>>d'interaction électromagnétique. Toutes les particules élémentaires
> >>>dont l'existence physique a été prouvée par collision avec d'autres
> >>>particules sont de nature électromagétique.
> >>
> >>Vos grains de limaille s'influencent magnétiquement les uns les
> >>autres, ce qui les amène fort naturellement à se regrouper le long
> >>de ce qui vous semble être des ligne de force (en fait, la
> >>*position* de ces lignes n'a rien de prévisible - à la différence
> >>de leur allure générale - et dépend essentiellement de l'endroit
> >>où sont tombés les premiers grains de limaille. *Rien à voir* donc avec les franges d'interférences des photons. Au
> >>demeurant, il suffit de faire l'expérience pour constater que les
> >>grains ne tombent pas spontanément sur ces lignes : ils viennent s'y
> >>placer, et d'autant plus docilement qu'on a le bon goût de tapoter
> >>le carton pour cela. faires la même chose avec des photons on vous
> >>souhaite bien du plaisir :o)))


Si je puis amener mon grain de sel à propos de la limaille de fer,
je dirais que ça n'a effectivenemt rien à voir avec Young. En effet,
la position que les lignes de champ va prendre avec la limaille n'est
pas déterminée. Refaites deux fois la même expression, des lignes de
champ vont se déplacer de quelques milimètres. Young pas : les franges
sont toujours au même endroit et elles sont déterminée par la
géométrie des trous.
Young n'est pas électromagnétique parcque ça marche aussi avec des
neutrons et des atomes neutres.
Sans savoir qui a dit lequel des eux bouts, je suis donc 100%
d'accord avec le second et pas d'accord avec le premier.


...Il n'est pas question de "croire", mais de constater qu'en effet


> puisque la gravitation dévie la lumière (peut-être n'êtes-vous pas
> au courant?), on ne voit pas bien pourquoi à partir d'un certain
> seuil elle ne se mettrait pas à la retenir.

Avec ça je ne suis qu'à moitié d'accord. Les trous noirs ont une
motivation plus profonde : les équations de la RG possèdent
effectivemet des solutions de collapse gravitationel. J'en saurai plus
dans quelque semaines que mon cours de RG sera fini.

...Parlez-nous des monopoles magnétiques, qu'on rigole un peu.
Là je suis d'accord. Si on commence à s'empoigner sur les
monopôles, on risque de bien se marrer : ils existent dans des
théories qui ne sont encore pas du tout testées (quelques indications
indirectes seulement), et on aurait aimé les voir chez Maxwell pour la
beauté de l'équation. Alors, sur quel matériel allons-nous discuter
???

Bonne journée à tous.
Moky (...)

Chargement d'autres messages en cours.
0 nouveau message